Sie sind auf Seite 1von 222

www.Vidyarthiplus.

com

Unit-1 AIRCRAFT GAS TURBINES


Impulse and reaction blading of gas turbines – Velocity triangles and power output – Elementary theory –
Vortex theory – Choice of blade profile, pitch and chord – Estimation of stage performance – Limiting factors
in gas turbine design- Overall turbine performance – Methods of blade cooling – Matching of turbine and
compressor – Numerical problems.

Axial and radial flow turbines


As with the compressor, there are two basic types of turbine—radial flow and axial flow. The vast majority of
gas turbines employ the axial flow turbine.

The radial turbine can handle low mass flows more efficiently than the axial flow machine and has been widely
used in the cryogenic industry as a turbo-expander, and in turbochargers for reciprocating engines. Although
for all but the lowest powers the axial flow turbine is normally the more efficient, when mounted back-to-back
with a centrifugal compressor the radial turbine offers the benefit of a very short and rigid rotor. This
configuration is eminently suitable for gas turbines where compactness is more important than low fuel
consumption. Auxiliary power units for aircraft (APUs), generating sets of up to 3 MW, and mobile power
plants are typical applications.

Impulse and Reaction Turbine

Work can be extracted from a gas at a higher inlet pressure to the lower back pressure by allowing
it to flow through a turbine. In a turbine as the gas passes through, it expands. The work done by the gas is
equivalent to the change of its enthalpy.

www.Vidyarthiplus.com
www.Vidyarthiplus.com

It is a well known fact that the turbines operate on the momentum principle. Part of the energy of the gas
during expansion is converted into kinetic energy in the flow nozzles. The gas leaves these stationary nozzles
at a relatively higher velocity. Then it is made to impinge on the blades over the turbine rotor or wheel.
Momentum imparted to the blades turns the wheel. Thus, the two primary parts of the turbine are,

(i) The stator nozzles, and

(ii) the turbine rotor blades.

Normally a turbine stage is classified as

(i) an impulsion stage, and

(ii) a reaction stage

An impulse stage is characterized by the expansion of the gas which occurs only in the stator nozzles.
The rotor blades act as directional vanes to deflect the direction of the flow. Further, they convert the kinetic
energy of the gas into work by changing the momentum of the gas more or less at constant pressure.

A reaction stage is one in which expansion of the gas takes place both in the stator and in the rotor.

The function of the stator is the same as that in the impulse stage, but the function in the rotor is two
fold.

(i) the rotor converts the kinetic energy of the gas into work, and

(ii) contributes a reaction force on the rotor blades.

The reaction force is due to the increase in the velocity of the gas relative to the blades. This results from the
expansion of the gas during its passage through the rotor.

A Single Impulse Stage


Impulse machines are those in which there is no change of static or pressure head of the fluid in the rotor.
The rotor blades cause only energy transfer and there is no energy transformation. The energy transformation
from pressure or static head to kinetic energy or vice versa takes place in fixed blades only. As can be seen
from the below figure that in the rotor blade passage of an impulse turbine there is no acceleration of the
fluid, i.e., there is no energy transformation.

Hence, the chances are greater for separation due to boundary layer growth on the blade surface. Due to
this, the rotor blade passages of the impulse machine suffer greater losses giving lower stage efficiencies.

The paddle wheel, Pelton wheel and Curtis stem turbine are some examples of impulse machines.

www.Vidyarthiplus.com
www.Vidyarthiplus.com

A Single Reaction Stage


The reaction stages are those, in which, changes in static or pressure head occur both in the rotor and stator
blade passages. Here, the energy transformation occurs both in fixed as well as moving blades. The rotor
experiences both energy transfer as well as energy transformation. Therefore, reaction turbines are
considered to be more efficient. This is mainly due to continuous acceleration of flow with lower losses.

www.Vidyarthiplus.com
www.Vidyarthiplus.com

The degree of reaction of a turbomachine stage may be defined as the ratio of the static or pressure head
change occurring in the rotor to the total change across the stage.

Note: Axial-flow turbine with 50% reaction have symmetrical blades in their rotor and stators. It may be noted
that the velocity triangles at the entry and exit of a 50% reaction stage are also symmetrical.

www.Vidyarthiplus.com
www.Vidyarthiplus.com

Velocity Triangles of a Single Stage Machine


The flow geometry at the entry and exit of a turbomachine stage is described by the velocity triangles at these
stations. The velocity triangles for a turbomachine contain the following three components.

1. The peripheral / whirl / tangential velocity (u) of a rotor blades

2. The absolute velocity (c ) of the fluid and

3. The relative velocity (w or v) of the fluid

These velocities are related by the following well-known vector equation.

This simple relation is frequently used and is very useful in drawing the velocity triangles for
turbomachines.

The notation used here to draw velocity triangles correspond to the x-y coordinates; the suffix (a or
α) identifies components in the axial direction and suffix (t) refers to tangential direction. Air angles

www.Vidyarthiplus.com
www.Vidyarthiplus.com

in the absolute system are denoted by alpha (α), where as those in the relative system are
represented by beta (β).

Since the stage is axial, the change in the mean diameter between its entry and exit can be neglected.
Therefore, the peripheral or tangential velocity (u) remains constant in the velocity triangles.

It can be proved from the geometry that

ct2 + ct3 = wt2 + wt3

It is often assumed that the axial velocity component remains constant through the stage. For such condition,

ca = ca1 = ca2 = ca3

For constant axial velocity yields a useful relation,

tan α2 + tan α3 = tan β2 + tan β3

www.Vidyarthiplus.com
www.Vidyarthiplus.com

Expression for Work Output


Though force and torque are exerted on both stationary and moving blades alike, work can only be done on
the moving rotor blades. Thus the rotor blades transfer energy from the fluid to the shaft. The stage work in
an axial turbine (u3 = u2 = u) can be written as,

W = u2 ct2 – u3ct3

= u{ct2- (-ct3)}

= u(ct2-ct3)

{Note: Usually this equation will be written with a minus sign between ct2 and ct3. Whenever this is written with
a plus sign it is implied that ct3 is negative}

This equation can also be expressed in another form,

u2 ct3
W = ct2 +
c
u u
The first term t2 in the bracket depends on the nozzle or fixed angle (α2) and the ratio
u

σ= u . The contribution of the second term ct3 to the work is generally small. It is also c2
u observed that the kinetic energy of the fluid leaving the stage is greater for larger values of c t3. The
leaving loss from the stage is minimum when ct3 = 0, i.e., when the discharge from the stage is axial (c3 =
ca3). However, this condition gives lesser stage work as can be seen from the above two equations.

www.Vidyarthiplus.com
www.Vidyarthiplus.com

Blade loading and Flow coefficients


Performance of turbomachines are characterized by various dimensionless parameters. For example, loading
coefficient (ψ) and the flow coefficient (Ф) have been defined as,

W
Ψ =
φ u2
=
ca
u
Since the work, W in the above equation is frequently referred to as the blade or stage work, the coefficient,
ψ would also be known as the blade or stage loading coefficient.

For constant axial velocity (ca), it can be shown that

Ψ = Ф(tan α2 + tan α3) = Ф(tan β2 + tan β3)

The Ф – ψ plots are useful in comparing the performances of various stages of different sizes and geometries.

Blade and Stage efficiencies


Even though the blade and stage work (outputs) are the same, the blade and the stage efficiencies need not
be equal. This is because the energy inputs to the rotor blades and the stage (fixed blade ring plus the rotor)
are different. The blade efficiency is also known as the utilization factor (ε) which is an index of the energy
utilizing capability of the rotor blades. Thus,

ε = ηb

= Rotor blade work / Energy supplied to the rotor blades

= W / Erb

W = u2 ct2 + u3 ct3

www.Vidyarthiplus.com
www.Vidyarthiplus.com

1 ( 2 2 )
1 ( 2 2 ) 1( 3 2 )
= c2 −c3 + u2 −u3 + w3 − w2
2 2 2
The energy supplied to the rotor blades is the absolute kinetic energy in the jet at the entry plus the kinetic
energy change within the rotor blades.

12
Erb = c2 2
( 2 2 ) 1 ( 3 2 )
For axial machines, u = u2 = u3
+w3 − w2 + u2 − u3
2
ε=ηb = 2

c2
Maximum utilization factor for a single impulse stage.

u(ct2 +ct3) ε=
1 2 c2
2
After rearranging the terms, we have

ηb =ε= 4(σsinα2 −σ2)


This shows that the utilization factor is a function of the blade-to-gas speed ratio and the nozzle angle.

www.Vidyarthiplus.com
www.Vidyarthiplus.com

Elementary theory of axial flow turbine

Fig.7.2 Typical representations of velocity triangles

The above Figures show the velocity triangles for one axial flow turbine stage and the nomenclature
employed. The gas enters the row of nozzle blades (These are also known as 'stator blades' and 'nozzle

www.Vidyarthiplus.com
www.Vidyarthiplus.com

guide vanes') with a static pressure and temperature, P1, T1 and a velocity C1 is expanded to P2,T2 and leaves
with an increased velocity C2 at an angle α2. The rotor blade inlet angle will be chosen to suit the direction β2
of the gas velocity V2 relative to the blade at inlet.

β2and V2 are found by vectorial subtraction of the blade speed U from the absolute velocity C2. After being
deflected, and usually further expanded, in the rotor blade passages, the gas leaves at P 3, T3 with relative
velocity V3 at angle β3. Vectorial addition of U yields the magnitude and direction of the gas velocity at exit
from the stage, C3 and α3. α3 is known as the swirl angle.

dimensional effects.

www.Vidyarthiplus.com
www.Vidyarthiplus.com

Vortex theory
It was pointed out earlier that the shape of the velocity triangles must vary from root to tip of the blade because the
blade speed U increases with radius. Another reason is that the whirl component in the flow at outlet from the nozzles
causes the static pressure and temperature to vary across the annulus. With a uniform pressure at inlet, or at least with
a much smaller variation because the whirl component is smaller, it is clear that the pressure drop across the nozzle
will vary giving rise to a corresponding variation in efflux velocity C2. Twisted blading designed to take account of the
changing gas angles is called vortex blading.
It has been common steam turbine practice, except in low-pressure blading where the blades are very long,
to design on conditions at the mean diameter, keep the blade angles constant from root to tip, and assume that no
additional loss is incurred by the variation in incidence along the blade caused by the changing gas angles. Comparative
tests have been conducted by the earlier researchers on a single-stage gas turbine of radius ratio 1-37, using in turn
blades of constant angle and vortex blading. The results showed that any improvement in efficiency obtained with vortex
blading was within the margin of experimental error. This contrasts with similar tests on a 6-stage axial compressor, by
another researcher, which showed a distinct improvement from the use of vortex blading. This was, however, not
so much an improvement in efficiency (of about 1-5 per cent) as in the delay of the onset of surging which of
course does not arise in accelerating flow. It appears, therefore, that steam turbine designers have been
correct in not applying vortex theory except when absolutely necessary at the LP end. They have to consider
the additional cost of twisted blades for the very large number of rows of blading required, and they know that
the Rankine cycle is relatively insensitive to component losses. Conversely, it is not surprising that the gas
turbine designer, struggling to achieve the highest possible component efficiency, has consistently
used some form of vortex blading which it is felt intuitively must give a better performance however
small.
Vortex theory has been outlined earlier by Cohen and others where it was shown that if the
elements of fluid are to be in radial equilibrium, an increase in static pressure from root to tip is
necessary whenever there is a whirl component of velocity. Figure 7.8 shows (see below) why the gas
turbine designer cannot talk of impulse or 50 per cent reaction stages. The proportion of the stage pressure
or temperature drop which occurs in the rotor must increase from root to tip. Although Fig. 7.8 refers to a
single-stage turbine with axial inlet velocity and no swirl at outlet, the whirl component at inlet and outlet of a
repeating stage will be small compared with CW2- the reaction will therefore still increase from root to tip, if
somewhat less markedly.

www.Vidyarthiplus.com
www.Vidyarthiplus.com

Choice of blade profile, pitch and chord


The next step is to choose stator and rotor blade shapes which will accept the gas incident upon the leading
edge, and deflect the gas through the required angle with the minimum loss. An overall blade loss coefficient
Y (or A) must account for the following sources of friction loss.

(a) Profile loss—associated with boundary layer growth over the blade profile (including separation loss
under adverse conditions of extreme angles of incidence or high inlet Mach number).

(b) Annulus loss—associated with boundary layer growth on the inner and outer walls of the annulus.

(c) Secondary flow loss—arising from secondary flows which are always present when a wall boundary
layer is turned through an angle by an adjacent curved surface.

(d) Tip clearance loss—near the rotor blade tip the gas does not follow the intended path, fails to
contribute its quota of work output, and interacts with the outer wall boundary layer.

The profile loss coefficient Yp is measured directly in cascade tests similar to those described for compressor
blading. Losses (b) and (c) cannot easily be separated, and they are accounted for by a secondary loss
coefficient Ys.

The tip clearance loss coefficient, which normally arises only for rotor blades, will be denoted by Y k. Thus the
total loss coefficient Y comprises the accurately measured two-dimensional loss Yp, plus the three-
dimensional loss (Ys+Yk) which must be deduced from turbine stage test results. All that is necessary for our
present purpose for finding the choice of blade profile is limited to the knowledge of the sources of loss.

www.Vidyarthiplus.com
www.Vidyarthiplus.com

Figure 7.11 shows a conventional steam turbine blade profile constructed from circular arcs and straight lines.
Gas turbines have until recently used profiles closely resembling this, although specified by aerofoil
terminology.

www.Vidyarthiplus.com
www.Vidyarthiplus.com

Note that the blade profile will be completely determined when (a) the pitch/width ratio (s/w) is established,
and (b) both the camber line angle α' and blade thickness/pitch ratio have been calculated for various values
of x between 0 and 1.

www.Vidyarthiplus.com
www.Vidyarthiplus.com

Turbine Performance
The performance of turbine is limited principally by two factors: compressibility and stress.
Compressibility limits the mass flow that can pass through a given turbine and, stress limits the wheel speed
U. The work per stage depends on the square of the wheel speed. However, the performance of the engine
depends very strongly on the maximum temperature. Of course, as the maximum temperature increases, the
allowable stress level diminishes; hence in the design of the engine there must be a compromise between
maximum temperature and maximum rotor tip speed U.

For given pressure ratio and adiabatic efficiency, the turbine work per unit mass is proportional to the inlet
stagnation temperature. Since, in addition, the turbine work in a jet or turboshaft engine is commonly two or
three times the useful energy output of the engine, a 1% increase in turbine inlet temperature can produce a
2% or 3% increase in engine output. This considerable advantage has supplied the incentive for the adoption
of fairly elaborate methods for cooling the turbine nozzle and rotor blades.

Estimation of stage performance


The last step in the process of arriving at the preliminary design of a turbine stage is to check that the design
is likely to result in values of nozzle loss coefficient and stage efficiency which were assumed at the outset.
If not, the design calculations may be repeated with more probable values of loss coefficient and efficiency.
When satisfactory agreement has been reached, the final design may be laid out on the drawing board and
accurate stressing calculations can be

performed. Before proceeding to describe a method of estimating the design point performance of a stage,
however, the main factors limiting the choice of design, which we have noted during the course of the worked
example, will be summarized. The reason we considered a turbine for a turbojet engine was simply that we
would thereby be working near those limits to keep size and weight to a minimum. The designer of an
industrial gas turbine has a somewhat easier task: he will be using lower temperatures and stresses to obtain
a longer working life, and this means lower mean blade speeds, more stages, and much less
stringentaerodynamic limitations. A power turbine, not mechanically coupled to the gas generator, is another
case where much less difficulty will be encountered in arriving at a satisfactory solution. The choice of gear
ratio between the power turbine and driven component is normally at the disposal of the turbine designer,
and thus the rotational speed can be varied to suit the turbine, instead of the compressor as we have assumed
here.

www.Vidyarthiplus.com
www.Vidyarthiplus.com

www.Vidyarthiplus.com
www.Vidyarthiplus.com

The cooled turbine

Figure 7.29 illustrates the methods of blade cooling that have received serious attention and research effort.
Apart from the use of spray cooling for thrust boosting in turbojet engines, the liquid systems have not proved
to be practicable. There are difficulties associated with channelling the liquid to and from the blades—whether
as primary coolant for forced convection or free convection open thermosyphon systems, or as secondary

www.Vidyarthiplus.com
www.Vidyarthiplus.com

coolant for closed thermosyphon systems. It is impossible to eliminate corrosion or the formation of deposits
in open systems, and very difficult to provide adequate secondary surface cooling area at the base of the
blades for closed systems. The only method used successfully in production engines has been internal,
forced convection, air cooling. With 1-5-2 per cent of the air mass flow used for cooling per blade row, the
blade temperature can be reduced by between 200 and 300 °C. Using current alloys, this permits turbine
inlet temperatures of more than 1650 К to be used. The blades are either cast, using cores to form the cooling
passages, or forged with holes of any desired shape produced by electrochemical or laser drilling.

Figure 7.30 shows the type of turbine rotor blade introduced in the 1980s. The next step forward is likely to
be achieved by transpiration cooling, where the cooling air is forced through a porous blade wall. This method
is by far the most economical in cooling air, because not only does it remove heat from the wall more
uniformly, but the effusing layer of air insulates the outer surface from the hot gas stream and so reduces the
rate of heat transfer to the blade. Successful application awaits further development of suitable porous
materials and techniques of blade manufacture. We are here speaking mainly of rotor blade cooling because
this presents the most difficult problem. Nevertheless it should not be forgotten that, with high gas
temperatures, oxidation becomes as significant a limiting factor as creep, and it is therefore equally important
to cool even relatively unstressed components such as nozzle blades and annulus walls.

Figure 7.31 (a) illustrates the principal features of nozzle blade cooling. The air is introduced in such a way
as to provide jet impingement cooling of the inside surface of the very hot leading edge. The spent air leaves
through slots or holes in the blade surface (to provide some film cooling) or in the trailing edge.

www.Vidyarthiplus.com
www.Vidyarthiplus.com

FIG. 7.31 (a) Turbine nozzle cooling [(b) courtesy Rolls-Royce]

www.Vidyarthiplus.com
www.Vidyarthiplus.com

www.Vidyarthiplus.com
www.Vidyarthiplus.com

Figure 7.35: Shows a typical temperature distribution at the mid-span of a blade


designed to operate with Tg= 1500 К and Tсг = 320 К.

It may be noted that the final design calculation for a cooled blade will involve an estimation of the two-
dimensional temperature distribution over the blade cross-section at several values of l/L. Finite difference
methods are used to solve the differential equations, and conduction within the blade is taken into account.
Figure 7.35 shows a typical temperature distribution at the midspan of a blade designed to operate with Tg=
1500 К and Гсг = 320 К. It emphasizes one of the main problems of blade cooling, i.e. that of obtaining
adequate cooling at the trailing edge. Finally, an estimation will be made of the thermal stresses incurred with
due allowance for redistribution of stress by creep: with cooled blades the thermal stresses can dominate the
gas bending stresses and be comparable with the centrifugal tensile stresses.

Finally, mention must be made of an alternative approach to the high- temperature turbine—the use of
ceramic materials which obviates the need for elaborate cooling passages. Much effort has been expended
on the development of silicon nitride and silicon carbide materials for small turbine rotors (both axial and
radial) in which it would be difficult to incorporate cooling passages. Adequate reliability and life are difficult
to achieve, but demonstrator engines have been ran for short periods. Ceramic rotor blades are being
investigated for use in stationary gas turbines for powers up to about 5 MW, and experimental trials are
expected in the late 1990s. The use of ceramic turbines in production engines, however, remains an elusive
goal three decades after optimistic forecasts about their introduction.

Blade Cooling
Blade cooling is the most effective way of maintaining high operating temperatures making use of the
available material. Blade cooling may be classified based on the cooling site as external cooling and internal
cooling. Another classification based on the cooling medium is liquid cooling and air cooling.

External Cooling

The external surface of the gas turbine blade is cooled by making use of compressed air from the
compressor.

www.Vidyarthiplus.com
www.Vidyarthiplus.com

Other methods of external cooling are film cooling and transpiration cooling.

Internal Cooling

Internal cooling of blades is achieved by passing air or liquid through internal cooling passages from hub
towards the blade tip. The cooling of the blades is achieved by conduction and convection.

Hollow blades can also be manufactured with a core and internal cooling passage. Based on the cooling
medium employed, blade cooling may be classified into liquid cooling and air cooling.

Requirements for Efficient Blade Cooling


In a conventional cooled blade, cooling is obtained due to convection by passing cooling air through internal
passages within the blade. The success in obtaining the large reduction in metal temperature at the expense
of a small quantity of cooling flow is governed by the skill in devising and machining the cooling passages.
Because the internal cooling relies on the cooling air scrubbing against the cooling surface, the internal
surface area must be large and the velocity of the cooling air must be high. This implies that the cross-
sectional flow area of the passage must be small.

The design of the blade internal geometry for cooling is more complex because of the various aerodynamic,
heat transfer, stress and mechanical design criteria that must be satisfied. The most successful designs have
incorporated radial passages through which cooling air passes, escaping at the tip.

The radial flow turbine


In a radial flow turbine, gas flow with a high tangential velocity is directed inwards and leaves the rotor with
as small a whirl velocity as practicable near the axis of rotation. The result is that the turbine looks very similar
to the centrifugal compressor, but with a ring of nozzle vanes replacing the diffuser vanes as in Fig. 7.37.
Also, as shown there would normally be a diffuser at the outlet to reduce the exhaust velocity to a negligible
value.

www.Vidyarthiplus.com
www.Vidyarthiplus.com

www.Vidyarthiplus.com
www.Vidyarthiplus.com

www.Vidyarthiplus.com
www.Vidyarthiplus.com

www.Vidyarthiplus.com
www.Vidyarthiplus.com

Turbine and Compressor Matching


The problem of matching turbine and compressor performance has great importance for jet engines, which
must operate under conditions involving large variations in thrust, inlet pressure, and temperature, and flight
Mach number. Matching the components of turbofan and turboprop engines involves similar considerations
and procedures.

Essentially the matching problem is simple, though the computation can be length. The steady-state engine
performance at each speed is determined by two conditions: continuity of flow and a power balance. The
turbine mass flow must be the sum of the compressor mass flow and the fuel flow, minus compressor bleed
flow. Also the power output of the turbine must be equal to that demanded by the compressor.

In principle, the matching computations could proceed as follows:

1. Select operating speed

2. Assume turbine inlet temperature

3. Assume compressor pressure ratio

4. Calculate compressor work per unit mass

5. Calculate turbine pressure ratio required to produce this work

6. Check to see if compressor mass flow plus fuel flow equals turbine mass flow; if not, assume a new
value of compressor pressure ratio and repeat steps 4, 5, and 6 until continuity is satisfied.

7. Now calculate the pressure ratio across the jet nozzle from the pressure ratios across the diffuser,
compressor, combustor, and turbine.

8. Calculate the area of jet nozzle outlet necessary to pass the turbine mass flow calculated in step 6
with pressure ratio calculated in step 7 and the stagnation temperature calculated. If the calculated
area does not equal the actual exit area, assume a new value of turbine inlet temperature (step-2)
and repeat the entire procedure.

www.Vidyarthiplus.com
www.Vidyarthiplus.com

The designer will try to match turbine and compressor so that the compressor is operating near its
peak efficiency through the entire range of operation, as shown in the below figure, where the
operating line (i.e., the locus of stead-state matching condition) runs through the centers of the
islands defined by the constant-efficiency lines.

www.Vidyarthiplus.com
www.Vidyarthiplus.com

The essence of compressor-turbine matching is to find a speed at which the turbine will run delivering power
to the compressor and permit excess power in the form of adequate jet pressure for expansion through the
nozzle or additional turbine stages from which power can be derived to run a propeller or helicopter rotor as
the case may be.

Problems
In a single-stage impulse turbine the nozzle discharges the fluid on to the blades at an angle of 65 0 to the
axial direction and the fluid leaves the blades with an absolute velocity of 300 m/s at an angle of 30 0 to the
axial direction. If the blades have equal inlet and outlet angles and there is no axial thrust, estimate the blade
angle, power produced per kg/s of the fluid and blade efficiency.

www.Vidyarthiplus.com
www.Vidyarthiplus.com

Solution:

Since the axial thrust is zero,

Ca3 = Ca2 = Ca

w3 = w2

ca3 = ca2 = ca = ca3 x cos α3

= 300 x cos 30 = 259.8 m/s c2

= ca2/cos α2 = 259.8/cos 65 = 614.7 m/s u =

ct2 – wt2 = wt3 - ct3 c2 sin α2 - ca2 tan β2 = ca3

tan β3 – c3 sin α3

www.Vidyarthiplus.com
www.Vidyarthiplus.com

Since ca2 = ca3 = ca and β2 = β3 = β 2ca tan β = c2 sin α2

+ c3 sin α3 tan β = (614.7 x sin 65 + 300 x sin 30) / 2

x 259.8

= 1.3609 β = 53.7 0

β2 = β3 = 53.7 0 u = c2 sin

α2 - ca2 tan β2

= 614.7 x sin 65 – 259.8 x tan 53.7

= 203.43 m/s

ct2 = c2 sin α2

= 614.7 x sin 65 =

557.1 m/s

ct3 = c3 sin α3
= 300 x sin 30

= 150 m/s

WT = 203.43 x (557.1 + 150) x 10-3

= 144 kJ/kg σ = u/c2 = 203.43 /

614.7 = 0.33

Blade efficiency = 4 (σ sin α2 - σ2)

= 4 x (0.33 x sin 65 – 0.332) = 0.761

= 76.1 %

www.Vidyarthiplus.com
www.Vidyarthiplus.com

Aircraft Engine Performance Parameters

Uninstalled thrust of a jet engine (single inlet and single exhaust) is given by

www.Vidyarthiplus.com
www.Vidyarthiplus.com

Uninstalled thrust of a jet engine (single inlet and single exhaust) is given by

It is most desirable to expand the exhaust gas to the ambient pressure, which gives

In this case, the uninstalled thrust equation becomes,

The installed thrust T is equal to the uninstalled thrust F minus the inlet drag D inlet and minus nozzle drag
Dnozzle , or

T = F – Dinlet - Dnozzle

Problem:

www.Vidyarthiplus.com
www.Vidyarthiplus.com

An advanced fighter engine operating at Mach 0.8 and 10 km altitude has the following unistalled performance
data and uses a fuel with low heating value 42,800 kJ/kg:

Uninstalled thrust = 50 kN

Mass flow rate of air = 45 kg/sec

Mass flow rate of fuel = 2.65 kg/sec

Determine the specific thrust, thrust specific fuel consumption, exit velocity, thermal efficiency, propulsive
efficiency, and overall efficiency (assume exit pressure equal to ambient pressure). Find the installed thrust
when the total drag is equal to 10% of the uninstalled thrust.

Solution:

Fs = 50/45 = 1.1111kN/(kg/sec)

= 1111.1 m/s

TSFC = 2.65 / 50

= 0.053 (kg/sec)/kN

= 53 mg/N-sec

V0 = Mo ao

= 239. 6 m/sec

www.Vidyarthiplus.com
www.Vidyarthiplus.com

Using the above equation exit velocity,

Ve is obtained as 1275.6 m/s

Power output of the engine = 37.475 x 106 W

Power input to the engine through fuel = 113.42 x 106 W

Propulsive power or Thrust Power = 50,000 x 239.6

Thermal efficiency = 37.475 x 106 / 113.42 x 106

= 33.04 %

Propulsive Efficiency = 50,000 x 239.6 / 37,475 x 106

= 31.97 %

Overall efficiency = 50,000 x 239.6 / 113.42 x 106

www.Vidyarthiplus.com
www.Vidyarthiplus.com

= 10.56 %

For additional numerical problems please refer the following books

1. Mathur, M., and Sharma, R.P., “Gas Turbines and Jet and Rocket Propulsion”, Standard
Publishers, New Delhi, 1988.

2. Ganesan, V., Gas Turbines, Tata McGraw-Hill Publishing Company Limited, New Delhi.
Second Edn. 2003.

Additional reading

1. Hill, P.G. & Peterson, C.R. “Mechanics & Thermodynamics of Propulsion” Addison – Wesley
Longman INC, 1999.

2. Cohen, H., Rogers, G.F.C. and Saravanamuttoo, H.I.H., “Gas Turbine Theory”, Longman Co., ELBS
Ed., 1989.

Unit-2 RAMJET PROPULSION


Operating principle – Sub critical, critical and supercritical operation – Combustion in ramjet engine – Ramjet
performance – Sample ramjet design calculations – Introduction to scramjet – Preliminary concepts in
supersonic combustion – Integral ram- rocket- Numerical problems.

Ramjets
Introduction

www.Vidyarthiplus.com
www.Vidyarthiplus.com

• Ramjets can be thought of as propulsive devices evolved out of turbojets.


• Ramjets operates well only at high speeds, typically between M = 2.0 and 4.0.
• The combustion mode being not very different from that of an afterburner, the specific fuel consumption is
comparable to that of afterburner.
• The ramjet, unlike turbojet and turbofan does not produce any thrust at zero speed.
• Ramjets are mostly contemplated for use in military applications.

SCRAM JET ENGINES

• A scram jet engine is an engine that is much lighter than a conventional jet engine, can propel an object at
speeds of over 5000 miles per hour and has no moving parts.
• If you could get it to work, the trip from London to Sydney would only take two hours!

www.Vidyarthiplus.com
www.Vidyarthiplus.com

• This technology would also be very useful to launch small satellites.


• The engine runs on oxygen, which it gets from the atmosphere, and a small amount of hydrogen.
• The engine would save a fantastic amount on the cost of fuel.
• This technology has been around since the 1950s but the problem is the motor will only become efficient at
five times the speed of sound or Mach 5.
• Because of this the plane would need two engines, an engine capable of getting it to Mach 5 and a Scram
Jet.

A ramjet engine

A scramjet engine

www.Vidyarthiplus.com
www.Vidyarthiplus.com


A ramjet has no moving parts and achieves compression of intake air by the forward speed of the air
vehicle.
• Air entering the intake of a supersonic aircraft is slowed by aerodynamic diffusion created by the inlet
and diffuser to velocities comparable to those in a turbojet augmenter.
• The expansion of hot gases after fuel injection and combustion accelerates the exhaust air to a
velocity higher than that at the inlet and creates positive push.
• Scramjet is an acronym for Supersonic Combustion Ramjet.
• The scramjet differs from the ramjet is that combustion takes place at supersonic air velocities
through the engine.
• It is mechanically simple, but vastly more complex aerodynamically than a jet engine.
• Hydrogen is normally the fuel used.
• A scramjet (supersonic combustion ramjet) is a variation of a ramjet with the key difference being
that the flow in the combustor is supersonic.
• At higher speeds it is necessary to combust supersonically to maximize the efficiency of the
combustion process.
• Projections for the top speed of a scramjet engine (without additional oxidizer input) vary between
Mach 12 and Mach 24 (orbital velocity), but the X-30 research gave Mach 17 due to combustion rate
issues.
• By way of contrast, the fastest conventional air-breathing, manned vehicles, such as the
U.S. Air Force SR-71, achieve slightly more than Mach 3.2 and rockets achieved Mach 30+ during
Apollo.
• Like a ramjet, a scramjet essentially consists of a constricted tube through which inlet air is
compressed by the high speed of the vehicle, fuel is combusted, and then the exhaust jet
leaves at higher speed than the inlet air.
• Also like a ramjet, there are few or no moving parts. In particular there is no high speed turbine as in
a turbofan or turbojet engine that can be a major point of failure.
• A scramjet requires supersonic airflow through the engine, thus, similar to a ramjet, scramjets have
a minimum functional speed. This speed is uncertain due to the low number of working scramjets,
relative youth of the field, and the largely classified nature of research using complete scramjet
engines.
• However it is likely to be at least Mach 5 for a pure scramjet, with higher Mach numbers
7-9 more likely. Thus scramjets require acceleration to hypersonic speed via other means.
• A hybrid ramjet/scramjet would have a lower minimum functional Mach number, and some sources
indicate the NASA X-43A research vehicle is a hybrid design.
• Recent tests of prototypes have used a booster rocket to obtain the necessary velocity.

www.Vidyarthiplus.com
www.Vidyarthiplus.com

• Air breathing engines should have significantly better specific impulse while within the
atmosphere than rocket engines.
• However scramjets have weight and complexity issues that must be considered. While very short
suborbital scramjets test flights have been successfully performed, perhaps significantly no flown
scramjet has ever been successfully designed to survive a flight test.
• The viability of scramjet vehicles is hotly contested in aerospace and space vehicle circles, in part
because many of the parameters which would eventually define the efficiency of such a vehicle
remain uncertain.
• This has led to grandiose claims from both sides, which have been intensified by the large amount
of funding involved in any hypersonic testing. Some notable aerospace gurus such as Henry Spencer
and Jim Oberg have gone so far as calling orbital scramjets 'the hardest way to reach orbit', or even
'scramjets' due to the extreme technical challenges involved.
• Major, well funded projects, like the X-30 were cancelled before producing any working hardware.
• The scramjet is a proposed solution to both of these problems, by modifications of the ramjet design.
The main change is that the blockage inside the engine is reduced, so that the air isn't slowed down
as much. This means that the air is cooler, so that the fuel can burn properly. Unfortunately the higher
speed of the air means that the fuel has to mix and burn in a very short time, which is difficult to
achieve.
• To keep the combustion of the fuel going at the same rate, the pressure and temperature in the
engine need to be kept constant. Unfortunately, the blockages which were removed from the ramjet
were useful to control the air in the engine, and so the scramjet is forced to fly at a particular speed
for each altitude. This is called a "constant dynamic pressure path" because the wind that the
scramjet feels in its face is constant, making the scramjet fly faster at higher altitude and slower at
lower altitude.
• The inside of a very simple scramjet would look like two kitchen funnels attached by their small ends.
The first funnel is the intake, and the air is pushed through, becoming compressed and hot. In the
small section, where the two funnels join, fuel is added, and the combustion makes the gas become
even hotter and more compressed. Finally, the second funnel is a nozzle, like the nozzle of a rocket,
and thrust is produced.
• Note that most artists' impressions of scramjet-powered vehicle designs depict waveriders where the
underside of the vehicle forms the intake and nozzle of the engine. This means that the intake and
nozzle of the engine are asymmetric and contribute directly to the lift of the aircraft. A waverider is
the required form for a hypersonic lifting body
• A scramjet is a type of engine which is designed to operate at the high speeds normally associated
with rocket propulsion.
• It differs from a classic rocket by using air collected from the atmosphere to burn its fuel, as opposed
to an oxidizer carried with the vehicle.
• Normal jet engines and ramjet engines also use air collected from the atmosphere in this way.

www.Vidyarthiplus.com
www.Vidyarthiplus.com


• The problem is that collecting air from the atmosphere causes drag, which increases quickly as the
speed increases.
Also, at high speed, the air collected becomes so hot that the fuel no longer burns properly.

Theory
• All scramjet engines have fuel injectors, a combustion chamber, a thrust nozzle and an inlet, which
compresses the incoming air.
• Sometimes engines also include a region which acts as a flame holder, although the high stagnation
temperatures mean that an area of focused waves may be used, rather than a discrete engine part
as seen in turbine engines.
• Other engines use pyrophoric fuel additives, such as silane, to avoid such issues.
• An isolator between the inlet and combustion chamber is often included to improve the homogeneity
of the flow in the combustor and to extend the operating range of the engine.
• A scramjet is reminiscent of a ramjet.
• In a typical ramjet, the supersonic inflow of the engine is decelerated at the inlet to subsonic speeds
and then reaccelerated through a nozzle to supersonic speeds to produce thrust. This deceleration,
which is produced by a normal shock, creates a total pressure loss which limits the upper operating
point of a ramjet engine.
• For a scramjet, the kinetic energy of the freestream air entering the scramjet engine is large
compared to the energy released by the reaction of the oxygen content of the air with a fuel (say
hydrogen).
• Thus the heat released from combustion at Mach 25 is around 10% of the total enthalpy of the
working fluid.
• Depending on the fuel, the kinetic energy of the air and the potential combustion heat release will be
equal at around Mach 8.
• Thus the design of a scramjet engine is as much about minimizing drag as maximizing thrust.
• This high speed makes the control of the flow within the combustion chamber more difficult. Since
the flow is supersonic, no upstream influence propagates within the freestream of the combustion
chamber.
• Thus throttling of the entrance to the thrust nozzle is not a usable control technique.
• In effect, a block of gas entering the combustion chamber must mix with fuel and have sufficient time
for initiation and reaction, all the while travelling supersonically through the combustion chamber,
before the burned gas is expanded through the thrust nozzle.
• This places stringent requirements on the pressure and temperature of the flow, and requires that
the fuel injection and mixing be extremely efficient.
• Usable dynamic pressures lie in the range 20 to 200 kPa (0.2-2 bar), where

www.Vidyarthiplus.com
www.Vidyarthiplus.com

• where
q is the dynamic pressure of the gas
ρ (rho) is the density of the gas
v is the velocity of the gas
• Fuel injection and management is also potentially complex.
• One possibility would be that the fuel is pressurized to 100 bar by a turbo pump, heated by the
fuselage, sent through the turbine and accelerated to higher speeds than the air by a nozzle.
• The air and fuel stream are crossed in a comb like structure, which generates a large interface.
• Turbulence due to the higher speed of the fuel lead to additional mixing. Complex fuels like kerosine
need a long engine to complete combustion.
• The minimum Mach number at which a scramjet can operate is limited by the fact that the
compressed flow must be hot enough to burn the fuel, and of high enough pressure that the reaction
is finished before the air moves out the back of the engine.
• Additionally, in order to be called a scramjet, the compressed flow must still be supersonic after
combustion.
• Here two limits must be observed:
• Firstly, since when a supersonic flow is compressed it slows down, the level of compression must be
low enough (or the initial speed high enough) not to slow down the gas below Mach 1.
• If the gas within a scramjet goes below Mach 1 the engine will "choke", transitioning to subsonic flow
in the combustion chamber.
• This effect is well known amongst experimenters on scramjets since the waves caused by choking
are easily observable.
• Additionally, the sudden increase in pressure and temperature in the engine can lead to an
acceleration of the combustion, leading to the combustion chamber exploding.
• Secondly, the heating of the gas by combustion causes the speed of sound in the gas to
increase (and the Mach number to decrease) even though the gas is still traveling at the same
speed. Forcing the speed of air flow in the combustion chamber under Mach 1 in this way is
called "thermal choking".
• It is clear that a pure scramjet can operate at Mach numbers of 6-8, but in the lower limit, it
depends on the definition of a scramjet.
• Certainly there are designs where a ramjet transforms into a scramjet over the Mach 3-6 range (Dual-
mode scramjets). In this range however, the engine is still receiving significant thrust from subsonic
combustion of "ramjet" type.
• The high cost of flight testing and the unavailability of ground facilities have hindered scramjet
development.

www.Vidyarthiplus.com
www.Vidyarthiplus.com


• A large amount of the experimental work on scramjets has been undertaken in cryogenic facilities,
direct-connect tests, or burners, each of which simulates one aspect of the engine operation.
Further, vitiated facilities, storage heated facilities, arc facilities and the various types of shock
tunnels each have limitations which have prevented perfect simulation of scramjet operation.
• The HyShot flight test showed the relevance of the 1:1 simulation of conditions in the T4 and HEG
shock tunnels, despite having cold models and a short test time.
• The NASA-CIAM tests provided similar verification for CIAM's C-16 V/K facility and the Hyper-X
project is expected to provide similar verification for the Langley AHSTF , CHSTF and 8 ft HTT.
• Computational fluid dynamics has only recently reached a position to make reasonable computations
in solving scramjet operation problems.
• Boundary layer modeling, turbulent mixing, two-phase flow, flow separation, and realgas
aerothermodynamics continue to be problems on the cutting edge of CFD.
• Additionally, the modeling of kinetic-limited combustion with very fast-reacting species such as
hydrogen makes severe demands on computing resources.
• Reaction schemes are numerically stiff, having typical times as low as 10-19 seconds, requiring
reduced reaction schemes.
• Much of scramjet experimentation remains classified. Several groups including the US
Navy with the SCRAM engine between 1968-1974, and the Hyper-X program with the X-43A have
claimed successful demonstrations of scramjet technology. Since these results have not been
published openly, they remain unverified and a final design method of scramjet engines still does not
exist.
• The final application of a scramjet engine is likely to be in conjunction with engines which can operate
outside the scramjet's operating range.
• Dual-mode scramjets combine subsonic combustion with supersonic combustion for operation at
lower speeds, and rocket-based combined cycle (RBCC) engines supplement a traditional rocket's
propulsion with a scramjet, allowing for additional oxidizer to be added to the scramjet flow.
• RBCCs offer a possibility to extend a scramjet's operating range to higher speeds or lower intake
dynamic pressures than would otherwise be possible.

Advantages and disadvantages of scramjets

Special cooling and materials


• Unlike a rocket that quickly passes mostly vertically through the atmosphere or a turbojet or ramjet
that flies at much lower speeds, a hypersonic airbreathing vehicle optimally flies a "depressed
trajectory", staying within the atmosphere at hypersonic speeds.

www.Vidyarthiplus.com
www.Vidyarthiplus.com

• Because scramjets have only mediocre thrust-to-weight ratios, acceleration would be limited.
Therefore time in the atmosphere at hypersonic speed would be considerable, possibly 15-30
minutes.
• Similar to a reentering space vehicle, heat insulation from atmospheric friction would be a formidable
task. The time in the atmosphere would be greater than that for a typical space capsule, but less than
that of the space shuttle.
• Therefore studies often plan on "active cooling", where coolant circulating throughout the vehicle skin
prevents it from disintegrating from the fiery atmospheric friction.
• Active cooling could require more weight and complexity. There is also safety concern since it's an
active system.
• Often, however, the coolant is the fuel itself, much in the same way that modern rockets use their
own fuel and oxidizer as coolant for their engines.
• Both scramjets and conventional rockets are at risk in the event of a cooling failure.

Half an engine

• The typical waverider scramjet concept involves, effectively, only half an engine. The shockwave of
the vehicle itself compresses the inlet gasses, forming the first half of the engine. Likewise, only fuel
(the light component) needs tankage, pumps, etc. This greatly reduces craft mass and construction
effort, but the resultant engine is still very much heavier than an equivalent rocket or conventional
turbojet engine of similar thrust.

Simplicity of design

• Scramjets have few to no moving parts. Most of their body consists of continuous surfaces. With
simple fuel pumps, reduced total components, and the reentry system being the craft itself, scramjet
development tends to be more of a materials and modelling problem than anything else.

Additional propulsion requirements

• A scramjet cannot produce efficient thrust unless boosted to high speed, around Mach 5, depending
on design, although, as mentioned earlier, it could act as a ramjet at low speeds. A horizontal take-
off aircraft would need conventional turbofan or rocket engines to take off, sufficiently large to move
a heavy craft. Also needed would be fuel for those engines, plus all engine associated mounting
structure and control systems. Turbofan engines are heavy and cannot easily exceed about Mach 2-
3, so another propulsion method would be needed to reach scramjet operating speed. That could be
ramjets or rockets. Those would also need their own separate fuel supply, structure, and systems.
Many proposals instead call for a first stage of droppable solid rocket boosters, which greatly
simplifies the design.

www.Vidyarthiplus.com
www.Vidyarthiplus.com

www.Vidyarthiplus.com
www.Vidyarthiplus.com

Pulsejet

Pulsejet is a constant volume combustion device which is quite efficient only at low speeds and uses unsteady
combustion for its operation. Once the system is started, it works by taking in air and fuel and combustion
them during a part of the cycle and exhausting it during the next part of the cycle. In order to promote and
sustain the operation, a flapper device of metal or plastic, in recent times, is used in front end.

Supersonic Diffuser
Super sonic diffuser may be divided into two separate parts: the supersonic inlet and the subsonic diffuser.
Although this division is a convenient one, it must be remembered that the phenomena in the supersonic and
subsonic parts of the diffuser can, and often do, interact.

It should be remembered that the higher the Mach number, the greater will be our percentage loss in
pressure across a normal shock. Thus, at very high mach numbers, there is an appreciable difference
between (1) isentropic compression through out and (2) compression with a normal shock.

The limitations imposed by the losses associated with a normal shock at higher Mach numbers stimulated
the development of a modified simple inlet design which incorporates a spike or wedge in the nose section
of the diffuser.

The modified design utilizes the spike or wedge to produce an oblique shock at the nose section, and, the
mach number after an oblique shock are greater than one. Consequently, in addition to the oblique shock,
we have a normal shock near the minimum area section. The compression after the normal shock is of the
standard subsonic diffuser type; that is, an increase in area decreases the velocity.

It might be asked why a diffuser which has two shock waves, namely, an oblique and a normal shock,
would give better results than a simple inlet which has only one normal shock. The reason for this is that the

www.Vidyarthiplus.com
www.Vidyarthiplus.com

losses across a series of weak shock are less than the losses across one or several strong shocks. This
means that the losses across the normal shock which follows the oblique shock are considerably less than
the losses across a normal shock in the free stream.

Increasing the number of oblique shocks before the normal shock reduces the losses through the shock
system. It must be noted that the complexity of the inlet also increases as the number shocks increases. In
addition to the number of shocks, the pressure losses are also function of the shock arrangement. It may be
noted that the maximum total pressure recovery occurs when the total pressure recovery is the same across
each of oblique shocks, and very nearly equal to that across the final normal shock.

Mode of Supersonic Diffuser Operation


The three basic modes operation frequently referred to are subcritical operation, critical operation,
and supercritical operation. All these three inlets are operated at the design Mach number, M D, which by
definition, means that the conical shock or conical shock extended will intersect the cowl lip.

• At the subcritical operation, the normal shock is external and subsonic velocities exit at the cowl.
• For this condition mass-flow ratio based on capture area is less than one; spillage exists; the inlet
is not swallowing air at maximum capacity; pressure recovery is low since some of the air goes
through a single, near normal shock; inlet drag is high because of the intense shock; operation is
generally unstable and conducive to a condition called “buzz” (normal shock moves in and out of
the inlet at relatively high frequencies). An important instability that occurs during the subcritical
operation of most supersonic inlets. This phenomenon, known as “buzz”, consists of a rapid
oscillation of the inlet shock and flow pattern; the resultant internal disturbance is very detrimental
to engine performance. In a ramjet, for instance, the onset of buzz usually extinguishes combustion.
Although the pulses of the shock system are similar, the interval between pulses is not constant;
hence buzz cannot be considered a periodic phenomenon. Although it is not thoroughly understood,
buzz has been shown to be a function of conditions only at , and immediately downstream of, the
inlet.
• In general subcritical operation is unsatisfactory and should be avoided.
• As the flow resistance downstream of the diffuser is increased, the mass-flow ratio can be reduced
to its limit value of 0 at which point no flow exits.
• For critical operation, both maximum mass flow and ram recovery are attained for the design Mach
number; thus condition represents the optimum performance condition. It has the disadvantage,
however, of being, marginally unstable in actual applications because small changes in angles of
attack or yaw or boundary layer separation can induce the critical mode of operation across the
threshold into the subcritical regime. Consequently, for actual operation it is usually better to
operate the inlet in a more stable condition, the supercritical regime.

www.Vidyarthiplus.com
www.Vidyarthiplus.com

Typical modes of inlet operation

The three basic modes operation frequently referred to aircraft inlets are subcritical operation, critical
operation, and supercritical operation, which are shown schematically for a typical case in the above figure.
With entirely diverging internal flow such as this, the normal shock position is determined by a downstream
flow restriction rather than by the inlet geometry. Hence operating mode is sensitive to variations in exhaust-
nozzle area and fuel flow rate. Subcritical operation entails “spilling” of flow and a normal shock upstream of
the inlet. “Low” and “high” subcritical operations differ only in the extent of spilling. Supercritical operation
occurs at the same mass flow as critical operation, but with increased losses, since the normal shock occurs
at a higher Mach number. All these three inlets are operated at the design Mach number, M D, which by
definition, means that the conical shock or conical shock extended will intersect the cowl lip.

www.Vidyarthiplus.com
www.Vidyarthiplus.com

Ramjet Performance

www.Vidyarthiplus.com
www.Vidyarthiplus.com

www.Vidyarthiplus.com
www.Vidyarthiplus.com

The ramjet has the virtue of maximum simplicity, with no need for turbomachinery, and maximum tolerance
to high-temperature operation and minimum mass-per-unit thrust at suitable flight Mach numbers. The ramjet
also has its limitations. As well as being generally incapable of steady operation at subsonic Mach numbers,
it has an upper Mach number limit. For the conventional ramjet (in which the supersonic inlet air is slowed to
subsonic speeds to provide stable combustion prior to the nozzle expansion), there is a limiting Mach number
of about 6, above which the temperature of the air entering the combustor is so high that combustion cannot
be completed. Most of the chemical energy of combustion is nonusefully transformed into dissociation
reactions that on expansion do not provide the exhaust velocity needed for satisfactory ramjet performance.
To avoid this problem, substantial research has been, and is still being, focused on the supersonic
combustion ramjet (SCRAMJET). The difference between this and the conventional ramjet is that combustion
is to take place in a supersonic stream. Fuel must be injected into the supersonic stream (without causing
disruptive shock waves) and must mix and burn in a millisecond or so. Conventional fuels do not ignite quickly
enough, and gaseous hydrogen seems the most likely candidate. Design of the fuel injector is a formidable
challenge; so is the problem of cooling a vehicle designed to operate at Mach numbers of 4 or higher.
Hydrogen could conceivably serve for structural cooling as well as for engine fuel.

It will be clear from this discussion that future designs for supersonic (or even hypersonic) aircraft must cope
with the design challenges of a whole range of flight Mach numbers.

Supersonic Combustion
The losses associated with subsonic ramjet combustion can be substantial. If ramjets are to be applied to
hypersonic flight, additional problems arise because of extremely high temperature at the entrance to the
combustion chamber. This not only makes vehicle cooling very difficult, but it leads to severe combustion
loss due to dissociation.

Figure shows the air temperature reached after adiabatic deceleration from a high-altitude ambient
temperature of 220 K and from flight Mach number M to a chamber pressure of either 1 or 10 atm. For
hypersonic flight (e.g. for M > 8) the temperature of the air in the chamber is quite dependent on the pressure:
The higher the pressure, the less dissociation and the higher the temperature of the mixture. The temperature
of the combustion products is likewise pressure dependent.

Figure shows, for a combustion pressure of 10 atm and a flight Mach number of 10, there is no temperature
rise due to combustion; all of the combustion energy is absorbed by dissociation. One sees from the above
Figure, the strange result that, at sufficiently high Mach number, the temperature of the combustion products
can be lower than that of the incoming air. Consideration of the speed with which the fuel and air can be
converted into dissociation products may show that there is sufficient residence time in the combustion
chamber to approach equilibrium composition. But during the subsequent expansion in the nozzle, one
cannot take equilibrium composition for granted. It is quite possible that the expansion will be too rapid for
the composition to readjust, after each step of temperature and pressure reduction, to a new equilibrium

www.Vidyarthiplus.com
www.Vidyarthiplus.com

composition. If the expansion extremely rapid, the mixture may be effectively “frozen” with the initial (high-
temperature) composition. This would mean that little of the combustion energy of the fuel (for the M = 10
case) would be available for acceleration of the combustion products to produce thrust. The chemical kinetics
of the recombination processes in the nozzle would, in general, have a strong effect on the thrust and the
propulsion efficiency of the engine.

Some researchers have proposed the concept of the supersonic combustion as a way to avoid this
dissociation loss as well as the stagnation pressure losses associated with deceleration in supersonic-to-
subsonic inlets. With supersonic combustion, fluid temperatures are relatively low, and this decreases the
dissociation loss because dissociation depends on static rather than stagnation temperature. Wall heat
transfer, in contrast, depends essentially on stagnation temperature, so the wall-cooling problem is not
removed by employing supersonic combustion. The stagnation pressure loss due to supersonic heating
depends on the extent to which the fluid is being accelerated while combustion is taking place.

The use of supersonic combustion requires fuel to be injected into, and mixed with, a supersonic stream
without excessive shock losses.

www.Vidyarthiplus.com
www.Vidyarthiplus.com

www.Vidyarthiplus.com
www.Vidyarthiplus.com

The main advantages of the ramjet engine are;

1. High temperature can be employed


2. In the absence of rotating machinery its construction is very simple and cheap
3. It can operate effectively at high supersonic Mach numbers
4. It is not very sensitive to the quality of fuel
5. It provides high thrust per unit weight and frontal area

www.Vidyarthiplus.com
www.Vidyarthiplus.com

Ramjet engine’s Main disadvantages are;

1. It requires a launching device at supersonic speed


2. It is unsuitable for subsonic flight
3. It has low thermal efficiency and high TSFC
4. Its maximum operating altitude is limited

Ideal Efficiency

Various process occurring in the ramjet engine can be represented by an open circuit Brayton cycle. This
cycle is considered here with the following assumptions:

1. Steady one-dimensional flow

2. Isentropic compression and expansion, ∆s = 0, ∆P0 = 0

3. Perfect gas

4. Constant pressure heat addition in the combustion chamber, P2 = P3

5. Very low Mach number in the combustion chamber.

P2 ≈ P02 ≈ P03

T2 ≈ T02 ≈ T03

Compression and expansion process here are thermodynamically different from turbo jet engine.
These process here experience only energy transformation; there is no energy transfer such as
compressor and turbine work within the engine. Thrust work is obtained from the energy supplied in
the fuel during process 2-3. Heat is rejected during process 41 outside the engine. However, the
ideal efficiency of the ramjet engine is still given by Equation,

1 1 η=1− = 1−

i t r(γ−1)/γ
Here, the temperature ratio, with the above assumption is given by

www.Vidyarthiplus.com
www.Vidyarthiplus.com

T2 T02 T01 γ−1 2


t= = = = 1+ Mi
T1 T1 T1 2
Substituting the value of t from the above Equation to the previous equation we get,

1M12 ηi = 1−=

1+ M 12 1 M 12

1
ηi = 2 1 = f (M1)
1+ 2 γ−1 M1

Though the actual thermal efficiency of the ramjet engine will be much lower than the value given by
the above equation it demonstrates an important characteristic i.e., the efficiency increases with the
flight Mach number and has a high value (76.19 % at M1 = 4) at higher operating Mach numbers.

Other performance parameters and efficiencies defined for air-breathing engines are also applicable
for the ram jet engine.

www.Vidyarthiplus.com
www.Vidyarthiplus.com

The thermal efficiency or the Air standard Efficiency of the ideal cycle 1-2s-3-4s is given by,

= work / heat supplied = 1 –

Qr/Qs

= 1 – (T4s – T1)/(T3-T2s)

The pressure ratio in the compressor and turbine is same,

i.e., r = P2/P1 = P3/P4


Therefore the corresponding temperature ratios are given by,

T T = r(γ−1)/γ
P3
t = 2s = 3 =
T1 T4s PP12 = P4

1 1
ηAS =1−
t = 1− r(γ−1)/γ
(γ−1)/γ (γ−1)/γ

www.Vidyarthiplus.com
www.Vidyarthiplus.com

Numerical Problem

Q. A ramjet engine operates at M = 1.5 at an altitude of 6500 m. The diameter of the inlet diffuser
at entry is 50 cm and the stagnation temperature at the nozzle entry is 1600 K. The calorific value of
the fuel used is 40 MJ/kg. The properties of the combustion gases are same as those of air (γ = 1.4,
R = 287 J/kg K). The velocity of air at the diffuser exit is negligible.

Calculate (a) the efficiency of the ideal cycle (b) flight speed (c) air flow rate (d) diffuser pressure
ratio (e) fuel-air ratio (f) nozzle pressure ratio (g) nozzle jet Mach number (h) propulsive efficiency (i)
and thrust. Assue the following values; ηD = 0.90, ηB = 0.98, ηj = 0.96, stagnation pressure loss in
the combustion chamber = 0.02P02.

At Z = 6500 m the properties of air are

T1 = 245.90 K, p1 = 0.440 bar, a1 = 414.5 m/s

Ρ = 0.624 kg/m3

(a) Ideal cycle efficiency

1
η=i

www.Vidyarthiplus.com
www.Vidyarthiplus.com

2 1
1+ γ−1 M 2
1

= 0.310 Ans.

(b) M1 = u/a1

U = M1 a1 = 1.5 x 314.5 = 471.75 m/s

Flight speed = 1698.3 kmph Ans.

( C) Area of cross section of the diffuser inlet A1 = π d2/4 =

π 0.52/4 = 0.1963 m2

ma =ρ1uA1
= 0.624 X 471.48 X 0.1963

Air flow rate = 57.752 kg/s Ans.

(d) For negligible velocity at the diffuser exit,

P02 = P2

(γ−1)/γ

P2

−1
P
ηD = 1

M12
ηD = 0.9, M1 = 1.5

www.Vidyarthiplus.com
www.Vidyarthiplus.com

P2/P1 = 3.2875 Ans.

P2 = 3.2875 x 0.44 = 1.446

T01 γ−1 2 = 1 + M
= 1.45
1

(e)
T1 2
= 1.45 x 245.9 = 356.55 K

• •
Qf
ma C p (T03 −T02) =ηB m f

m
f= •f = C p (T03 − T02)/ηBQ f
ma
f = 1.005 (1600-356.55)/0.98 x 40000

Fuel air ratio = 0.03188 Ans

(f) P03 = P02 – 0.02 P02 = 0.98 P02


= 0.98 x 1.446 = 1.417 bar

Nozzle pressure ratio,

P03 = = 3.22

P4 1.
Ans
0.

www.Vidyarthiplus.com
www.Vidyarthiplus.com

(g) The Mach number at the nozzle exit for a pressure ratio of 3.22 in an isentropic expansion would be M4s
= 1.41; however, on account of irreversible expansion (ηj = 0.96) the exit
velocity and Mach number will be slightly lower.

T04 γ−1 2 2 = 1 + M4s = 1 + 0.2


x 1.41 1.3976
T 2 =
4s

T4s = 1600/1.3976 = 1144.82 K

T04 – T4 = ηj (T04-T4s)

= 0.96 (1600-1144.82)

= 1600 – 436.973 = 1163.027 K

T04 = T4 + C24/2Cp

C4 = 937.185 m/s a4 = (1.4 x 287 x

1163.027)0.5

= 683.596 m/s

Nozzle jet Mach number,

M4 = C4/a4 = 937.185/683.596 = 1.371 Ans

(h) σ = u/c4 = 471.48/937.185 = 0.503

ηp = 2σ/(1+ σ) = 2 x 0.503 /(1+0.503)


Propulsive efficiency = 0.6693 Ans
m = ma + m f =
(i)
57.752
• • •

m f = 57.752 x 0.03188 = 1.841 kg / s


• • •
+ 1.841 = 59.593 kg / s

www.Vidyarthiplus.com
www.Vidyarthiplus.com

F = mC4 − ma u
= (59.593 x 937.185 – 57.752 x 471.75) 10-3

Thrust = 28.614 kN Ans

Ram rocket
Air-augmented rockets (also known as rocket-ejector, ramrocket, ducted
rocket, integral rocket/ramjets, or ejector ramjets) use the supersonic exhaust of some kind of rocket engine
to further compress air collected by ram effect during flight to use as additional working mass, leading to
greater effective thrust for any given amount of fuel than either the rocket or a ramjet alone.

They represent a hybrid class of rocket/ramjet engines, similar to a ramjet, but able to give useful thrust from
zero speed, and are also able in some cases to operate outside the atmosphere, with fuel efficiency not
worse than both a comparable ramjet or rocket at every point.

Operation
A normal chemical rocket engine carries oxidizer and a fuel, sometimes pre-mixed, as in a solid rocket, which
are then burned. The heat generated greatly increases the temperature of the mixture, which is then
exhausted through a nozzle where it expands and cools. The exhaust is directed rearward through the nozzle,
thereby producing a thrust forward. In this conventional design, the fuel/oxidizer mixture is both the working
mass and energy source that accelerates it.

One method of increasing the overall performance of the system is to collect either the fuel or the oxidizer
during flight. Fuel is hard to come by in the atmosphere, but oxidizer in the form of gaseous oxygen makes
up 20% of the air and there are a number of designs that take advantage of this fact. These sorts of systems
have been explored in the LACE (liquid air cycle engine) concept.
Another idea is to collect the working mass instead. With an air-augmented rocket, an otherwise conventional
rocket engine is mounted in the center of a long tube, open at the front. As the rocket moves through the
atmosphere the air enters the front of the tube, where it is compressed via the ram effect. As it travels down
the tube it is further compressed and mixed with the fuelrich exhaust from the rocket engine, which heats the
air much as a combustor would in a ramjet. In this way a fairly small rocket can be used to accelerate a much
larger working mass than normally, leading to significantly higher thrust within the atmosphere.

A liquid air cycle engine (LACE) is a spacecraft propulsion engine that attempts to gain efficiency by
gathering part of its oxidizer from the atmosphere. In a LOX/LH2 bipropellant rocket the liquid oxygen needed

www.Vidyarthiplus.com
www.Vidyarthiplus.com

for combustion is the majority of the weight of the spacecraft on lift-off, so if some of this can be collected
from the air on the way, it might dramatically lower the take-off weight of the spacecraft.

Principle of operation
LACE works by compressing and then quickly liquefying the air. Compression is achieved through the ram-
air effect in an intake similar to that found on a high-speed aircraft like Concorde, where intake ramps create
shock waves that compress the air. The LACE design then blows the compressed air over a heat exchanger,
in which the liquid hydrogen fuel is flowing. This rapidly cools the air, and the various constituents quickly
liquefy. By careful mechanical arrangement the liquid oxygen can be removed from the other parts of the air,
notably water, nitrogen and carbon dioxide, at which point it can be fed into the engine as normal. The
hydrogen is so much lighter than oxygen that the now-warm hydrogen is often dumped overboard instead of
being re-used as fuel, at a net gain.

One issue with the LACE system is that in order to appreciably reduce the mass of the oxygen carried at
launch, a LACE vehicle needs to spend more time in the lower atmosphere where it can collect enough
oxygen to supply the engines. This leads to greatly increased vehicle heating and drag, which offset
somewhat the savings in oxidizer weight, but this in turn is offset by higher Isp (Specific impulse) permitting
a lifting trajectory which greatly reduces gravity losses. More significantly the LACE system is far heavier
than a rocket engine, and the performance of launch vehicles of all types is particularly affected by dry mass,
rather than any oxidizer mass which would be burnt off over the course of the flight.

Additional Reading:

1. Hill, P.G. & Peterson, C.R. “Mechanics & Thermodynamics of Propulsion” Addison – Wesley Longman INC, 1999.
2. Cohen, H., Rogers, G.F.C. and Saravanamuttoo, H.I.H., “Gas Turbine Theory”, Longman Co., ELBS Ed., 1989.
3. Gorden, C.V., “Aero thermodynamics of Gas Turbine and Rocket Propulsion”, AIAA Education Series, New York,
1989.
4. Mathur, M., and Sharma, R.P., “Gas Turbines and Jet and Rocket Propulsion”, Standard Publishers, New Delhi,
1988.
Unit-3 FUNDAMENTALS OF ROCKET PROPULSION
Operating principle – Specific impulse of a rocket – internal ballistics- Rocket nozzle classification – Rocket
performance considerations – Numerical Problems.

Rockets – Special Features and Applications Historical Reference


• The basic principles of all propulsive devices lie with the laws of motion due to Newton (17 th Century
AD). These laws are phenomenological and therefore one can expect that even before Newton there
may have existed many devices working on the principles of reaction.

www.Vidyarthiplus.com
www.Vidyarthiplus.com

• Rockets working directly on the principle of reaction are perhaps the simplest of the propulsive
engines.
• The reciprocating engines and gas turbine engines are relatively more complex.
• The Chinese are credited with the invention of rockets probably in 12-14th century AD.
• Indians used the rockets as effective weapons in late 18th century against British and in 19th century,
the rockets became a part of the warfare in Europe. But it was only in the early part of the present
century that man has recognized the full potential of rocket owing to the interests in space
travel/satellite technology and like.
• Tsiolkovsky (USSR 1903) Goddard (USA, 1912) and Oberth (1921) are the pioneers of modern
rocketry.
• The liquid propellant rocket owe their genesis to these people.
• The German V-2 rockets (25 tons, 65 sec, LOX-Alchol) and the post-second World war progress in
rocketry are too familiar to all.

Principle
All the conventional propulsion systems work by causing a change of momentum in a working fluid in a
direction opposite to the intended motion. Rockets fall under the category of direct acting engines – since the
energy liberated by the chemical process is directly used to obtain thrust. Being non-air breathing devices
the basic component of a rocket are

(i) Combustion chamber where exothermic processes produces gases at high temperature and
pressure, and
(ii) Nozzle, which accelerate the fluid to high velocities and discharge them into surrounding atmosphere
thereby deriving the desired force or thrust.
Some Special Features
The non-air breathing nature of rockets makes them very distinct among the propulsive devices.
(a) The reaction system does not depend on the surrounding atmosphere. There are no velocity
limitations and altitude ceiling.
(b) Since it has to carry its own oxidizer required for combustion reaction, the specific propellant
consumption is very high. Rockets consume approximately 15kg/kg-hr of propellant compared to
about 1 kg/kg-hr of fuel by turbojet engine.
(c) High pressure operation is possible and hence the ratio of energy liberation per unit volume (and
also unit weight of hardware) is very high.
(d) Main part of the rockets contains no moving element. Hence there is no constraint on internal
aerodynamics and the reliability is high. This also implies quick response times, which makes them
ideal control components.
With the above features, it is clear that the rockets are the most suitable power plants for

www.Vidyarthiplus.com
www.Vidyarthiplus.com

(i) High altitude and space applications where atmospheric oxygen is not available, eg. Launch
vehicles and satellite control rockets.
(ii) All applications where high thrust are required for short duration: missiles, boosters, JATO etc.

Rockets in Space Applications


There are a variety of rockets when it comes to launching and satellite control. Many of these are non-
chemical in nature but are restricted to extremely low thrust levels.
Order of
Sl
Type Magnitude of F/W Operational Applications
No
Thrust (N) Time Isp (sec)
Solar sail (not a
Satellite Altitude
1 rocket in fact) 10-5 10-4 Years ∞
control

Electric Prop.
(Electro thermal, Satellite control,
2 Electro Static, 10-6 - 10-2 10-5-10-3 Years 150 - 6000 stabilization, orbit
Electro maneuver
Magnetic)
Stored cold gas
3 10-2 - 10-1 ~ 10-3 Years 50 - 100 -do-
(N2, NH3 etc.)
Minute to Interplanetary and
4 Nuclear Rocket upto 105 20-30 800
hours space travel
Launch vehicles,
Chemical Rocket
Seconds to Missiles, Control
5 (Solid, Liquid and upto 107 upto 80 150-450
minutes@ rockets, Sounding
Hybrid)
rockets, JATO etc
@ shuttle main engine operate for about 8 min at a time but over 7 hrs cumulatively.

Classification of Chemical Rockets


Depending on the context, the chemical rockets are classified in many ways as follows:

(a) Type of propellant: Solid, Liquid (mono propellant / bipropellant and hybrid rockets) (b) Application:
Launch vehicle, ABM’s, JATO’s, ICBM, IRBM, SAM etc.
(c) Size of Unit (and thrust level sometimes): 10 ton, 100 kg etc.
(d) Type of subsystem: Turbopump fed, clustering, grain type etc.

Specific Impulse
• The specific impulse of a rocket, Isp, is the ratio of the thrust to the flow rate of the weight ejected, that is

www.Vidyarthiplus.com
www.Vidyarthiplus.com

where F is thrust, is the rate of mass flow, and g is the acceleration of gravity at ground level.
• Specific impulse is expressed in seconds. When the thrust and the flow rate remain constant throughout the
burning of the propellant, the specific impulse is the time for which the rocket engine provides a thrust equal
to the weight of the propellant consumed.

• For a given engine, the specific impulse has different values on the ground and in the vacuum of space
because the ambient pressure is involved in the expression for the thrust. It is therefore important to state
whether specific impulse is the value at sea level or in a vacuum.

• There are a number of losses within a rocket engine, the main ones being related to the inefficiency of the
chemical reaction (combustion) process, losses due to the nozzle, and losses due to the pumps.

• Overall, the losses affect the efficiency of the specific impulse. This is the ratio of the real specific impulse
(at sea level, or in a vacuum) and the theoretical specific impulse obtained with an ideal nozzle from gases
coming from a complete chemical reaction. Calculated values of specific impulse are several percent higher
than those attained in practice.

• From Equation (2.8) we can substitute C for F in Equation (2.23), thus obtaining

• Equation (2.24) is very useful when solving Equations (2.18) through (2.21). It is rare we are given the value
of C directly, however rocket engine specific impulse is a commonly given parameter.

Internal Ballistics
The parameters that govern the burning rate and mass discharge rate of rocket motors are called internal
ballistic properties; they include r – propellant burning rate (velocity of consumption), m/sec or
mm/sec or in/sec.
K– ratio of burning surface to throat area, Ab/At

σp – temperature sensitivity of burning rate, expressed as percent change of burning rate per degree
change in propellant temperature at a particular value of chamber pressure.

πK - temperature sensitivity of pressure expressed as percent change of chamber pressure per


degree change in propellant temperature at a particular value of K,
and the influences caused by pressure, propellant ingredients, gas velocity, or acceleration. The
subsequent solid propellant rocket parameters are performance parameters; they include thrust, ideal

www.Vidyarthiplus.com
www.Vidyarthiplus.com

exhaust velocity, specific impulse, propellant mass fraction, flame temperature, temperature limits and
duration.

Propellant Burning Rate


The rocket motor’s operation and design depend on the combustion characteristics of the propellant, its
burning rate, burning surface, and grain geometry. The branch of applied science describing these is known
as internal ballistics.

Solid propellant burns normal to its surface. The (average) burning rate, r, is defined as the regression of
the burning surface per unit time. For a given propellant, the burning rate is mainly dependent on the pressure,
p, and the initial temperature, Ti, of the propellant. Burning rate is also a function of propellant composition.
For composite propellants it can be increased by changing the propellant characteristics:

1. Add a burning rate catalyst, often called burning rate modifier (0.1 to 3.0% of propellant) or increase
percentage of existing catalyst.

2. Decrease the oxidizer particle size.

3. Increase oxidizer percentage

4. Increase the heat of combustion of the binder and/or the plasticizer 5. Imbed wires or metal staples

in the propellant

Apart from the propellant formulation and propellant manufacturing process, burning ratein a full-scale motor
can be increased by the following

1. Combustion chamber pressure


2. Initial temperature of the solid propellant prior to start
3. Combustion gas temperature
4. Velocity of the gas flow parallel to the burning surface
5. Motor motion ( acceleration and spin-induced grain stress)
Burning rate data are usually obtained in three ways – namely, from testing by:

1. Standard strand burners, often called Crawford burners


2. Small-scale ballistic evaluation motors
3. Full-scale motors with good instrumentation
A strand burner is a small pressure vessal (usually with windows) in which a thin strand or bar of
propellant is ignited at one end and burned to the other end. The strand can be inhibited with an

www.Vidyarthiplus.com
www.Vidyarthiplus.com

external coating so that it will burn only on the exposed cross-sectional surface; chamber pressure
is simulated by pressurizing the container with inert gas. The burning rate can be measured by
electric signals from embedded wires, by ultrasonic waves, or by optical means. The burning rate
measured on strand burners is usually lower than that obtained from actual rocket motor firing (by 4
to 12%) because it does not truly simulate the hot chamber environment of an actual rocket motor.
Also small ballistic evaluation motors usually have a slightly lower burning rate than full-scale large
motors, because of scaling factors.

During development of a new or modified solid propellant, it is tested extensively or characterized.


This includes the testing of the burn rate (in several different ways) under different temperatures,
pressures, impurities, and conditions. It also requires measurements of physical, chemical, and
manufacturing properties, ignitability, aging, sensitivity to various energy inputs or stimuli (e.g.,
shock, friction, fires), moister absorption, compatibility with other materials (liners, insulators, cases),
and other characteristics. It is a lengthy, expensive, often hazardous program with many tests,
samples, and analyses.

The burning rate of propellant in a motor is a function of many parameters, and at any

instant governs the mass flow rate m of hot gas generated and flowing from the motor (stable
combustion);

m = Ab r ρb

Here Ab is the burning area of the propellant grain, r the burning rate, and ρb the solid propellant
density prior to motor start. The total mass m of effective propellant burned can be determined by integrating
the above equation,

m = ∫mdt =ρb ∫ Ab r dt

Where Ab and r vary with time and pressure.


Burning Rate Relation with Pressure
Classical equations relating to burning rate are helpful in preliminary design, data extrapolation, and
understanding the phenomena. Unless otherwise stated, burning rate is expressed for 70 oF or 294 K
propellant (prior to ignition) burning at a reference chamber pressure of 1000 psia or 6.895 MPa. For most
production-type propellant the burning rate is approximated as a function of chamber pressure, at least for a
limited range of chamber pressures, which is given as r = a Pn
where r, the burn rate, is usually in centimeter per second and chamber pressure P is in MPa; a is an empirical
constant influenced by ambient temperature. Also a is known as the temperature coefficient and it is NOT

www.Vidyarthiplus.com
www.Vidyarthiplus.com

dimensionless. The burning rate exponent n, sometimes called the combustion index, is independent of the
initial grain temperature and describes the influence of chamber pressure on the burning rate.

Burning Rate Relation with Temperature

Temperature affects chemical reaction rates and the initial ambient temperature of a propellant grain prior to
combustion influences burning rate.

The sensitivity of burning rate to propellant temperature can be expressed in the form of temperature
coefficient, the two most common being
1
= ∂r
σp = ∂lnr
r ∂T p
∂T p
1 ∂P
πK = ∂ln p =
P ∂T K

∂T K
with σp , temperature sensitivity of burning rate and πK, temperature sensitivity of pressure.

The coefficient σp (typically 0.001 – 0.009 / K) for a new propellant is usually calculated from strand burner
test data, and πK (typically 0.067 – 0.278 % / oC) from small-scale or full-scale motors. Mathematically, these
coefficients are the partial derivatives of the natural logarithm of the burning rate r or the chamber pressure
p, respectively, with respect to propellant temperature T.

The values of πK and σp depend primarily on the nature of the propellant burning rate, the composition, and
the combustion mechanism of the propellant. It is possible to derive a relationship between the two
temperature sensitivities, namely
1
πK = σp
1− n
This formula is usually valid when the three variables are constant over the chamber pressure and
temperature range.

The temperature sensitivity σp can be also expressed as

∂ln(aPn ) 1 da σp = =
∂T p a dT

www.Vidyarthiplus.com
www.Vidyarthiplus.com

www.Vidyarthiplus.com
www.Vidyarthiplus.com

Equilibrium chamber pressure

In the above figure the straight line through the origin and point ‘S’ depicts the mass flow through the nozzle
as a function of Pc. At point S there is a balance between mass production and
__

outflux of the mass. At higher pressures (> Pc ) the mass flow through the nozzle is larger than the production
at the burning surface in case n < 1 and the reverse happens for n > 1. Thus if
__

n < 1 the pressure will drop to its steady-state value Pc . Note that when n < 1 even at higher chamber pressure
rocket motor will back to its designed equilibrium chamber pressure and ensure a stable operation. On the
other hand when n>1 these types of situations will possibly lead to over-pressurization and rupture of the
rocket motor or depressurization and flame out.

Erosive Burning
Erosive burning refers to the increase in the propellant burning caused by the high-velocity flow of combustion
gases over the burning propellant surface. It can seriously affect the performance of solid propellant rocket
motors. It occurs primarily in the port passages or perforations of the grain as the combustion gases flow
toward the nozzle; it is more likely to occur ehen the port passage cross-sectional area A is small relative to
the throat area At with a port-to-throat area ratio of 4 or less. The high velocity near the burning surface and

www.Vidyarthiplus.com
www.Vidyarthiplus.com

the turbulent mixing in the boundary layers increase the heat transfer to the solid propellant and thus increase
the burning rate.

Erosive burning increases the mass flow and thus also the chamber pressure and thrust during the early
portion of the burning for a particular motor (see above Fig.). Erosive burning causes early burnout of the
web, usually at the nozzle end, and exposes the insulation and aft closure to hot combustion gas for a longer
period of time; this usually requires more insulation layer thickness (and more inert mass) to prevent local
thermal failure. In designing motors, erosive burning is either avoided or controlled to be reproducible from
one motor to the next.

Total burning rate = steady state burning rate (aPcn ) + erosive burning

Basic Performance Relations


One basic performance relation derived from the principle of conservation of matter. The propellant mass
burned per unit time has to equal the sum of the change in gas mass per unit time in the combustion chamber
grain cavity and the mass flowing out through the exhaust nozzle per unit time.

(k+1)/(k−1) d
k 2
Ab rρb = dt (ρ1V1) + At P1 RT1 k +1

www.Vidyarthiplus.com
www.Vidyarthiplus.com

The term on the left side of the equation gives the mass rate of gas generation. The first term on the right
gives the change in propellant mass in the gas volume of the combustion chamber, and the last term gives
the nozzle flow. The burning rate of propellant is r; Ab is the propellant burning area; ρb is the solid propellant
density; ρ1 is the combustion gas density; V1 is the chamber gas cavity volume, which becomes larger as the
propellant is expended; At is the throat area; P1 is the chamber pressure; T1 is the absolute chamber
temperature, which is usually assumed to be constant; and k is the specific heat ratio of the combustion
gases. During startup the changing mass of propellant in the grain cavity becomes important.

Isentropic Flow through Nozzles


In a converging diverging nozzle a large fraction of the thermal energy of the gases in the chamber is
converted into kinetic energy. As will be explained, the gas pressure and temperature drop dramatically and
gas velocity can reach values in excess of around 3.2 km/sec. This is a reversible, essentially isentropic flow
process.

If a nozzle inner wall has a flow obstruction or a wall protrusion (a piece of weld splatter or slag), then the
kinetic gas energy is locally converted back into thermal energy essentially equal to the stagnation
temperature and stagnation pressure in the chamber. Since this would lead quickly to a local overheating and
failure of the wall, nozzle inner walls have to be smooth without any protrusion.
Nozzle exit velocity can be derived as,

v2 = k2−k1RT1 1− PP12 (k−1)/ k +v12

www.Vidyarthiplus.com
www.Vidyarthiplus.com

This relation also holds for any two points within the nozzle. Note that when the chamber section is large
compared to the nozzle throat section, the chamber velocity or nozzle approach velocity is comparatively
small and the v12 can be neglected. The chamber temperature T1 is at the nozzle inlet and, under isentropic
condition, differ little from the stagnation temperature or (for a chemical rocket) from combustion temperature.
This leads to an important simplified expression of the exhaust velocity v2, which is often used in the analysis.

2k 1− P (k−1)/ k

v2 = −1RT1 P12 k

= k2−k1RM'To 1− PP12 (k−1) / k

Thrust and Thrust Coefficient

F = mv2 + (p2 −p3)A2

www.Vidyarthiplus.com
www.Vidyarthiplus.com

F =C F At P1
Where CF is the thrust coefficient, which can be derived as a function of gas property k, the nozzle area ratio
(A2/At), and the pressure ratio across the nozzle p1/p2 , but independent of chamber temperature. For any
fixed pressure ratio (p1/p3) the thrust coefficient CF and the thrust F have a peak when p2 = p3. This peak value
is known as optimum thrust coefficient.

2k 2 2 (k+1)/(k−1)

CF = 1− pp12 (k−1)/ k + p2 p−1p3 AA2t k−1 k+1

Effective Exhaust Velocity


In a rocket nozzle the actual exhaust velocity is not uniform over the entire exit cross-section and does not
represent the entire thrust magnitude. The velocity profile is difficult to measure accurately. For convenience
a uniform axial velocity ‘c’ is assumed which allows a onedimensional description of the problem. This
effective exhaust velocity ‘c’ is the average equivalent velocity at which propellant is ejected from the vehicle.
It is defined as

F
c = Isp go = •

It is usually given in meters per second.

The concept of weight relates to the gravitational attraction at or near sea level, but in space or outer satellite
orbits, “weight” signifies the mass multiplied by an arbitrary constant, namely g o. In system international (SI)
or metric system of units Isp can be expressed simply in “seconds”, because of the use of the constant g o.

Specific Propellant Consumption


Specific propellant consumption is the reciprocal of the specific impulse.

Mass Ratio
The mass ratio of a vehicle or a particular vehicle stage is defined to be the final mass mf (after rocket
operation has consumed all usable propellant) divided by initial mass mo (before rocket operation).

www.Vidyarthiplus.com
www.Vidyarthiplus.com

This applies to a single or multi-stage vehicle. The final mass mf is the mass of the vehicle after the rocket
has ceased to operate when all the useful propellant mass m p has been consumed and ejected. The final
vehicle mass mf includes all those components that are not useful propellant and may include guidance
devices, navigation gear, payload (e.g., scientific instruments or a military warhead), flight control systems,
communication devices, power supplies, tank structure, residual or unusable propellant, and all the propulsion
hardware. In some vehicles it can also include wings, fins, a crew, life support systems, reentry shields,
landing gears etc. Typical value of Mass ratio can range from 60% for tactical missiles to less than 10 % for
unmanned launch vehicle stages. This mass ratio is an important parameter in analyzing flight performance.
Note that when mass-ratio is applied to a single stage of a multi-stage rocket, then its upper stages become
the “payload”

Propellant Mass Fraction, ζ

Propellant mass fraction ‘ζ’ is defined as the ratio of propellant mass ‘mp’ to the initial mass ‘mo’

mp
ζ= mo
mo = mf + mp

Characteristic Velocity
The characteristic velocity has been used frequently in the rocket propulsion literature. It is represented by a
symbol C*. It is defined as,

∗ p1 A t
C= •

m
The characteristic velocity is used in comparing the relative performance of different chemical rocket
propulsion system designs and propellants. It is basically a function of the propellant

characteristics. It is easily determined from data of m , p1, and At. It relates to the efficiency of the combustion
and is essentially independent of nozzle characteristics. However, the specific impulse and the effective
exhaust velocities are functions of the nozzle geometry, such as the nozzle area ratio.

www.Vidyarthiplus.com
www.Vidyarthiplus.com

Problems:
Q1.

A rocket projectile has the following characteristics:


Initial mass = 200kg
Mass after rocket operation = 130 kg
Payload, nonpropulsive structure, etc., = 110 kg
Rocket operation duration = 3.0 sec
Average specific impulse of propellant = 240 sec
Determine the vehicle’s (i) mass ratio, (ii) propellant mass fraction, (iii) propellant flow rate, (iv) thrust, (v)
thrust-to-weight ratio, (vi) acceleration of the vehicle, (vii) effective exhaust velocity, (viii) total impulse, and
(ix) the impulse to weight ratio.

Solution:

(i) Mass ratio of vehicle, mf/mp = 130/200 = 0.65


Mass ratio of rocket system = mf/mo = (130-110)/(200-110) = 20/90 = 0.222

(ii) Propellant mass fraction = (mo – mf)/mo = (90-20)/90 = 0.778


(iii)Propellant mass flow rate = 70/3 = 23.3 kg/sec

(iv) Thrust = Isp mgo = 240 x 23.3 x 9.81 = 54.857 N

(v) Thrust-to-weight ratio of the vehicle is


Initial value = 54,857 / (200 x 9.81) = 28

Final value = 54,857 / (130 x 9.81) = 43

(vi) Maximum acceleration of the vehicle is 43 x 9.81 = 421 m/sec2


(vii) The effective exhaust velocity is
c = Isp go = 240 x 9.81 = 2354 m/sec

(viii) Total impulse = Isp w = 240 x 70 x 9.81 = 164, 808 N-sec


This result can also be obtained by multiplying the thrust by the duration.

(ix) The impulse to weight ratio of the propulsion system is

www.Vidyarthiplus.com
www.Vidyarthiplus.com

= 164,808 / [(200-110) x 9.81] = 187

Q2. The following measurements were made in a sea level test of a solid propellant rocket motor:
Burn duration = 40 sec
Initial mass before test = 1210 kg
Mass of rocket motor after test = 215 kg
Average thrust = 62,250 N
Chamber pressure = 7.00 MPa
Nozzle exit pressure = 0.070 MPa
Nozzle throat diameter = 0.855 m Nozzle exit
diameter = 0.2703 m

Determine m,v2 ,c∗ ,c and Isp at sea level, and c and Isp at 1000 and 25,000 m altitude.
Assume an invariant thrust and mass flow rate and negligible short start and stop transients.

Solution:

Mass flow rate = (initial motor mass – final motor mass)/burn time
= (1210 – 215) / 40 = 24.9 kg/sec
The nozzle areas at the throat and exit are
At = π D2/4 = π x 0.08552 /4 = 0.00574 m2
A2 = π D2/4 = π x 0.27032 /4 = 0.0574 m2 The actual
exhaust velocity

(F−(p2 − p3)A2) v2 =

m
= (62,250 – (0.070-0.1013) 106 x 0.0574) / 24.9
= 2572 m/sec

∗ p1 At
C= • m
= 7.00 x 106 x
0.00574/24.9 = 1613 m/sec Isp =
62,250 / (24.9 x 9.81) = 255 sec c = 255 x 9.81 =
2500 m/sec
For altitudes of 1000 and 25, 000 m the ambient pressure (see atmospheric table) is 0.898 and
0.00255 MPa.

www.Vidyarthiplus.com
www.Vidyarthiplus.com

(p2 −p3)A2 c=v2 +


• m
At 1000 m altitude,
c = 2572 + (0.070-0.898) x 106 x 0.0574/24.9 = 2527 m/sec
Isp = 2527/9.81 = 258 sec

At 25,000 m altitude,
c = 2572 + (0.070-0.00255) x 106 x 0.0574/24.9 = 2727 m/sec
Isp = 2727/9.81 = 278 sec

Rocket Nozzles
Purpose:

www.Vidyarthiplus.com
www.Vidyarthiplus.com

The nozzle is the component of a rocket or air-breathing engine that produces thrust. This is accomplished
by converting the thermal energy of the hot chamber gases into kinetic energy and directing that energy along
the nozzle's axis, as illustrated below.

Simple representation of a rocket nozzle

Although simplified, this figure illustrates how a rocket nozzle works. The propellant is composed of a fuel,
typically liquid hydrogen (H 2), and an oxidizer, typically liquid oxygen (O
2).The propellant is pumped into a combustion chamber at some rate (mdot) where the fuel and oxidizer
are mixed and burned. The exhaust gases from this process are pushed into the throat region of the nozzle.
Since the throat is of less cross-sectional area than the rest of the engine, the gases are compressed to a
high pressure. The nozzle itself gradually increases in cross-sectional area allowing the gases to expand. As
the gases do so, they push against the walls of the nozzle creating thrust.

Mathematically, the ultimate purpose of the nozzle is to expand the gases as efficiently as possible so as to
maximize the exit velocity (v exit). This process will maximize the thrust (F) produced by the system since the
two are directly related by the equation

where

F = thrust force
= mass flow rate

www.Vidyarthiplus.com
www.Vidyarthiplus.com

v exit = exhaust gas velocity at the nozzle exit p exit =


pressure of the exhaust gases at the nozzle exit
p = ambient pressure of the atmosphere
A exit = cross-sectional area of the nozzle exit

Expansion Area Ratio:

In theory, the only important parameter in rocket nozzle design is the expansion area ratio ( ), or the ratio of
exit area (A exit) to throat area (A throat).

Fixing all other variables (primarily the chamber pressure), there exists only one such ratio that optimizes
overall system performance for a given altitude (or ambient pressure). However, a rocket typically does not
travel at only one altitude. Thus, an engineer must be aware of the trajectory over which a rocket is to travel
so that an expansion ratio that maximizes performance over a range of ambient pressures can be selected.

Nevertheless, other factors must also be considered that tend to alter the design from this expansion ratio-
based optimum. Some of the issues designers must deal with are nozzle weight, length, manufacturability,
cooling (heat transfer), and aerodynamic characteristics.

Typical temperatures (T) and pressures (p) and speeds (v) in a De Laval Nozzle
Maximum thrust for a rocket engine is achieved by maximizing the momentum contribution of the equation
without incurring penalties from over expanding the exhaust. This occurs when Pe = Pamb. Since ambient
pressure changes with altitude, most rocket engines spend very little time operating at peak efficiency.

www.Vidyarthiplus.com
www.Vidyarthiplus.com

If the pressure of the exhaust jet varies from atmospheric pressure, nozzles can be said to be underexpanded,
ambient or overexpanded. If under or overexpanded then loss of efficiency occurs, grossly overexpanded
nozzles lose less efficiency, but the exhaust jet is usually unstable. Rockets become progressively more
underexpanded as they gain altitude. Note that almost all rocket engines will be momentarily grossly
overexpanded during startup in an atmosphere.

Rocket Nozzle Shapes

Not all rocket nozzles are alike, and the shape selected usually depends on the application.
This section discusses the basic characteristics of the major classes of nozzles used today.

Nozzle Comparisons:

To date three major types of nozzles, the cone, the bell or contoured, and the annular or plug, have been
employed. Each class satisfies the design criteria to varying degrees. Examples of these nozzle types
can be seen below.

www.Vidyarthiplus.com
www.Vidyarthiplus.com

Size comparison of optimal cone, bell, and radial nozzles for a given set of
conditions

Conical Nozzle:

The conical nozzle was used often in early rocket applications because of its simplicity and ease of
construction. The cone gets its name from the fact that the walls diverge at a constant angle. A small
angle produces greater thrust, because it maximizes the axial component of exit velocity and produces a
high specific impulse (a measure of rocket efficiency). The penalty, however, is a longer and heavier
nozzle that is more complex to build. At the other extreme, size and weight are minimized by a large
nozzle wall angle. Unfortunately, large angles reduce performance at low altitude because the high
ambient pressure causes overexpansion and flow separation.

Bell Nozzle:

The bell, the most commonly used nozzle shape, offers significant advantages over the conical nozzle,
both in size and performance. Referring to the above figure, note that the bell consists of two sections.
Near the throat, the nozzle diverges at a relatively large angle but the degree of diveregence tapers off
further downstream. Near the nozzle exit, the diveregence angle is very small. In this way, the bell is a
compromise between the two extremes of the conical nozzle since it minimizes weight while maximizing
performance. The most important design issue is to contour the nozzle to avoid oblique shocks and
maximize performance. However, we must remember that the final bell shape will only be the optimum at
one particular altitude.
Annular Nozzles:

www.Vidyarthiplus.com
www.Vidyarthiplus.com

The annular nozzle, also sometimes known as the plug or "altitude-compensating" nozzle, is the least
employed of those discussed due to its greater complexity. The term "annular" refers to the fact that
combustion occurs along a ring, or annulus, around the base of the nozzle. "Plug" refers to the centerbody
that blocks the flow from what would be the center portion of a traditional nozzle. "Altitude-compensating"
is sometimes used to describe these nozzles since that is their primary advantage, a quality that will be
further explored later.

Before describing the various forms of annular nozzles, it is useful to mention some key differences in
design parameters from the conical or bell nozzles. The expansion area ratio for a traditional nozzle has
already been discussed. When considering an annular nozzle, the area of the centerbody (A plug) must
also be taken into account.

Another parameter particular to this type of nozzle is the annular diameter ratio, D p / D t, or the ratio of
the centerbody diameter to that of the throat. The ratio is used as a measure of the nozzle geometry for
comparison with other plug nozzle shapes. Typical values of this ratio appear in the above figure.

Annular Nozzles I
Having introduced the three principal families of nozzle shapes, we will now look more closely at the two
major subclasses of annular, or plug, nozzles.

Radial Out-Flow Nozzles:

Two major types of plug nozzles have been developed to date. They are distinguished by the method in which
they expand the exhaust, outward or inward. The radial out-flow nozzle was the subject of much research in
the late 1960s and early 1970s. Examples of this type are the expansion-deflection (E-D), reverse-flow (R-
F), and horizontal-flow (H-F) nozzles shown in the figure above.

The name of each of these nozzles indicates how it functions. The expansion-deflection nozzle works much
like a bell nozzle since the exhaust gases are forced into a converging throat region of low area before
expanding in a bell-shaped nozzle. However, the flow is deflected by a plug, or centerbody, that forces the
gases away from the center of the nozzle. Thus, the E-D is a radial out-flow nozzle.

The reverse-flow nozzle gets its name because the fuel is injected from underneath, but the exhaust gases
are rotated 180° thereby reversing their direction. Similarly, the fuel in the horizontal-flow nozzle is injected
sideways, but the exhaust is rotated 90°.
Judging by the amount of literature obtained on this subject, little work has been done on the R-F and H-F
nozzles, and they will not be considered further. The E-D, on the other hand, has been one of the most
studied forms of annular nozzles. While similar in nature to the bell nozzle, the most notable difference is the

www.Vidyarthiplus.com
www.Vidyarthiplus.com

addition of a centerbody. As shown below, this "plug" may be located upstream of, downstream of, or in the
throat, with each location resulting in better performance for a given set of operating conditions.

Comparison of centerbody locations in Expansion-Deflection nozzles


[from Conley et al, 1984]

The purpose of the centerbody is to force the flow to remain attached to, or to stick to, the nozzle walls, as
illustrated in the following figure.

www.Vidyarthiplus.com
www.Vidyarthiplus.com

Expansion-deflection nozzle flow behavior at low altitude


[from Sutton, 1992]

This behavior is desirable at low altitudes because the atmospheric pressure is high and may be greater than
the pressure of the exhaust gases. When this occurs, the exhaust is forced inward and no longer exerts force
on the nozzle walls, so thrust is decreased and the rocket becomes less efficient. The centerbody, however,
increases the pressure of the exhaust gases by squeezing the gases into a smaller area thereby virtually
eliminating any loss in thrust at low altitude.

Annular Nozzles II
Having introduced the three principal families of nozzle shapes and discussed the radial out-flow nozzle, we
will now look more closely at the second class of annular nozzles.

Radial In-Flow Nozzles:

The second major variety of annular nozzles is the radial in-flow type, exemplified by the spike shown above.

This type of nozzle, named for the prominent spike centerbody, is often described as a bell turned inside out.
However, the nozzle shown above is only one of many possible spike configurations. Variations of this design,
shown below, include

(a) a traditional curved spike with completely external supersonic expansion


(b) a similar shape in which part of the expansion occurs internally
(c) a design similar to the expansion-deflection nozzle in which all expansion occurs internally.

www.Vidyarthiplus.com
www.Vidyarthiplus.com

Comparison of spike nozzles with (a) external expansion, (b) internal-external expansion, and (c)
internal expansion [from Berman and Crimp, 1961]

Note that each of the above spike nozzles features a curved, pointed spike, the most ideal shape. This spike
shape allows the exhaust gases to expand through an isentropic, or constant entropy, process. In so doing,
the nozzle efficiency is maximized and no energy is lost because of turbulent mixing in the exhaust flow.
While the isentropic spike may be most efficient, it also tends to be prohibitively long and heavy. However,
theoretical studies have shown that replacing the curved shape by a much shorter and easier to construct
cone results in very little performance loss. The following graph illustrates that the thrust decreases by less
than 1% for cone halfangles up to 30°. Furthermore, the graph gives an indication of how much the spike
length can be reduced by employing a cone-shaped spike.

Aerospike Nozzles
A further subclass of the radial in-flow family of spike nozzles is known as the aerospike.

Aerospike Nozzles:

Previously, we discussed methods of reducing the length of a spike nozzle centerbody by replacing the ideal
spike with a conical spike. While this method does indeed result in a much shorter nozzle length, we can go
even further by removing the pointed spike altogether and replacing it with a flat base. This configuration is
known as a truncated spike, an example of which is shown below.

www.Vidyarthiplus.com
www.Vidyarthiplus.com

Example of a truncated, conical spike [from Berman and Crimp, 1961]

As any fluid dynamicist recognizes, the significant disadvantage of the "flat" plug is that a turbulent wake
forms aft of the base at high altitudes resulting in high base drag and reduced efficiency. However, this
problem can be greatly alleviated in an improved version of the truncated spike that introduces a "base bleed,"
or secondary subsonic flow, into the region aft of the base.

Example of an aerospike nozzle with a subsonic, recirculating flow


[from Hill and Peterson, 1992]

The circulation of this secondary flow and its interaction with the engine exhaust creates an "aerodynamic
spike" that behaves much like the ideal, isentropic spike. In addition, the secondary flow re-circulates upward

www.Vidyarthiplus.com
www.Vidyarthiplus.com

pushing on the base to produce additional thrust. It is this artificial aerodynamic spike for which the aerospike
nozzle is named.

Linear Aerospike:

All of the nozzles we have studied thus far have been annular, or circular when viewed from below. Still
another variation of the aerospike nozzle is not an annular nozzle at all. A second approach, pioneered by
the Rocketdyne company (now a division of Boeing) in the 1970s, places the combustion chambers in a line
along two sides of the nozzle:

Rocketdyne RS-2200 linear aerospike engine [from Flinn, 1996]

This approach results in a more versatile design allowing the use of lower-cost modular combustors. These modules
can be combined in varying configurations depending on the application.

Aerospike Flowfield:

The exact nature of the exhaust flowfield behind an aerospike nozzle is currently the subject of much
research. The most notable features of a typical aerospike nozzle flowfield are shown in more detail below.

www.Vidyarthiplus.com
www.Vidyarthiplus.com

Flowfield characteristics of an aerospike nozzle [from Ruf and McConnaughey, 1997]

The primary exhaust can be seen expanding against the centerbody and then around the corner of the base
region. The interaction of this flow with the re-circulating base bleed creates an inner shear layer. The outer
boundary of the exhaust plume is free to expand to ambient pressure. Expansion waves can be seen
emanating from the thruster exit lip, and these waves reflect from the centerbody contour to the free jet
boundary. Compression waves are then reflected back and may merge to form the envelope shock seen in
the primary exhaust.

At low altitude (high ambient pressure), the free boundary remains close to the nozzle (see below) causing
the compression waves to reflect onto the centerbody and shear layer themselves. The waves impacting the
centerbody increase pressure on the surface, thereby increasing the centerbody component of thrust. The
waves impacting the shear layer, on the other hand, increase the circulation of the base flow thereby
increasing the base component of thrust.

www.Vidyarthiplus.com
www.Vidyarthiplus.com

Aerospike nozzle behavior during flight [from Rocketdyne, 1999]

• Thrust vectoring: Because the combustion chambers can be controlled individually, the vehicle can
be maneuvered using differential thrust vectoring. This eliminates the need for the heavy gimbals
and actuators used to vary the direction of traditional nozzles.

Aerospike thrust vectoring control [from Rocketdyne, 1999]

Additional Reading

1. Sutton, G.P., “Rocket Propulsion Elements”, John Wiley & Sons Inc., New York, 7th Edn., 2001.

Unit-4
CHEMICAL ROCKETS
Solid propellant rockets – Selection criteria of solid propellants – Important hardware components of solid
rockets – Propellant grain design considerations – Liquid propellant rockets – Selection of liquid propellants
– Thrust control in liquid rockets – Cooling in liquid rockets – Limitations of hybrid rockets – Relative
advantages of liquid rockets over solid rockets- Numerical Problems.

Introduction to Propulsion
Definitions and Basic Relations

• Fluid dynamics of compressible flows is generally referred to as Gas dynamics. This deals with an
unified analysis of dynamics and thermodynamics of compressible flows.

• Convectional fluid mechanics analyses are inadequate for high speed flows of gases and vapours
due to non-compressibility approach.

www.Vidyarthiplus.com
www.Vidyarthiplus.com

• Therefore in the application like high speed aerodynamics, rocket and missile propulsion, steam
and gas turbines, and high speed turbocompressors compressible fluid dynamics is used to obtain
solutions of a number of design problems.

• The properties of fluid which are generally considered in compressible flow problems are
temperature, pressure, density, internal energy, enthalpy, entropy and viscosity.

• A major portion covered by the fluid dynamics of compressible flows deals with the relation
between force, mass and velocity.

• The following laws are frequently used in dealing with a variety of compressible flow problems:

• (i) First law of thermodynamics


• (Energy equation).
• (ii) Second law of thermodynamics
• (Entropy relation)
• (iii) Law of conservation of mass
• (Continuity equation)
• (iv) Newton’s second law of motion
• (Momentum equation)

Goal: Create a Force to Propel a Vehicle

Two options:

1) Take mass stored in a vehicle and throw it backwards (rocket propulsion). Use the reaction force to
propel the vehicle.

Propellant ---> burn ---> expand through nozzle

(chem. energy) (thermal energy) (kinetic energy & momentum)

2) Seize mass from the surroundings and set the mass in motion backwards. Use the reaction force to propel
vehicle (air-breathing propulsion).

• Continuously:

a) Draw in air.

b) Compress it.

www.Vidyarthiplus.com
www.Vidyarthiplus.com

c) Add fuel and burn (convert chemical energy to thermal energy).

d) Expand through a turbine to drive compressor (extract work).

e.1) Then expand in a nozzle to convert thermal energy to kinetic energy & momentum (turbojet).

e.2) Or expand in a second turbine (extract work), use this to drive a shaft for a fan
(turbofan), or a propeller (turboshaft). The fan or propeller impart k.e. & mom. to the air.

*Remember:

• Overall goal: take at Vo (flight speed), throw it out at Vo + DV

• Flying model rockets is a relatively safe and inexpensive way for students to learn the basics of forces and
the response of a vehicle to external forces. Like an airplane, a model rocket is subjected to the forces of
weight, thrust, and aerodynamics during its flight.

• On this slide we show the parts of a single stage model rocket. We have laid the rocket on its side and cut a
hole in the body tube so that we can see what is inside. Beginning at the far right, the body of the rocket is a
green cardboard tube with black fins attached at the rear. The fins can be made of either plastic or balsa
wood and are used to provide stability during flight. Model rockets use small, pre-packaged, solid fuel

www.Vidyarthiplus.com
www.Vidyarthiplus.com

engines The engine is used only once, and then is replaced with a new engine for the next flight. Engines
come in a variety of sizes and can be purchased at hobby stores and at some toy stores.

www.Vidyarthiplus.com
www.Vidyarthiplus.com

www.Vidyarthiplus.com
www.Vidyarthiplus.com

www.Vidyarthiplus.com
www.Vidyarthiplus.com

We live in a world that is defined by three spatial dimensions and one time dimension. Objects move within this
domain in two ways. An object translates, or changes location, from one point to another. And an object rotates, or
changes its attitude. In general, the motion of any object involves both translation and rotation. The translations are in
direct response to external forces. The rotations are in direct response to external torques or moments (twisting
forces).

• The motion of a rocket is particularly complex because the rotations and translations are coupled together; a
rotation affects the magnitude and direction of the forces which affect translations.

• To understand and describe the motion of a rocket, we usually try to break down the complex problem into a
series of easier problems.

• We can, for instance, assume that the rocket translates from one point to another as if all the mass of the
rocket were collected into a single point called the center of gravity.

• We can describe the motion of the center of gravity by using Newton's laws of motion.

• In general, there are four forces acting on the rocket; the weight, thrust, drag and lift.

www.Vidyarthiplus.com
www.Vidyarthiplus.com

• The thrust of the engine is transmitted to the body of the rocket through the engine mount.

• This part is fixed to the rocket and can be made of heavy cardboard or wood.

• There is a hole through the engine mount to allow the ejection charge of the engine to pressurize the body
tube at the end of the coasting phase and eject the nose cone and the recovery system.

• Recovery wadding is inserted between the engine mount and the recovery system to prevent the hot gas of
the ejection charge from damaging the recovery system.

• The recovery wadding is sold with the engine.

• The recovery system consists of a parachute (or a streamer) and some lines to connect the parachute to the
nose cone.

• Parachutes and streamers are made of thin sheets of plastic.

• The nose cone can be made of balsa wood, or plastic, and may be either solid or hollow. The nose cone is
inserted into the body tube before flight.

www.Vidyarthiplus.com
www.Vidyarthiplus.com

• An elastic shock cord is connected to both the body tube and the nose cone and is used to keep all the parts
of the rocket together during recovery.

• The launch lugs are small tubes (straws) which are attached to the body tube.

• The launch rail is inserted through these tubes to provide stability to the rocket during launch.

www.Vidyarthiplus.com
www.Vidyarthiplus.com

www.Vidyarthiplus.com
www.Vidyarthiplus.com

www.Vidyarthiplus.com
www.Vidyarthiplus.com

Temperature Variation in the Atmosphere

www.Vidyarthiplus.com
www.Vidyarthiplus.com

SOLID ROCKETS
Specific Impulse: 100-400 sec

www.Vidyarthiplus.com
www.Vidyarthiplus.com

Thrust: 103-107 N

• Solid rockets are the simplest and earliest types of rocket propulsion dating back to the first rockets used
by the Chinese.
• Solid rockets are filled with a solid mixture of a propellant and an oxidizer. Little else is actually required
for these rockets.
• The designs are very simple and therefore very reliable.
• The main drawback of solid rockets is that once ignited, they burn until all of the fuel is gone. Because of
this, they aren't used often in space where propulsion systems are usually required to be turned on and
off many times. However, they are good for getting things into space. In fact, the space shuttles use solid
rocket boosters (SRBs) during takeoff.

Quick Fact : The SRBs are the largest solid-propellant motors ever flown and the first designed for reuse.
Each is 149.16 feet long and 12.17 feet in diameter.

MONOPROPELLANT ROCKETS
Specific Impulse: 100-300 sec
Thrust: 0.1-100 N
• Monopropellant rockets are simple propulsion systems that rely on special chemicals which, when
energized, decompose. This decomposition creates both the fuel and an oxidizer (which allows the fuel
to burn), which then react with each other. Because they only use a single propellant, monopropellant
rockets are quite simple and reliable. Unfortunately, they are not very efficient. They are mainly used to
make small adjustments such as attitude control. Main propulsion systems usually use some other
technology.

BIPROPELLANT ROCKETS
Specific Impulse: 100-400 sec Thrust: 0.1-
107 N

• Bipropellant rockets separate the fuel and oxidizer and mix them in the chamber where they burn.
Bipropellant rockets are widely used and more efficient than monopropellant rockets. The reaction
given in the lesson on chemistry gives an example of a fuel(H2)/oxidizer(O2) combination. It's actually
a very good combination in that it releases a large amount of energy. It's the combination used by
the space shuttle's main engines. Unfortunately, large tanks kept at extremely low temperatures are
required to carry them. In fact, the main purpose of the giant red external tank attached to the space
shuttle on take-off is to carry enough fuel to get the space shuttle into space.

The main drawback of bipropellant rockets is that they are more complex than solid or monopropellant
rockets. The fuel and oxidizer have to be stored separately and fed together in exactly the right ratios to

www.Vidyarthiplus.com
www.Vidyarthiplus.com

achieve maximum efficiency. Despite the extra complexity, bipropellant rockets are still one of the preferred
systems for primary propulsion

ROCKET PROPULSION
Isaac Newton stated in his third law of motion that "for every action there is an equal and opposite reaction." It is upon this principle
that a rocket operates. Propellants are combined in a combustion chamber where they chemically react to form hot gases which
are then accelerated and ejected at high velocity through a nozzle, thereby imparting momentum to the engine. The thrust force of
a rocket motor is the reaction experienced by the motor structure due to ejection of the high velocity matter. This is the same
phenomenon which pushes a garden hose backward as water flows from the nozzle, or makes a gun recoil when fired.

Thrust
Thrust is the force that propels a rocket or spacecraft and is measured in pounds,
kilograms or Newtons. Physically speaking, it is the result of pressure which is
exerted on the wall of the combustion chamber.

The figure to the right shows a combustion chamber with an opening, the nozzle,
through which gas can escape. The pressure distribution within the chamber is
asymmetric; that is, inside the chamber the pressure varies little, but near the
nozzle it decreases somewhat. The force due to gas pressure on the bottom of
the chamber is not compensated for from the outside. The resultant force F due
to the internal and external pressure difference, the thrust, is opposite to the
direction of the gas jet. It pushes the chamber upwards.

To create high speed exhaust gases, the necessary high temperatures and
pressures of combustion are obtained by using a very energetic fuel and by
having the molecular weight of the exhaust gases as low as possible. It is also
necessary to reduce the pressure of the gas as much as possible inside the
nozzle by
creating a large section ratio. The section ratio, or expansion ratio, is defined as the
area of the exit Ae divided by the area of the throat At.

The thrust F is the resultant of the forces due to the pressures exerted on the
inner and outer walls by the combustion gases and the surrounding atmosphere,
taking the boundary between the inner and outer surfaces as the cross section of the exit of the nozzle. As we shall see in the next
section, applying the principle of the conservation of momentum gives

where q is the rate of the ejected mass flow, Pa the pressure of the ambient atmosphere, Pe the pressure of the exhaust gases
and Ve their ejection speed. Thrust is specified either at sea level or in a vacuum.

Conservation of Momentum
The linear momentum (p), or simply momentum, of a particle is the product of its mass and its velocity. That is,

www.Vidyarthiplus.com
www.Vidyarthiplus.com

Newton expressed his second law of motion in terms of momentum, which can be stated as "the resultant of the forces acting on
a particle is equal to the rate of change of the linear momentum of the particle". In symbolic form this becomes

which is equivalent to the expression F=ma.

If we have a system of particles, the total momentum P of the system is the sum of the momenta of the individual particles. When
the resultant external force acting on a system is zero, the total linear momentum of the system remains constant. This is called
the principle of conservation of linear momentum. Let's now see how this principle is applied to rocket mechanics.

Consider a rocket drifting in gravity free space. The rocket's engine is fired for time t and, during this period, ejects gases at a
constant rate and at a constant speed relative to the rocket (exhaust velocity). Assume there are no external forces, such as gravity
or air resistance.

The figure below-left (a) shows the situation at time t. The rocket and fuel have a total mass M and the combination is moving with
velocity v as seen from a particular frame of reference. At a time t later the configuration has changed to that shown below-right
(b). A mass M has been ejected from the rocket and is moving with velocity u as seen by the observer. The rocket is reduced to
mass M- M and the velocity v of the rocket is changed to v+ v.

Because there are no external forces, dP/dt=0. We can write, for the time interval t

where P2 is the final system momentum, figure (b), and P1 is the initial system momentum, figure (a). We write

If we let t approach zero, v/ t approaches dv/dt, the acceleration of the body. The quantity M is the mass ejected in t; this leads to
a decrease in the mass M of the original body. Since dM/dt, the change in mass of the body with time, is negative in this case, in
the limit the quantity M/ t is replaced by -dM/dt. The quantity u-(v+ v) is Vrel, the relative velocity of the ejected mass with respect
to the rocket. With these changes, equation (1.4) can be written as

www.Vidyarthiplus.com
www.Vidyarthiplus.com

The right-hand term depends on the characteristics of the rocket and, like the left-hand term, has the dimensions of a force. This
force is called the thrust, and is the reaction force exerted on the rocket by the mass that leaves it. The rocket designer can make
the thrust as large as possible by designing the rocket to eject mass as rapidly as possible (dM/dt large) and with the highest
possible relative speed (Vrel large). In rocketry, the basic thrust equation is written as

where q is the rate of the ejected mass flow, Ve is the exhaust gas ejection speed, Pe is the pressure of the exhaust gases at the
nozzle exit, Pa is the pressure of the ambient atmosphere, and Ae is the area of the nozzle exit. The product qVe, which we derived
above (Vrel x dM/dt), is called the momentum, or velocity, thrust. The product (Pe-Pa)Ae, called the pressure thrust, is the result
of unbalanced pressure forces at the nozzle exit. As we shall see latter, maximum thrust occurs when Pe=Pa.

PROBLEM 1.1
A spacecraft's engine ejects mass at a rate of 30 kg/s with an exhaust velocity of
3,100 m/s. The pressure at the nozzle exit is 5 kPa and the exit area is 0.7 m2.
What is the thrust of the engine in a vacuum?

SOLUTION,
Given: q = 30 kg/s
Ve = 3,100 m/s
Ae = 0.7 m2
Pe = 5 kPa = 5,000 N/m2
Pa = 0

Equation (1.6),

F = q x Ve + (Pe - Pa) x Ae
F = 30 x 3,100 + (5,000 - 0) x 0.7
F = 96,500 N

Equation (1.6) may be simplified by the definition of an effective exhaust gas velocity, C, defined as

Equation (1.6) then reduces to

Impulse & Momentum


In the preceding section we saw that Newton's second law may be expressed in the form

www.Vidyarthiplus.com
www.Vidyarthiplus.com

Multiplying both sides by dt and integrating from a time t1 to a time t2, we write

The integral is a vector known as the linear impulse, or simply the impulse, of the force F during the time interval considered. The
equation expresses that, when a particle is acted upon by a force F during a given time interval, the final momentum p2 of the
particle may be obtained by adding its initial momentum p1 and the impulse of the force F during the interval of time.

When several forces act on a particle, the impulse of each of the forces must be considered. When a problem involves a system
of particles, we may add vectorially the momenta of all the particles and the impulses of all the forces involved. When can then
write

For a time interval t, we may write equation (1.10) in the form

Let us now see how we can apply the principle of impulse and momentum to rocket mechanics.

Consider a rocket of initial mass M which it launched vertically at time t=0. The fuel is consumed at a constant rate q and is expelled
at a constant speed Ve relative to the rocket. At time t, the mass of the rocket shell and remaining fuel is M-qt, and the velocity is
v. During the time interval t, a mass of fuel q t is expelled. Denoting by u the absolute velocity of the expelled fuel, we apply the
principle of impulse and momentum between time t and time t+ t. Please note, this derivation neglects the effect of air resistance.

We write

We divide through by t and replace u-(v+ v) with Ve, the velocity of the expelled mass relative to the rocket. As t approaches zero, we obtain

www.Vidyarthiplus.com
www.Vidyarthiplus.com

Separating variables and integrating from t=0, v=0 to t=t, v=v, we obtain

which equals

The term -gt in equation (1.15) is the result of Earth's gravity pulling on the rocket. For a rocket drifting in space, -gt is not applicable
and can be omitted. Furthermore, it is more appropriate to express the resulting velocity as a change in velocity, or V. Equation
(1.15) thus becomes

PROBLEM 1.2
The spacecraft in problem 1.1 has an initial mass of 30,000 kg. What is the change in velocity if the spacecraft burns its
engine for one minute?

SOLUTION,
Given: M = 30,000 kg q = 30
kg/s Ve = 3,100 m/s t = 60
s

Equation (1.16),

V= Ve x LN[ M / (M - qt) ]
V= 3,100 x LN[ 30,000 / (30,000 - (30 x 60)) ]
V= 192 m/s

Note that M represents the initial mass of the rocket and M-qt the final mass. Therefore, equation (1.16) is often written as

where mo/mf is called the mass ratio. Equation (1.17) is also known as Tsiolkovsky's rocket equation, named after Russian rocket
pioneer Konstantin E. Tsiolkovsky (1857-1935) who first derived it.

In practical application, the variable Ve is usually replaced by the effective exhaust gas velocity, C. Equation (1.17) therefore becomes

Alternatively, we can write

www.Vidyarthiplus.com
www.Vidyarthiplus.com

where e is a mathematical constant approximately equal to 2.71828.

PROBLEM 1.3
A spacecraft's dry mass is 75,000 kg and the effective exhaust gas velocity of its main engine is 3,100 m/s. How much
propellant must be carried if the propulsion system is to produce a total v of 700 m/s?

SOLUTION,
Given: Mf = 75,000 kg
C = 3,100 m/s V = 700 m/s

Equation (1.20),

Mo = Mf x e^( V / C)
Mo = 75,000 x e^(700 / 3,100)
Mo = 94,000 kg

Propellant mass,

Mp = Mo - Mf
Mp = 94,000 - 75,000 Mp = 19,000 kg

For many spacecraft maneuvers it is necessary to calculate the duration of an engine burn required to achieve a specific change in velocity.
Rearranging variables, we have

www.Vidyarthiplus.com
PROBLEM 1.4
A 5,000 kg spacecraft is in Earth orbit traveling at a velocity of 7,790 m/s. Its engine is burned to accelerate it to a velocity of
12,000 m/s placing it on an escape trajectory. The engine expels mass at a rate of 10 kg/s and an effective velocity of 3,000 m/s.
Calculate the duration of the burn.

SOLUTION,
Given: M = 5,000 kg q = 10 kg/s
C = 3,000 m/s
V = 12,000 - 7,790 = 4,210 m/s

Equation (1.21),
t = M / q x [ 1 - 1 / e^(
The optimum mixture ratio is typically that which will deliver the highest engine performance (measured by specific impulse),
however in some situations a different O/F ratio results in a better overall system. For a volume-constrained vehicle with a low-
density fuel such as liquid hydrogen, significant reductions in vehicle size can be achieved by shifting to a higher O/F ratio. In that
case, the losses in performance are more than compensated for by the reduced fuel tankage requirement. Also consider the
example of bipropellant systems using NTO/MMH, where a mixture ratio of 1.67 results in fuel and oxidizer tanks of equal size.
Equal sizing simplifies tank manufacturing, system packaging, and integration.

As we have seen previously, impulse thrust is equal to the product of the propellant mass flow rate and the exhaust gas ejection
speed. The ideal exhaust velocity is given by

where k is the specific heat ratio, R' is the universal gas constant (8,314.51 N-m/kg mol-K in SI units, or 49,720 ft-lb/slug mol-oR in
U.S. units), Tc is the combustion temperature, M is the average molecular weight of the exhaust gases, Pc is the combustion
chamber pressure, and Pe is the pressure at the nozzle exit.

Specific heat ratio(2) varies depending on the composition and temperature of the exhaust gases, but it is usually about 1.2. The
thermodynamics involved in calculating combustion temperatures are quite complicated, however, flame temperatures generally
range from about 2,500 to 3,600 oC (4,500-6,500 oF). Chamber pressures can range from about about 7 to 250 atmospheres. Pe
should be equal to the ambient pressure at which the engine will operate, more on this later. See below the charts providing
optimum mixture ratio, adiabatic flame temperature, gas molecular weight, and specific heat ratio for some common rocket
propellants.

From equation (1.22) we see that high chamber temperature and pressure, and low exhaust gas molecular weight results in high
ejection velocity, thus high thrust. Based on this criterion, we can see why liquid hydrogen is very desirable as a rocket fuel.

Liquid Oxygen & Liquid Hydrogen


Optimum Mixture Ratio
www.Vidyarthiplus.com

Adiabatic Flame Temperature

www.Vidyarthiplus.com
www.Vidyarthiplus.com

Gas Molecular Weight

Specific Heat Ratio

www.Vidyarthiplus.com
www.Vidyarthiplus.com

• Molecular weight equals the sum of the atomic weights of the atoms in the molecule. For NaCl, the
atomic weight of sodium is 23, of chlorine is 35 and a molecule contains one sodium and one chlorine,
so 23 + 35 = 58, the molecular weight of NaCl. The formula for glucose ( a very common sugar) is
C6H12O6. The subscripts to the right mean that it contains 6 atoms of carbon, 12 atoms of hydrogen,
and 6 atoms of oxygen. The atomic weight for carbon is 12, for hydrogen is 1, and for oxygen is 16,
so the molecular weight of glucose can be calculated thus:
----------------------------------------------------------
Element Atomic
Weight No. of Atoms Total Weight
-----------------------------------------------------------
C 12 6 6 × 12 = 72
H 1 12 12 × 1 = 12 O 16 6 6 × 16 = 96
--------------------------------------------------------------- Total Molecular Weight = 180

PROPELLANT COMBUSTION CHARTS


How To Use These Charts
For each of the propellant combinations shown above, four graphs have been provided. These graphs can be used to estimate (1)
the optimum mixture ratio of the combustion reactants, (2) the adiabatic flame temperature of the combustion reaction, (3) the
average molecular weight of the combustion products, and (4) the specific heat ratio of the combustion products. This data is
necessary to determine the velocity of the exhaust gases expelled from a rocket engine, which in turn determines the engine's
thrust. Adiabatic flame temperature and gas molecular weight have been calculated using the freeware program STANJAN.

Optimum Mixture Ratio


Mixture Ratio is the ratio of oxidizer mass t

o fuel mass. We define the optimum mixture ratio as that which will produce the highest specific impulse for the given reactants. A
propellant's optimum mixture ratio is a function of the pressures at which the rocket engine will operate. An engine with a high
combustion chamber pressure and a low nozzle exit pressure, i.e. a large section ratio, will have the highest optimum mixture ratio.

Below we see a graph of optimum mixture ratio versus combustion chamber pressure for liquid oxygen and kerosene at two
different nozzle exit pressures (Pe). To use this graph, select the desired chamber pressure across the bottom axis of the graph
and draw a vertical line. When the vertical line intersects the curve for the desired exit pressure, draw a horizontal line to the left
and read the corresponding mixture ratio off the vertical axis of the graph. If an exit pressure other than those shown is desired,
estimate the position of the exit pressure curve by interpolating between those given. For instance, the curve for a Pe of 0.7
atmosphere lies approximately one-third the distance from the Pe = 1.0 curve to the Pe = 0.1 curve. In the given example we've

www.Vidyarthiplus.com
www.Vidyarthiplus.com

selected a combustion chamber pressure of 75 atmospheres and a nozzle exit pressure of 1 atmosphere, which gives us an
optimum mixture ratio of 2.30.

www.Vidyarthiplus.com
www.Vidyarthiplus.com

Gas Molecular Weight


The exhaust gas molecular weight is the average molar weight of the combustion products, that is, the mass of the exhaust gas
divided by the number of moles.

Below is a graph of gas molecular weight versus combustion chamber pressure for liquid oxygen and kerosene at three different
mixture ratios. To use this graph, select the desired pressure across the bottom axis of the graph and draw a vertical line. When
the vertical line intersects the curve for the desired mixture ratio, draw a horizontal line to the left and read the corresponding gas
molecular weight off the vertical axis of the graph. For mixture ratios other than those shown, estimate by interpolating between the
given curves. In the provided example we've selected a pressure of 75 atmospheres and a mixture ratio of 2.30, which gives us an
average gas molecular weight of about 21.65.

The gas molecular weights shown below are taken at the combustion chamber. The molecular weight will increase slightly as the
gas expands and cools while moving toward the nozzle exit.

Specific Heat Ratio

www.Vidyarthiplus.com
www.Vidyarthiplus.com

www.Vidyarthiplus.com

Liquid Oxygen & Liquid Methane

Optimum Mixture Ratio

Adiabatic Flame Temperature


www.Vidyarthiplus.com

www.Vidyarthiplus.com
www.Vidyarthiplus.com

Gas Molecular Weight

Specific Heat Ratio

www.Vidyarthiplus.com
www.Vidyarthiplus.com

Liquid Oxygen & Kerosene*


* n-Dodecane, C12H26

Optimum Mixture Ratio

Adiabatic Flame Temperature

www.Vidyarthiplus.com
www.Vidyarthiplus.com

Gas Molecular Weight

Specific Heat Ratio

www.Vidyarthiplus.com
www.Vidyarthiplus.com

where F is thrust, q is the rate of mass flow, and g is the acceleration of gravity at ground level.

Specific impulse is expressed in seconds. When the thrust and the flow rate remain constant throughout the burning of the
propellant, the specific impulse is the time for which the rocket engine provides a thrust equal to the weight of the propellant
consumed.

For a given engine, the specific impulse has different values on the ground and in the vacuum of space because the ambient
pressure is involved in the expression for the thrust. It is therefore important to state whether specific impulse is the value at sea
level or in a vacuum.

There are a number of losses within a rocket engine, the main ones being related to the inefficiency of the chemical reaction
(combustion) process, losses due to the nozzle, and losses due to the pumps. Overall, the losses affect the efficiency of the specific
impulse. This is the ratio of the real specific impulse (at sea level, or in a vacuum) and the theoretical specific impulse obtained
with an ideal nozzle from gases coming from a complete chemical reaction. Calculated values of specific impulse are several
percent higher than those attained in practice.

PROBLEM 1.6
A rocket engine produces a thrust of 1,000 kN at sea level with a propellant flow rate of 400 kg/s. Calculate the specific
impulse.

SOLUTION,
Given: F = 1,000,000 N q = 400 kg/s

Equation (1.23),

Isp = F / (q x g)
Isp = 1,000,000 / (400 x 9.80665)
Isp = 255 s (sea level)

From Equation (1.8) we can substitute qC for F in Equation (1.23), thus obtaining

Equation (1.24) is very useful when solving Equations (1.18) through (1.21). It is rare we are given the value of C directly, however
rocket engine specific impulse is a commonly given parameter from which we can easily calculate C.

Engines & Nozzles


A typical rocket motor consists of the combustion chamber, the nozzle, and the injector, as shown in the figure below. The
combustion chamber is where the burning of propellants takes place at high pressure. The chamber must be strong enough to
contain the high pressure generated by, and the high temperature resulting from, the combustion process. Because of the high

www.Vidyarthiplus.com
temperature and heat transfer, the chamber and nozzle are usually cooled. The chamber must also be of sufficient length to ensure
complete combustion before the gases enter the nozzle.
www.Vidyarthiplus.com

The figure above-right shows three different exhaust nozzles. The most efficient nozzle (1) is contoured to the exhaust stream,
allowing the escaping gas to expand just enough to fill the nozzle. A nozzle that lets the gas expand too much (2), or too little (3),
wastes the energy and thrust potential of the exhaust system.

The nozzle throat area, At, can be found if the total propellant flow rate is known and the propellants and operating conditions have
been selected. Assuming perfect gas law theory, we have

where q is the propellant mass flow rate, Pt is the gas pressure at the nozzle throat, Tt is the gas temperature at the nozzle throat,
R' is the universal gas constant, and k is the specific heat ratio. Pt and Tt are given by

where Pc is the combustion chamber pressure and Tc is the combustion chamber flame temperature.

PROBLEM 1.7
A rocket engine uses the same propellant, mixture ratio, and combustion chamber pressure as that in problem 1.5. If the
propellant flow rate is 500 kg/s, calculate the area of the exhaust nozzle throat.

SOLUTION,
Given: Pc = 50 x 0.101325 = 5.066 MPa
Tc = 3,470<SUP.O< sup> K M = 21.40
k = 1.221 q = 500 kg/s

Equation (1.26),

Pt = Pc x [1 + (k - 1) / 2]-k/(k-1)
Pt = 5.066 x [1 + (1.221 - 1) / 2]-1.221/(1.221-1)
Pt = 2.839 MPa = 2.839x106 N/m2

Equation (1.27),

Tt = Tc / (1 + (k - 1) / 2)
Tt = 3,470 / (1 + (1.221 - 1) / 2)
Tt = 3,125 K

www.Vidyarthiplus.com
Equation (1.25),

At = (q / Pt) x SQRT[ (R' x Tt) / (M x k) ]


At = (500 / 2.839x106) x SQRT[ (8,314.51 x 3,125) / (21.40 x 1.221) ] At = 0.1756 m2

The hot gases must be expanded in the diverging section of the nozzle to obtain maximum thrust. The pressure of these gases will
decrease as energy is used to accelerate the gas. We must find that area of the nozzle where the gas pressure is equal to the
outside atmospheric pressure. This area will then be the nozzle exit area.

Mach number Nm is the ratio of the gas velocity to the local speed of sound. The Mach number at the nozzle exit is given by the
perfect gas expansion expression
www.Vidyarthiplus.com

Equation (1.28),

Nm2 = (2 / (k - 1)) x [(Pc / Pa)(k-1)/k - 1]


Nm2 = (2 / (1.221 - 1)) x [(5.066 / 0.0795)(1.221-1)/1.221 - 1]
Nm2 = 10.15
Nm = (10.15)1/2 = 3.185

Equation (1.29),

Ae = (At / Nm) x [(1 + (k - 1) / 2 x Nm2)/((k + 1) / 2)](k+1)/(2(k-1))


Ae = (0.1756 / 3.185) x [(1 + (1.221 - 1) / 2 x 10.15)/((1.221 + 1) /
2)](1.221+1)/(2(1.221-1))
Ae = 1.426 m2

Section Ratio,

Ae / At = 1.426 / 0.1756 = 8.12

For launch vehicles (particularly first stages) where the ambient pressure varies during the burn period, trajectory computations
are performed to determine the optimum exit pressure. However, an additional constraint is the maximum allowable diameter for
the nozzle exit cone, which in some cases is the limiting constraint. This is especially true on stages other than the first, where the
nozzle diameter may not be larger than the outer diameter of the stage below. For space engines, where the ambient pressure is
zero, thrust always increases as nozzle expansion ratio increases. On these engines, the nozzle expansion ratio is generally
increased until the additional weight of the longer nozzle costs more performance than the extra thrust it generates.

Rocket Nozzle Design: Optimizing Expansion for


Maximum Thrust
A rocket engine is a device in which propellants are burned in a combustion chamber and the resulting high pressure gases are
expanded through a specially shaped nozzle to produce thrust. The function of the nozzle is to convert the chemical-thermal energy
generated in the combustion chamber into kinetic energy. The nozzle converts the slow moving, high pressure, high temperature
gas in the combustion chamber into high velocity gas of lower pressure and temperature. Gas velocities from 2 to 4.5 kilometers
per second can be obtained in rocket nozzles. The nozzles which perform this feat are called DeLaval nozzles (after the inventor)
and consist of a convergent and divergent section. The minimum flow area between the convergent and divergent section is called
the nozzle throat. The flow area at the end of the divergent section is called the nozzle exit area.

Hot exhaust gases expand in the diverging section of the nozzle. The pressure of these gases will decrease as energy is used to
accelerate the gas to high velocity. The nozzle is usually made long enough (or the exit area great enough) such that the pressure
in the combustion chamber is reduced at the nozzle exit to the pressure existing outside the nozzle. It is under this condition that
thrust is maximum and the nozzle is said to be adapted, also called optimum or correct expansion. To understand this we must
examine the basic thrust equation:

F = q x Ve + (Pe - Pa) x Ae

where F = Thrust
q = Propellant mass flow rate Ve = Velocity of exhaust
gases

www.Vidyarthiplus.com
Pe = Pressure at nozzle exit Pa = Ambient pressure
Ae = Area of nozzle exit
The product qVe is called the momentum, or velocity, thrust and the product (Pe-Pa)Ae is called the pressure thrust. As we have
seen, Ve and Pe are inversely proportional, that is, as one increases the other decreases. If a nozzle is under-extended we have
Pe>Pa and Ve is small. For an over-extended nozzle we have Pe<Pa and Ve is large. Thus, momentum thrust and pressure thrust
are inversely proportional and, as we shall see, maximum thrust occurs when Pe=Pa.

Let us now consider an example. Assume we have a rocket engine equipped with an extendible nozzle. The engine is test fired in
an environment with a constant ambient pressure. During the burn, the nozzle is extended from its fully retracted position to its fully
extended position. At some point between fully retracted and fully extended Pe=Pa (see figure below).
www.Vidyarthiplus.com

The gas pressure and temperature at the nozzle throat is less than in the combustion chamber due to the loss of thermal energy
in accelerating the gas to the local speed of sound at the throat. Therefore, we calculate the pressure and temperature at the nozzle
throat,

Pt = Pc x [1 + (k - 1) / 2]-k/(k-1)
Pt = 5 x [1 + (1.20 - 1) / 2]-1.20/(1.20-1)
Pt = 2.82 MPa = 2.82x106 N/m2

Tt = Tc x [1 / (1 + (k - 1) / 2)]
Tt = 3,600 x [1 / (1 + (1.20 - 1) / 2)] Tt = 3,273 K

The area at the nozzle throat is given by

At = (q / Pt) x SQRT[ (R' x Tt) / (M x k) ]


At = (100 / 2.82x106) x SQRT[ (8,314 x 3,273) / (24 x 1.20) ] At = 0.0345 m2

The hot gases must now be expanded in the diverging section of the nozzle to obtain maximum thrust. The Mach number at the
nozzle exit is given by

Nm2 = (2 / (k - 1)) x [(Pc / Pa)(k-1)/k - 1]


Nm2 = (2 / (1.20 - 1)) x [(5 / 0.05)(1.20-1)/1.20 - 1]
Nm2 = 11.54
Nm = (11.54)1/2 = 3.40

The nozzle exit area corresponding to the exit Mach number is given by

Ae = (At / Nm) x [(1 + (k - 1) / 2 x Nm2)/((k + 1) / 2)](k+1)/(2(k-1))


Ae = (0.0345 / 3.40) x [(1 + (1.20 - 1) / 2 x 11.54)/((1.20 + 1) /
2)](1.20+1)/(2(1.20-1))
Ae = 0.409 m2

The velocity of the exhaust gases at the nozzle exit is given by

Ve = SQRT[ (2 x k / (k - 1)) x (R' x Tc / M) x (1 - (Pe / Pc)(k-1)/k) ]


Ve = SQRT[ (2 x 1.20 / (1.20 - 1)) x (8,314 x 3,600 / 24) x (1 - (0.05 /
5)(1.20-1)/1.20) ]
Ve = 2,832 m/s

Finally, we calculate the thrust,

F = q x Ve + (Pe - Pa) x Ae
F = 100 x 2,832 + (0.05x106 - 0.05x106) x 0.409
F = 283,200 N

Let's now consider what happens when the nozzle is under-extended, that is Pe>Pa. If we assume Pe=Pa x 2, we have

Pe = 0.05 x 2 = 0.10 MPa

At = 0.0345 m2

www.Vidyarthiplus.com
Nm2 = (2 / (1.20 - 1)) x [(5 / 0.10)(1.20-1)/1.20 - 1]
www.Vidyarthiplus.com

Nm2 = 9.19
Nm = (9.19)1/2 = 3.03

Ae = (0.0345 / 3.03) x [(1 + (1.20 - 1) / 2 x 9.19)/((1.20 + 1) /


2)](1.20+1)/(2(1.20-1))
Ae = 0.243 m2
Ve = SQRT[ (2 x 1.20 / (1.20 - 1)) x (8,314 x 3,600 / 24) x (1 - (0.10 /
5)(1.20-1)/1.20) ]
Ve = 2,677 m/s

F = 100 x 2,677 + (0.10x106 - 0.05x106) x 0.243


F = 279,850 N

," . D@

Pe = 0.05 / 2 = 0.025 MPa

At = 0.0345 m2

Nm2 = (2 / (1.20 - 1)) x [(5 / 0.025)(1.20-1)/1.20 - 1] Nm2 = 14.18


Nm = (14.18)1/2 = 3.77

Ae = (0.0345 / 3.77) x [(1 + (1.20 - 1) / 2 x 14.18)/((1.20 + 1) /


2)](1.20+1)/(2(1.20-1))
Ae = 0.696 m2

Ve = SQRT[ (2 x 1.20 / (1.20 - 1)) x (8,314 x 3,600 / 24) x (1 - (0.025 /


5)(1.20-1)/1.20) ]
Ve = 2,963 m/s

F = 100 x 2,963 + (0.025x106 - 0.05x106) x 0.696


F = 278,900 N

) ," ,"
)
H

www.Vidyarthiplus.com
As can be easily seen, thrust is maximum when Pa/Pe=1, or when Pe=Pa.

Power Cycles
Liquid bipropellant rocket engines can be categorized according to their power cycles, that is, how power is derived to feed
propellants to the main combustion chamber. Described below are some of the more common types.

Gas-generator cycle: The gas-generator cycle, also called open cycle, taps off a small amount of fuel and oxidizer from the main
flow (typically 3 to 7 percent) to feed a burner called a gas generator. The hot gas from this generator passes through a turbine to
generate power for the pumps that send propellants to the combustion chamber. The hot gas is then either dumped overboard or
sent into the main nozzle downstream. Increasing the flow of propellants into the gas generator increases the speed of the turbine,
which increases the flow of propellants into the main combustion chamber, and hence, the amount of thrust produced. The gas
generator must burn propellants at a less-than-optimal mixture ratio to keep the temperature low for the turbine blades. Thus, the
cycle is appropriate for moderate power requirements but not high-power systems, which would have to divert a large portion of
the main flow to the less efficient gas-generator flow.

As in most rocket engines, some of the propellant in a gas generator cycle is used to cool the nozzle and combustion chamber,
increasing efficiency and allowing higher engine temperature.
www.Vidyarthiplus.com

www.Vidyarthiplus.com
can achieve higher chamber pressures than the closed expander cycle although at lower efficiency because of the overboard flow.
Regenerative cooling is the most widely used method of cooling a thrust
chamber and is accomplished by flowing high-velocity coolant over the back
side of the chamber hot gas wall to convectively cool the hot gas liner. The
coolant with the heat input from cooling the liner is then discharged into the
injector and utilized as a propellant.

Earlier thrust chamber designs, such as the V-2 and Redstone, had low
chamber pressure, low heat flux and low coolant pressure requirements,
which could be satisfied by a simplified "double wall chamber" design with
regenerative and film cooling. For subsequent
rocket engine applications, however, chamber pressures were
increased and the cooling requirements became more difficult to satisfy. It
became necessary to design new coolant configurations that were more
efficient structurally and had improved heat transfer characteristics.

This led to the design of "tubular wall" thrust chambers, by far the most widely
used design approach for the vast majority of large rocket engine
applications. These chamber designs have been successfully used for the
Thor, Jupiter, Atlas, H-1, J-2, F-1, RS-27 and several other Air Force and NASA rocket engine applications. The primary advantage
of the design is its light weight and the large experience base that has accrued. But as chamber pressures and hot gas wall heat
fluxes have continued to increase (>100 atm), still more effective methods have been needed.

One solution has been "channel wall" thrust chambers, so named because the hot gas wall cooling is accomplished by flowing
coolant through rectangular channels, which are machined or formed into a hot gas liner fabricated from a high-conductivity
material, such as copper or a copper alloy. A prime example of a channel wall combustion chamber is the SSME, which operates
at 204 atmospheres nominal chamber pressure at 3,600 K for a duration of 520 seconds. Heat transfer and structural characteristics
are excellent.

In addition to the regeneratively cooled designs mentioned above, other thrust chamber designs have been fabricated for rocket
engines using dump cooling, film cooling, transpiration cooling, ablative liners and radiation cooling. Although regeneratively cooled
combustion chambers have proven to be the best approach for cooling large liquid rocket engines, other methods of cooling have
also been successfully used for cooling thrust chamber assemblies. Examples include:

Dump cooling, which is similar to regenerative cooling because the coolant flows through small passages over the back side of the
thrust chamber wall. The difference, however, is that after cooling the thrust chamber, the coolant is discharged overboard through
openings at the aft end of the divergent nozzle. This method has limited application because of the performance loss resulting from
dumping the coolant overboard. To date, dump cooling has not been used in an actual application.

www.Vidyarthiplus.com

Film cooling provides protection from excessive heat by introducing a thin film of coolant or propellant through orifices around the
injector periphery or through manifolded orifices in the chamber wall near the injector or chamber throat region. This method is
typically used in high heat flux regions and in combination with regenerative cooling.

Transpiration cooling provides coolant (either gaseous or liquid propellant) through a porous chamber wall at a rate sufficient to
maintain the chamber hot gas wall to the desired temperature. The technique is really a special case of film cooling.

With ablative cooling, combustion gas-side wall material is sacrificed by melting, vaporization and chemical changes to dissipate
heat. As a result, relatively cool gases flow over the wall surface, thus lowering the boundary-layer temperature and assisting the
cooling process.
With radiation cooling, heat is radiated from the outer surface of the combustion chamber or nozzle extension wall. Radiation
cooling is typically used for small thrust chambers with a high-temperature wall material (refractory) and in low-heat flux regions,
such as a nozzle extension.

Solid Rocket Motors


Solid rockets motors store propellants in solid form. The fuel is typically powdered aluminum and the oxidizer is ammonium
perchlorate. A synthetic rubber binder such as polybutadiene holds the fuel and oxidizer powders together. Though lower
performing than liquid propellant rockets, the operational simplicity of a solid rocket motor often makes it the propulsion system of
choice.

Solid Fuel Geometry

A solid fuel's geometry determines the area and contours of its exposed surfaces, and thus its burn pattern. There are two main
types of solid fuel blocks used in the space industry. These are cylindrical blocks, with combustion at a front, or surface, and
cylindrical blocks with internal combustion. In the first case, the front of the flame travels in layers from the nozzle end of the block
towards the top of the casing. This so-called end burner produces constant thrust throughout the burn. In the second, more usual
case, the combustion surface develops along the length of a central channel. Sometimes the channel has a star shaped, or other,
geometry to moderate the growth of this surface.

The shape of the fuel block for a rocket is chosen for the particular type of mission it will perform. Since the combustion of the block
progresses from its free surface, as this surface grows, geometrical considerations determine whether the thrust increases,
decreases or stays constant.

www.Vidyarthiplus.com
www.Vidyarthiplus.com

www.Vidyarthiplus.com
a grain L/D ratio of 6. A greater Aport/At ratio should be used for grains with larger L/D ratios.

• In an operating rocket motor, there is a pressure drop along the axis of the combustion chamber, a drop that is physically
necessary to accelerate the increasing mass flow of combustion products toward the nozzle. The static pressure is
greatest where gas flow is zero, that is, at the front of the motor. Since burn rate is dependant upon the local pressure,
the rate should be greatest at this location. However, this effect is relatively minor and is usually offset by the counter-
effect of erosive burning.
• Burning rate is enhanced by acceleration of the motor. Whether the acceleration is a result of longitudinal force (e.g.
thrust) or spin, burning surfaces that form an angle of about 60-90o with the acceleration vector are prone to increased
burn rate.

It is sometimes desirable to modify the burning rate such that it is more suitable to a certain grain configuration. For example, if
one wished to design an end burner grain, which has a relatively small burning area, it is necessary to have a fast burning propellant.
In other circumstances, a reduced burning rate may be sought after. For example, a motor may have a large L/D ratio to generate
sufficiently high thrust, or it may be necessary for a particular design to restrict the diameter of the motor. The web would be
consequently thin, resulting in short burn duration. Reducing the burning rate would be beneficial.

There are a number of ways of modifying the burning rate: decrease the oxidizer particle size, increase or reduce the percentage
of oxidizer, adding a burn rate catalyst or suppressant, and operate the motor at a lower or higher chamber pressure. These factors
are discussed below.

• The effect of the oxidizer particle size on burn rate seems to be influenced by the type of oxidizer. Propellants that use
ammonium perchlorate (AP) as the oxidizer have a burn rate that is significantly affected by AP particle size. This most
likely results from the decomposition of AP being the rate-determining step in the combustion process.
• The burn rate of most propellants is strongly influenced by the oxidizer/fuel ratio. Unfortunately, modifying the burn rate
by this means is quite restrictive, as the performance of the propellant, as well as mechanical properties, are also greatly
affected by the O/F ratio.
• Certainly the best and most effective means of increasing the burn rate is the addition of a catalyst to the propellant
mixture. A catalyst is a chemical compound that is added in small quantities for the sole purpose of tailoring the burning
rate. A burn rate suppressant is an additive that has the opposite effect to that of a catalyst -- it is used to decrease the
burn rate.
• For a propellant that follows the Saint-Robert's burn rate law, designing a rocket motor to operate at a lower chamber
pressure will provide for a lower burning rate. Due to the nonlinearity of the pressure-burn rate relationship, it may be
necessary to significantly reduce the operating pressure to get the desired burning rate. The obvious drawback is reduced
motor performance, as specific impulse similarly decays with reducing chamber pressure.

Product Generation Rate

The rate at which combustion products are generated is expressed in terms of the regression speed of the grain. The product
generation rate integrated over the port surface area is

where q is the combustion product generation rate at the propellant surface, p is the solid propellant density, Ab is the area of the
burning surface, and r is the propellant burn rate.

It is important to note that the combustion products may consist of both gaseous and condensed-phase mass. The condensed-
phase, which manifests itself as smoke, may be either solid or liquid particles. Only the gaseous products contribute to pressure
development. The condensed-phase certainly does, however, contribute to the thrust of the rocket motor, due to its mass and
velocity.
www.Vidyarthiplus.com
PROBLEM 1.9
A solid rocket motor burns along the face of a central cylindrical channel 10 meters long and 1 meter in diameter. The propellant
has a burn rate coefficient of 5.5, a pressure exponent of 0.4, and a density of 1.77 g/ml. Calculate the burn rate and the
product generation rate when the chamber pressure is 5.0 MPa.

SOLUTION,
Given: a = 5.5 n = 0.4
Pc = 5.0 MPa p = 1.77 g/ml
Ab = x 1 x 10 = 31.416 m2

Equation (1.30),

r = a x Pcn
r = 5.5 x 5.00.4 = 10.47 mm/s

Equation (1.31),
q = with
associated burn
rate variation.
Other factors
may play a role,
however, such
as nozzle throat
erosion and
erosive burn rate
augmentation.

Monopropellant Engines
By far the most widely used type of propulsion for spacecraft attitude and velocity control is monopropellant hydrazine. Its excellent
handling characteristics, relative stability under normal storage conditions, and clean decomposition products have made it the
standard. The general sequence of operations in a hydrazine thruster is:

• When the attitude control system signals for thruster operation, an electric solenoid valve opens allowing hydrazine to
flow. The action may be pulsed (as short as 5 ms) or long duration (steady state).
• The pressure in the propellant tank forces liquid hydrazine into the injector. It enters as a spray into the thrust chamber
and contacts the catalyst beds.
• The catalyst bed consists of alumina pellets impregnated with iridium. Incoming hydrazine heats to its vaporizing point
by contact with the catalyst bed and with the hot gases leaving the catalyst particles. The temperature of the hydrazine
rises to a point where the rate of its decomposition becomes so high that the chemical reactions are self-sustaining.
• By controlling the flow variables and the geometry of the catalyst chamber, a designer can tailor the proportion of
chemical products, the exhaust temperature, the molecular weight, and thus the enthalpy for a given application. For a
thruster application where specific impulse is paramount, the designer attempts to provide 30-40% ammonia dissociation,
which is about the lowest percentage that can be maintained reliably. For gas-generator application, where lower
temperature gases are usually desired, the designer provides for higher levels of ammonia dissociation.
• Finally, in a space thruster, the hydrazine decomposition products leave the catalyst bed and exit from the chamber
through a high expansion ratio exhaust nozzle to produce thrust.

www.Vidyarthiplus.com
Monopropellant hydrazine thrusters typically produce a specific impulse of about 230 to 240 seconds.

Other suitable propellants for catalytic decomposition engines are hydrogen peroxide and nitrous oxide, however the performance
is considerably lower than that obtained with hydrazine - specific impulse of about 150 s with H2O2 and about 170 s with N2O.

Monopropellant systems have successfully provided orbit maintenance and attitude control functions, but lack the performance to
provide weight-efficient large V maneuvers required for orbit insertion. Bipropellant systems are attractive because they can
provide all three functions with one higher performance system, but they are more complex than the common solid rocket and
monopropellant combined systems. A third alternative are dual mode systems. These systems are hybrid designs that use
hydrazine both as a fuel for high performance bipropellant engines and as a monopropellant with conventional low-thrust catalytic
thrusters. The hydrazine is fed to both the bipropellant engines and the monopropellant thrusters from a common fuel tank.

Cold gas propulsion is just a controlled, pressurized gas source and a nozzle. It represents the simplest form of rocket engine. Cold
gas has many applications where simplicity and/or the need to avoid hot gases are more important than high performance. The
Manned Maneuvering Unit used by astronauts is an example of such a system.

Staging
Multistage rockets allow improved payload capability for vehicles with a high V requirement such as launch vehicles or
interplanetary spacecraft. In a multistage rocket, propellant is stored in smaller, separate tanks rather than a larger single tank as
in a singlestage rocket. Since each tank is discarded when empty, energy is not expended to accelerate the empty tanks, so a
higher total V is obtained. Alternatively, a larger payload mass can be accelerated to the same total V. For convenience, the
separate tanks are usually bundled with their own engines, with each discardable unit called a stage.

Multistage rocket performance is described by the same rocket equation as single-stage rockets, but must be determined on a
stage-by-stage basis. The velocity increment, Vi, for each stage is calculated as before,

where moi represents the total vehicle mass when stage i is ignited, and mfi is the total vehicle mass when stage i is burned out but
not yet discarded. It is important to realize that the payload mass for any stage consists of the mass of all subsequent stages plus
the ultimate payload itself. The velocity increment for the vehicle is then the sum of those for the individual stages where n is the
total number of stages.

PROBLEM 1.10
A two-stage rocket has the following masses: 1st-stage propellant mass 120,000 kg,
1st-stage dry mass 9,000 kg, 2nd-stage propellant mass 30,000 kg, 2nd-stage dry mass
3,000 kg, and payload mass 3,000 kg. The specific impulses of the 1st and
2nd stages are 260 s and 320 s respectively. Calculate the rocket's total C1 = Isp1g
C1 = 260 x 9.80665 = 2,550 m/s

C2 = Isp2g
C2 = 320 x 9.80665 = 3,138 m/s

Equation (1.33),
www.Vidyarthiplus.com
V1 = C1 x LN[ Mo1 / Mf1 ]
V1 = 2,550 x LN[ 165,000 / 45,000 ]
V1 = 3,313 m/s

V2 = C2 x LN[ Mo2 / Mf2 ]


V2 = 3,138 x LN[ 36,000 / 6,000 ]
V2 = 5,623 m/s

Equation (1.34),

VTotal = V1 + V2
VTotal = 3,313 + 5,623
VTotal = 8,936 m/s

We define the payload fraction as the ratio of payload mass to initial mass, or mpl/mo.

For a multistage vehicle with dissimilar stages, the overall vehicle payload fraction depends on how the V requirement is
partitioned among stages. Payload fractions will be reduced if the V is partitioned suboptimally. The optimal distribution may be
determined by trial and error. A V distribution is postulated and the resulting payload fraction calculated. The V distribution is
varied until the payload fraction is maximized. Once the V distribution is selected, vehicle sizing is accomplished by starting
with the uppermost or final stage (whose payload is the actual deliverable payload) and calculating the initial mass of this
assembly. This assembly then forms the payload for the previous stage and the process repeats until all stages are sized. Results
reveal that to maximize payload fraction for a given V requirement:

1. Stages with higher Isp should be above stages with lower Isp. 2. More V should be provided by the stages with the higher Isp.
3. Each succeeding stage should be smaller than its predecessor.
4. Similar stages should provide the same V.

www.Vidyarthiplus.com

Solid Rocket Components


The key inert components of solid propellant rocket motors are the motor case, nozzle, and igniter case.
Thrust vector control (TVC) mechanism also a component of many rocket motors.

Motor Case:

The case not only contains the propellant grain, but also serves as a highly loaded pressure vessel. Case
design is usually governed by a combination of motor and vehicle requirements. Besides constituting the

www.Vidyarthiplus.com
structural body of the rocket motor with its nozzle, propellant grain, and so on, the case frequently serves
also as the primary structure of the missile or launch vehicle.

Different types of loads and their sources must be considered at the beginning of a case design. In addition,
the environmental conditions peculiar to a specific motor and its usage must be carefully considered.
Typically, these conditions include the following; (1) temperature (internal heating, temperature cycling during
storage, or thermal stress and strains); (2) corrosion (moisture/chemical, galvanic, stress corrosion etc.); (3)
space conditions: vacuum or radiation.

Three classes of materials have been used: high-strength metals (such as steel, aluminum, or titanium
alloys), wound-filament reinforced plastics, and a combination of these.

Rocket Motor Case Loads

(Ref: G.P.Sutton)
Origin of Load Type of Load/Stress
Internal pressure Tension biaxial, vibration
Axial thrust Axial, vibration
Motor nozzle Axial, bending, shear
Thrust vector control actuators Axial, bending, shear
Thrust termination equipment Biaxial, bending
Aerodynamic control surface or wings Tension, compression, bending, shear, torsion
mounted to case
Staging Bending, shear
Flight maneuvering Axial, bending, shear, torsion
Vehicle mass and wind forces on launch pad Axial, bending, shear
Dynamic loads from vehicle oscillations Axial, bending, shear
Ground transport, Ground handling, including lifting Tension, compression, bending, shear, torsion,
vibration
Earthquakes (large motors) Axial, bending, shear

www.Vidyarthiplus.com
www.Vidyarthiplus.com

ROCKET PROPELLANTS
Propellant is the chemical mixture burned to produce thrust in rockets and consists of a fuel and an oxidizer. A fuel is a
substance which burns when combined with oxygen producing gas for propulsion. An oxidizer is an agent that releases oxygen
for combination with a fuel. The ratio of oxidizer to fuel is called the mixture ratio. Propellants are classified according to their
state - liquid, solid, or hybrid.

The gauge for rating the efficiency of rocket propellants is specific impulse, stated in seconds. Specific impulse indicates how many
pounds (or kilograms) of thrust are obtained by the consumption of one pound (or kilogram) of propellant in one second. Specific
impulse is characteristic of the type of propellant, however, its exact value will vary to some extent with the operating conditions
and design of the rocket engine.

Liquid Propellants
In a liquid propellant rocket, the fuel and oxidizer are stored in separate tanks, and are fed through a system of pipes, valves, and
turbopumps to a combustion chamber where they are combined and burned to produce thrust. Liquid propellant engines are more
complex than their solid propellant counterparts, however, they offer several advantages. By controlling the flow of propellant to
the combustion chamber, the engine can be throttled, stopped, or restarted.

A good liquid propellant is one with a high specific impulse or, stated another way, one with a high speed of exhaust gas ejection.
This implies a high combustion temperature and exhaust gases with small molecular weights. However, there is another important
factor which must be taken into consideration: the density of the propellant. Using low density propellants means that larger storage
tanks will be required, thus increasing the mass of the launch vehicle. Storage temperature is also important. A propellant with a
low storage temperature, i.e. a cryogenic, will require thermal insulation, thus further increasing the mass of the launcher. The
toxicity of the propellant is likewise important. Safety hazards exist when handling, transporting, and storing highly toxic
compounds. Also, some propellants are very corrosive, however, materials that are resistant to certain propellants have been
identified for use in rocket construction.

Liquid propellants used in rocketry can be classified into three types: petroleum, cryogens, and hypergols.

Petroleum fuels are those refined from crude oil and are a mixture of complex hydrocarbons, i.e. organic compounds containing
only carbon and hydrogen. The petroleum used as rocket fuel is a type of highly refined kerosene, called RP-1 in the United States.
Petroleum fuels are usually used in combination with liquid oxygen as the oxidizer. Kerosene delivers a specific impulse
considerably less than cryogenic fuels, but it is generally better than hypergolic propellants.

Specifications for RP-1 where first issued in the United States in 1957 when the need for a clean burning petroleum rocket fuel was
recognized. Prior experimentation with jet fuels produced tarry residue in the engine cooling passages and excessive soot, coke
and other deposits in the gas generator. Even with the new specifications, kerosene-burning engines still produce enough residues
that their operational lifetimes are limited.

www.Vidyarthiplus.com
www.Vidyarthiplus.com

Liquid oxygen and RP-1 are used as the propellant in the first-stage boosters of the Atlas and Delta II launch vehicles. It also
powered the first-stages of the Saturn 1B and Saturn V rockets.

Cryogenic propellants are liquefied gases stored at very low temperatures, most frequently liquid hydrogen (LH 2) as the fuel and
liquid oxygen (LO2 or LOX) as the oxidizer. Hydrogen remains liquid at temperatures of -253 oC (-423 oF) and oxygen remains in a
liquid state at temperatures of -183 oC (-297 oF) .

Because of the low temperatures of cryogenic propellants, they are difficult to store over long periods of time. For this reason, they
are less desirable for use in military rockets that must be kept launch ready for months at a time. Furthermore, liquid hydrogen has
a very low density (0.071 g/ml) and, therefore, requires a storage volume many times greater than other fuels. Despite these
drawbacks, the high efficiency of liquid oxygen/liquid hydrogen makes these problems worth coping with when reaction time and
storability are not too critical. Liquid hydrogen delivers a specific impulse about 30%-40% higher than most other rocket fuels.

Liquid oxygen and liquid hydrogen are used as the propellant in the high efficiency main engines of the Space Shuttle. LOX/LH2
also powered the upper stages of the Saturn V and Saturn 1B rockets, as well as the Centaur upper stage, the United States' first
LOX/LH2 rocket (1962).

Another cryogenic fuel with desirable properties for space propulsion systems is liquid methane (-162 oC). When burned with liquid
oxygen, methane is higher performing than state-of-the-art storable propellants but without the volume increase common with
LOX/LH2 systems, which results in an overall lower vehicle mass as compared to common hypergolic propellants. LOX/methane
is also clean burning and non-toxic. Future missions to Mars will likely use methane fuel because it can be manufactured partly
from Martian in-situ resources. LOX/methane has no flight history and very limited ground-test history.

Liquid fluorine (-188 oC) burning engines have also been developed and fired successfully. Fluorine is not only extremely toxic; it
is a super-oxidizer that reacts, usually violently, with almost everything except nitrogen, the lighter noble gases, and substances
that have already been fluorinated. Despite these drawbacks, fluorine produces very impressive engine performance. It can also
be mixed with liquid oxygen to improve the performance of LOX-burning engines; the resulting mixture is called FLOX. Because of
fluorine's high toxicity, it has been largely abandoned by most space-faring nations.

Some fluorine containing compounds, such as chlorine pentafluoride, have also been considered for use as an 'oxidizer' in deep-
space applications.

Hypergolic propellants are fuels and oxidizers which ignite spontaneously on contact with each other and require no ignition source.
The easy start and restart capability of hypergols make them ideal for spacecraft maneuvering systems. Also, since hypergols
remain liquid at normal temperatures, they do not pose the storage problems of cryogenic propellants. hypergols are highly toxic
and must be handled with extreme care.

Hypergolic fuels commonly include hydrazine, monomethyl hydrazine (MMH) and unsymmetrical dimethyl hydrazine (UDMH).
Hydrazine gives the best performance as a rocket fuel, but it has a high freezing point and is too unstable for use as a coolant.
MMH is more stable and gives the best performance when freezing point is an issue, such as

www.Vidyarthiplus.com
www.Vidyarthiplus.com

Solid Propellants
Solid propellant motors are the simplest of all rocket designs. They consist of a casing, usually steel, filled with a mixture of solid
compounds (fuel and oxidizer) which burn at a rapid rate, expelling hot gases from a nozzle to produce thrust. When ignited, a
solid propellant burns from the center out towards the sides of the casing. The shape of the center channel determines the rate
and pattern of the burn, thus providing a means to control thrust. Unlike liquid propellant engines, solid propellant motors can not
be shut down. Once ignited, they will burn until all the propellant is exhausted.

There are two families of solids propellants: homogeneous and composite. Both types are dense, stable at ordinary temperatures,
and easily storable.

Homogeneous propellants are either simple base or double base. A simple base propellant consists of a single compound, usually
nitrocellulose, which has both an oxidation capacity and a reduction capacity. Double base propellants usually consist of
nitrocellulose and nitroglycerine, to which a plasticiser is added. Homogeneous propellants do not usually have specific impulses
greater than about 210 seconds under normal conditions. Their main asset is that they do not produce traceable fumes and are,
therefore, commonly used in tactical weapons. They are also often used to perform subsidiary functions such as jettisoning spent
parts or separating one stage from another.

Modern composite propellants are heterogeneous powders (mixtures) which use a crystallized or finely ground mineral salt as an
oxidizer, often ammonium perchlorate, which constitutes between 60% and 90% of the mass of the propellant. The fuel itself is
generally aluminum. The propellant is held together by a polymeric binder, usually polyurethane or polybutadienes, which is also
consumed as fuel. Additional compounds are sometimes included, such as a catalyst to help increase the burning rate, or other
agents to make the powder easier to manufacture. The final product is rubberlike substance with the consistency of a hard rubber
eraser.

Composite propellants are often identified by the type of polymeric binder used. The two most common binders are polybutadiene
acrylic acid acrylonitrile (PBAN) and hydroxyterminator polybutadiene (HTPB). PBAN formulations give a slightly higher specific
impulse, density, and burn rate than equivalent formulations using HTPB. However, PBAN propellant is the more difficult to mix
and process and requires an elevated curing temperature. HTPB binder is stronger and more flexible than PBAN binder. Both
PBAN and HTPB formulations result in propellants that deliver excellent performance, have good mechanical properties, and offer
potentially long burn times.

Solid propellant motors have a variety of uses. Small solids often power the final stage of a launch vehicle, or attach to payloads
to boost them to higher orbits. Medium solids such as the Payload Assist Module (PAM) and the Inertial Upper Stage (IUS) provide
the added boost to place satellites into geosynchronous orbit or on planetary trajectories.

The Titan, Delta, and Space Shuttle launch vehicles use strap-on solid propellant rockets to provide added thrust at liftoff. The
Space Shuttle uses the largest solid rocket motors ever built and flown. Each booster contains 500,000 kg (1,100,000 pounds) of
propellant and can produce up to 14,680,000 Newtons (3,300,000 pounds) of thrust.

www.Vidyarthiplus.com
ROCKET PROPELLANT PERFORMANCE

Combustion chamber pressure, Pc = 68 atm (1000 PSI) ... Nozzle exit pressure, Pe = 1 atm
Oxidizer Fuel Hypergolic Mixture Ratio
www.Vidyarthiplus.com

An Introduction to Hybrid Rockets

Hybrid Rocket Engines are those which use liquid oxidizer and a solid fuel. Below figure shows typical
elements of an Hybrid Rocket Engine.

The liquid oxidizer is atomized and sprayed over the fuel block. In hypergolic systems, only gas phase
reactions occur. The oxidizer content of the hot product gases decreases along the port and the length of
the grain.

• Two of the issues in this combustion process are


• (i) mixing of the oxidizer rich and fuel rich gases across the diffusion flame occurs much later than
the length of the fuel grain and
• (ii) fuel regression rate is small.
www.Vidyarthiplus.com
The first issue is resolved by adding mixing devices and second issue is solved by adding a certain
amount of oxidizer into the fuel.

www.Vidyarthiplus.com
• Hybrid rocket engines retain the advantage of controllability like liquid rockets. The added safety is
an attraction for use of hybrid rockets in situations calling for safety similar to civil aircraft
operations. There may be possibilities for their use in single stageto-orbit vehicles providing low
cost access to space.

When considering different methods of propelling an aerospace vehicle, it must be realized that there is an
overall hierarchy of engines that produce a desired thrust. There are air-breathing engines, which include
most sub-orbital vehicles such as airplanes and jets, and then there are spacecraft engines.
Among spacecraft engines there are two general types, those being electric propulsion and chemical
propulsion. Electric motors are very efficient and make excellent use of fuel, but provide very little thrust.
Chemical rockets, however, are powerful enough to launch payloads from the ground into orbit.
In chemical rockets, the idea is to combine two substances, a fuel and an oxidizer, in some mixing region.
The chemical energy associated with combining these two substances is transferred to the total flow as
thermal (kinetic) energy. This high-energy flow can then be expanded out a nozzle to provide thrust for the
attached vehicle.
One major issue involved is apparent, for we need to what substance are best usable as oxidizer and fuel.
However, the even larger question is : what is the best way to mix the fuel and oxidizer? The two long-
standing answers to this question involve liquid and solid rockets. However, a third response to this question
seems to be feasible these days, and that answer involves hybrid rockets.
To review, liquid rockets utilize liquid fuel and liquid oxidizer stored in tanks. By either pressure feeding or by
mechanically pumping the propellants from their tanks, they are forced into a mixing chamber where
chemical combustion occurs. These types of systems generally provide good thrust and can be thrust-
controlled (throttled). In addition, they tend to be the most efficient of high-thrust engines. However, the
complexity of these systems is also high. There are stopvalves, pressure regulators, injectors, turbopump
machinery and all sorts of “plumbing”. When considering that there needs to be redundancies on all of these
systems in order to make a reliable motor, it easy to see that the overall cost and weight of liquid rockets will
be excessive. In addition, due to the liquid nature of the propellants involved, there can also be storage
problems.
Solid rockets are somewhat different in nature, but also have a specific set of advantages and drawbacks.
In solid rocket motors, the fuel and oxidizer are chemically premixed to form a solid fuel grain. By simply
igniting this substance, the oxidizer and fuel in the solid react and produce the high-energy combustion gases
desired. A variety of designs for the central burning port of the solid fuel can be created so as to produce the
desired thrust performance. Solid rockets provide good thrust and are the most simple systems available.
On the down side, they also are fairly inefficient fuel burners and cannot be throttled. In some cases there
may also be explosion dangers since the oxidizer and fuel are not separated.
It appears necessary to obtain some "optimal" solution to this dilemma. On the one hand, we have a high-
thrust rocket engine with good performance but high complexity and cost, while on the other hand to get low

www.Vidyarthiplus.com
complexity we must accept lower performance as well. It is at this point where hybrid rockets become an
attractive alternative. Hybrid rockets combine elements
Coasting Flight
Coasting is defined as the free flight of a space vehicle during which the thrust acting on it is zero.

The thrust is zero after the “burn out” and the rocket coasts. During this flight the rocket ascends to the
maximum altitude and decelerates to zero velocity. Therefore,

u = up - gtc = 0

The coasting time is given by, tc = up / g

The gain in altitude during coasting is given by,

Zc = uptc− gtc2

Thrust vectoring
• Thrust vectoring is the ability of an aircraft or rocket or other vehicle to deflect the angle of its thrust
away from the vehicles longitudinal axis.
www.Vidyarthiplus.com
www.Vidyarthiplus.com

• The advantages of thrust vectoring systems on aircraft include improved post stall performance, the
ability (ability to change the body's position, and requires a combination of balance, coordination, speed,
strength, endurance etc.) to operate on damaged airfields due to reduced takeoff distances and overall
enhanced agility.

• These factors can provide substantial benefits for military aircraft, which are primarily concerned with
manoeuvrability and control.

• The concept of thrust vectoring is not a new one. The Germans used graphite control vanes in the
exhaust stream of their V-2 ballistic missile in World War II for some directional control.

• Thrust vectoring in aircraft though is a relatively new practice and the concept came under widespread
consideration during the cold war.

• There are several methods employed to produce thrust vectoring.

• Most current production aircraft with thrust vectoring use turbofan engines with rotating nozzles or turning
vanes to deflect the exhaust stream. This method can deflect thrust to as much as 90 degrees providing
a vertical take off and landing capability. However for vertical thrust the engine has to be more powerful
to overcome the weight of the aircraft, this means the aircraft requires a bigger heavier engine. As a
result of the increased overall weight of the aircraft the manoeuvrability and agility are reduced in normal
horizontal flight.

• Another method to produce thrust vectoring is through fluidic thrust vector control. This is achieved using
a static nozzle and a secondary flow between the primary jet and the nozzle. This method is desirable
for its lower weight, mechanical simplicity and lower radar cross section.

Advantages and Disadvantages of Thrust Vector Control

• Thrust-vectoring research to date has successfully identified and demonstrated many potential benefits
to high-performance aircraft.

• These include enhanced aircraft manoeuvrability, performance, survivability, and stealth.

• The full extent of these benefits, however, has yet to be realized even with new generation aircraft
because current mechanical thrust-vectoring configurations are heavy, complex, and expensive.

Countercurrent shear layer enhancement for fluidic thrust vector control

www.Vidyarthiplus.com
www.Vidyarthiplus.com

Photo courtesy Pratt & Whitney, A United Technologies Company

F119 engine for F/A-22 Raptor showing the 2 extreme vectoring cases

• Thrust Vector Control or Thrust Vectoring is a technology that deflects the mean flow of an engine jet
from the centerline in order to transfer some force to the aimed axis. By that imbalance, a momentum is
created and used to control the change of attitude of the aircraft.

• Among other things, thrust vectoring greatly improves maneuverability, even at high angles of attack or
low speeds where conventional aerodynamic control surfaces lose all effectiveness.

• Thrust Vector Control is currently achieved by complex arrays of mechanical actuators capable of
modifying the geometry of the nozzle and thus defect the flow.

• This variable geometry greatly increases weight and maintenance to the engine, and therefore limits the
benefits from vectoring the thrust.

• Fluidic Thrust Vector Control is a technology aiming at the above listed benefits by the use of fluidic
means, implying less complexity and faster dynamic responses.

• Different concepts have been developed in the last decade to redirect the thrust without mechanical
actuators.

• Induction to flow separation, countercurrent shear layer, synthetic pulses or skewing of the sonic line are
some of the proven concepts.
www.Vidyarthiplus.com

• Early in the 20th century, an American scientist, Robert H. Goddard (1882-1945), he began to try various
types of solid fuels and to measure the exhaust velocities of the burning gases.

• Since the earliest days of discovery and experimentation, rockets have evolved from simple gunpowder
devices into giant vehicles capable of travelling into outer space.

• Rockets have opened the universe to direct exploration by humankind.

• A third great space pioneer, Hermann Oberth (1894-1989) of Germany, published a book in 1923 about
travel into outer space has led to the development of the V-2 rocket. The V-2 rocket (in Germany called
the A-4) was small by comparison to today's designs.

• It achieved its great thrust by burning a mixture of liquid oxygen and alcohol at a rate of about one ton
every seven seconds.

• Once launched, the V-2 was a formidable weapon that could devastate whole city blocks. Other than
that, the V-2 rocket use graphite vanes in the exhaust to achieve the thrust vector control.

www.Vidyarthiplus.com
www.Vidyarthiplus.com

www.Vidyarthiplus.com
www.Vidyarthiplus.com

www.Vidyarthiplus.com
www.Vidyarthiplus.com

• The concept of thrust vectoring is not a new one. The Germans used graphite control vanes in the
exhaust stream of their V-2 ballistic missile in World War II for some directional control. Thrust vectoring
in aircraft though is a relatively new practice and the concept came under widespread consideration
during the cold war.

• There are several methods employed to produce thrust vectoring. Most current production aircraft with
thrust vectoring use turbofan engines with rotating nozzles or turning vanes to deflect the exhaust
stream.

• This method can deflect thrust to as much as 90 degrees providing a vertical take off and landing
capability. However for vertical thrust the engine has to be more powerful to overcome the weight of the
aircraft, this means the aircraft requires a bigger heavier engine. As a result of the increased overall
weight of the aircraft the manoeuvrability and agility are reduced in normal horizontal flight.
Thrust vector control in rockets

• All chemical propulsion systems can be provided with one of several types of thrust vector control (TVC)
mechanisms.

• Some of these apply either to solid, hybrid, or to liquid propellant rocket propulsion systems, but most
are specific to only one of these propulsion categories.

• Thrust vector control is effective only while the propulsion system is operating and creating an exhaust
jet. For the flight period, when a rocket propulsion system is not firing and therefore its TVC is inoperative,
a separate mechanism needs to be provided to the flying vehicle for achieving control over its attitude
or flight path.
Hence, there are two types of thrust vector control concept:
(1) for an engine or a motor with a single nozzle; and (2) for
those that have two or more nozzles.

TVC Mechanisms with a single nozzle


• Mechanical deflection of the nozzle or thrust chamber.

• Insertion of heat-resistant movable bodies into the exhaust jet; these experience aerodynamic forces
and cause a deflection of a part of the exhaust gas flow.

• Injection of fluid into the side of the diverging nozzle section, causing an asymmetrical distortion of
the supersonic exhaust flow.

• Separate thrust-producing devices that are not part of the main flow through nozzle.

www.Vidyarthiplus.com
Types of rocket engines
Physically powered
Type Description Advantages Disadvantages
Partially filled pressurised Altitude typically limited to a few
Very simple to
water carbonated drinks container with hundred feet or so (world record is
rocket tail and nose weighting build
623 meters/2044 feet)
Non contaminating
cold gas A non combusting form, used for thruster
exhaust Extremely low performance
attitude jets
Hot water is stored in a tank at Simple, fairly safe Low performance due to heavy tank
hot water high temperature/pressure and
rocket
turns to steam in exhaust

Chemically powered
Type Description Advantages Disadvantages
Once lit, extinguishing it is
difficult although often
possible, cannot be throttled in
real time; handling issues from

Simple, often no moving ignitable mixture,


Ignitable, self sustaining lower parts, reasonably good performance
solid fuel/oxidiser mixture than liquid mass fraction, reasonable rockets, if
Solid rocket
("grain") with central hole grain cracks it Isp. A thrust schedule can can block
and nozzle nozzle with be designed into the disastrous
results, cracks grain. burn and widen during burn.
Refuelling grain harder than simply filling
tanks, Lower specific Impulse than Liquid
Rockets.

Some oxidisers are


Quite simple, solid fuel monopropellants, can
Separate oxidiser/fuel, is essentially inert explode in own
right;
Hybrid rocket typically oxidiser is without oxidiser, safer; mechanical failure of solid
liquid and kept in a cracks do not escalate, propellant can block
tank, the other solid throttleable and easy to nozzle, central hole widens
with central hole
switch off. over burn and negatively
affects mixture ratio.
www.Vidyarthiplus.com

Monopropellant Propellant such as Simple in concept, catalysts can be easily rocketHydrazine, Hydrogen
throttleable, low contaminated,
Peroxide or Nitrous temperatures in monopropellants can Oxide, flows over combustion
chamber detonate if contaminated or catalyst and provoked, Isp is perhaps 1/3
exothermically of best liquids decomposes and hot gases are emitted
through nozzle

Up to ~99% efficient Pumps needed for high Two fluid


(typically combustion with performance are expensive liquid) propellants are excellent
Liquid mixture to design, huge thermal introduced through control, throttleable, can fluxes
Bipropellant across combustion injectors into be used with turbopumps chamber wall can impact
rocket combustion chamber which permits incredibly reuse, failure modes and burnt
lightweight tanks, can be include major explosions, a safe with extreme care lot of
plumbing is needed.

Dual mode Rocket takes off as a bipropellant


propulsion rocket,
rocket Simplicity and ease of Lower performance than
then turns to using just
control bipropellants
one propellant as a monopropellant
Three different propellants
(usually hydrogen,
hydrocarbon
Tripropellant rocket
added to the airstream dangerous. Much heavier to increase maximum than
simple rockets. altitude
Easily tested on ground.
High thrust/weight ratios
Intake air is chilled to are possible (~14) very low
temperatures together with good fuel
Precooled jet at inlet before passing efficiency over a wide Exists only at the lab engine / LACE through
a ramjet or range of airspeeds, mach prototyping stage.
(combined cycle turbojet engine. Can be 0-5.5+; this combination Examples include RB545, with rocket)
combined with a of efficiencies may SABRE, ATREX
rocket engine for permit launching to orbital
insertion. orbit, single stage, or very rapid
intercontinental travel.

Electrically powered

Type Description Advantages Disadvantages

www.Vidyarthiplus.com
A monopropellant is Higher Isp than
Resistojet rocket electrically heated by monopropellant Uses a lot of power and hence
(electric heating) a filament for extra alone, about 40% gives typically low thrust
performance higher.
Similar to resistojet in
Arcjet rocket concept but with inert
Very low thrust and high power,
(chemical burning propellant, except an
1600 seconds I aided by electrical arc is used which sp
performance is similar to Ion
drive.
discharge) allows higher
temperatures
Pulsed plasma
Plasma is used to High Isp , can be
thruster (electric erode a solid pulsed on and off Low energetic efficiency
arc heating; emits
propellant for attitude control
plasma)
similar thrust/weight ratio with ion
drives (worse), thermal
Variable specific issues, as with ion drives very
Microwave heated Variable Isp from
impulse high power requirements for plasma with magnetic 1000 seconds to magnetoplasma
significant thrust, really needs throat/nozzle 10,000 seconds
rocket advanced nuclear reactors, never
flown, requires low temperatures for
superconductors to work
Solar powered
The Solar thermal rocket would make use of solar power to directly heat reaction mass, and therefore does
not require an electrical generator as most other forms of solar-powered propulsion do. A solar thermal rocket
only has to carry the means of capturing solar energy, such as concentrators and mirrors. The heated
propellant is fed through a conventional rocket nozzle to produce thrust. The engine thrust is directly related
to the surface area of the solar collector and to the local intensity of the solar radiation.

Type Description Advantages Disadvantages


Simple design. Using hydrogen
Only useful once in space, as
propellant, 900 seconds of Isp is
thrust is fairly low, but
comparable to Nuclear Thermal rocket,
Solar Propellant is hydrogen is not easily stored without the problems and complexity
thermal heated by in space, otherwise
of controlling a fission reaction. Using
rocket solar collector moderate/low Isp if higher–
higher–molecular-weight propellants,
molecular-mass propellants
for example water water, lowers are used
performance.

Beam powered rocket from a distance Advantages Disadvantages


~1 MW of power per kg of payload is needed
Type Description to achieve orbit, relatively high accelerations,
lasers are blocked by
simple in clouds, fog, reflected laser light may be
Propellant is heated by principle, in dangerous, pretty much needs hydrogen
light beam (often principle very high monopropellant for good performance
light beam laser) aimed at powered exhaust speeds can
which needs heavy tankage, some
vehicle from a rocket
be achieved
distance, either directly or designs are limited to ~600 seconds due to
reemission of light since
indirectly via heat exchanger propellant/heat exchanger gets white hot
~1 MW of power per kg of payload is needed
microwave Propellant is heated to achieve orbit, relatively high accelerations,
microwaves are absorbed
beam by microwave beam
microwaves avoid
powered aimed at vehicle to a degree by rain, reflected
reemission of microwaves may be dangerous, pretty
energy, so ~900 much needs hydrogen monopropellant
seconds exhaust for good performance which needs
www.Vidyarthiplus.com

speeds might be heavy tankage, achieveable measured in kilometres to achieve a fine enough beam
transmitter to hit a vehicle at up to 100 km.
diameter is
Nuclear powered
Nuclear propulsion includes a wide variety of propulsion methods that use some form of nuclear reaction
as their primary power source. Various types of nuclear propulsion have been proposed, and some of them
tested, for spacecraft applications:

Type Description Advantages Disadvantages


Radioisotope
Heat from
rocket/"Poodle about 700-800
radioactive decay
thruster" seconds, almost no low thrust/weight ratio.
is used to heat
(radioactive decay moving parts
hydrogen
energy)
Maximum temperature is limited
propellant (typ. by materials technology, some
Isp can be high, hydrogen) is radioactive particles
can be
Nuclear thermal perhaps 900 seconds passed through a present in exhaust in
some
rocket (nuclear or more, above unity
nuclear reactor to designs, nuclear reactor shielding
fission energy) thrust/weight ratio heat to high is heavy, unlikely to be permitted with some designs
temperature from surface of the
Earth, thrust/weight ratio is not
high.
Nuclear reaction Very hot propellant, difficulties in heating propellant using a
gaseous not limited by without losing fissionables in
Gas core reactor state fission keeping reactor solid, exhaust, exhaust inherently
rocket (nuclear reactor in intimate Isp between 1500 and highly radioactive, massive
fission energy) contact with 3000 seconds but thermal issues particularly for propellant with
very high thrust
below Earth's magnetosphere.
Antimatter Containment of antimatter,
Nuclear pulse
catalyzed nuclear production of antimatter in propulsion with Smaller sized
vehicle
pulse propulsion macroscopic quantities isn't antimatter assist might be possible

www.Vidyarthiplus.com
(fission and/or currently feasible. Theoretical
for smaller bombs
fusion energy) only at this point.
Fusion rocket
Fusion is used to Very high exhaust Largely beyond current state of
(nuclear fusion
heat propellant velocity the art.
energy)
Problems with antimatter
Antimatter rocket Antimatter Extremely energetic, production and handling; energy
(annihilation annihilation heats very high theoretical losses in neutrinos, gamma rays, energy)
propellant exhaust velocity muons; thermal issues.
Theoretical only at this point

www.Vidyarthiplus.com

Solid Propellant Rockets


www.Vidyarthiplus.com
Advantages Disadvantages

Simple Design (few or no moving parts) Explosion and fire potential is larger; failure can be
catastrophic

Easy to operate (little preflight checkout) Many require environmental permit and safety
features for transport on public conveyances

Ready to operate quickly Under certain conditions some solid


propellants can detonate

Will not leak, spill, or slosh Cumulative grain damage occurs through
temperature cycling or rough handling; this limits
useful life

Can be stored for 5 to 25 years If designed for reuse, it requires extensive factory
rework and new propellants

Usually, higher overall density; this allows a more Requires an ignition system
compact package, a small vehicle (less drag)

Can provide TVC, but at increased complexity Once ignited, cannot change predetermined thrust or
duration

Some propellants have nontoxic, clean exhaust Integrity of grain (cracks, unbounded areas) is difficult
gases, but at a performance penalty to determine in the field

Some grain and case design can be used with several Large boosters take a few seconds to start
nozzles

Thrust termination devices permit control over total Cannot be tested prior to use
impulse

Can be designed for recovery and reuse Thermal insulation is required in almost all rocket
motors

Some tactical missile motors have been produced in Needs a safety provision to prevent inadvertent
large quantities ignition, which would lead to an unplanned motor
firing. Can cause a disaster.

www.Vidyarthiplus.com
www.Vidyarthiplus.com

Liquid Propellant Rockets


Advantages Disadvantages

ulse than solid propellant rockets Relatively complex design, more parts or components, more
things to go wrong!

throttled and randomly stopped and Cryogenic propellants cannot be stored for long periods
except when tanks are well insulated and escaping vapours
are recondensed. Propellant loading occurs at the launch
stand and requires cryogenic propellant storage facilities

e can be randomly controlled; this allows a Spills or leaks of several propellants can be hazardous,
t trajectory corrosive, toxic, and cause fires, but this can be minimized
with gelled propellants

an be controllable with thrust termination More overall weight for most short-duration, lowtotal-impulse
ntrol of vehicle terminal velocity) applications.

full thrust on ground or launch pad prior to Non-hypergolic propellants require an ignition system

for reuse after field services and checkout Tanks need to be pressurized by a separate pressurization
system. This can require high pressure inert gas storage for
long periods of time.

(or some part of the vehicle) can be cooled Bullet impact will cause leaks, sometimes a fire, but usually
eight no detonations; gelled propellants can minimize or eliminate
these hazards.

opellants have been kept in vehicle for more Usually requires more volume due to lower average
d engine can be ready to operate quickly. propellant density and relatively inefficient packaging of
engine components

ts have nontoxic exhaust, which is Sloshing in tank can cause a flight stability problem, but can
acceptable be minimized with baffles.

ating conditions during firing to prevent some Smoky exhaust (soot) plume can occur with some
d otherwise result in the loss of the mission hydrocarbon fuels

ponent redundancy (e.g., dual check valves Needs special design provisions for start in zero gravity
amber) to enhance reliability
www.Vidyarthiplus.com
and smoke are usually low High-thrust unit requires several seconds to start

Appendix
Chemical Rocket Systems
The primary rocket engine technology at the present time is the chemical rocket engine, and it is likely to remain the
dominant technology for some time to come. This chapter provides a short introduction to chemical rocket engine
systems.

mass of the exhaust flow and the greater the velocity of that flow, the greater the recoil generated by the
rocket engine, and the greater the thrust.

There are various ways to generate this thrust, though in all cases the result is the same, to expel a gas at
high velocity. Nuclear rocket engines run a fluid through a nuclear reactor. Electric rocket engines typically
accelerate ions to high velocities using electrified grids. Chemical rocket engines, the subject of this chapter,
burn a "fuel" and an "oxidizer", either in a solid mixture or stored as liquids in separate tanks, and blast the
exhaust out a usually bell-shaped ("convergentdivergent" or "con-di") nozzle.

Rocket engine thrust is formally measured in newtons (N) in the metric system; in pounds force
(lbf) in the English system; and sometimes in kilograms force (kgp, where the "p" stands for the French
"puissance / force"). Since the kilogram is a measure of mass, not force, kgp is a little dubious from the strict
physics point of view, but it is equivalent to newtons divided by 9.81, and at least at one time was a fairly
common measure of thrust.

Efficiency of a rocket engine can be measured in terms of exhaust velocity, but since the actual thrust is also
dependent on the mass of the exhaust gas, a more useful measure is "specific impulse (Isp)", or thrust
produced by a unit mass of propellant per second. In metric units, Isp is defined as "newtons per kilogram of
propellant per second", and in English units it is defined as "pounds thrust per pound of propellant per

www.Vidyarthiplus.com
second". The second definition, by the way, evaluates to "seconds", and that's normally how specific impulse
is described. Specific impulse can be thought of as an index of the "mass ratio" of a rocket vehicle, or the
ratio of payload to vehicle mass: the higher the specific impulse, the greater the efficiency in terms of the
amount of payload per fuel mass.

However, this is a somewhat narrow definition of "efficiency", a much more practical one being how much
payload can be lifted at a given cost, and in such terms an engine with the highest specific impulse may not
be the most efficient. In addition, high specific impulse does not necessarily mean high thrust, and in fact as
later chapters will show, highly efficient engines with high values of specific impulse tend to have very low
thrust.

* A rocket vehicle obviously consists of some sort of airframe or casing mounting a rocket engine and
providing storage for propellants. Less obviously, it must also carry guidance and control systems.

Holiday firework rockets simply have a stick or fins to keep them flying straight. Unguided rocket projectiles
used by combat aircraft to attack ground targets or launched as a form of "barrage" artillery also generally
use fins, though they also may have multiple rocket exhausts canted at an angle around the centerline of the
rocket to cause them to spin for stabilization.

For large rocket vehicles, such as long-range missiles or space launch boosters, which are the focus of this
discussion, more sophisticated control schemes are required. While some large rocket vehicles do have fins,
fins are only useful at low altitudes, since they are ineffective once the vehicle leaves the atmosphere. There
are several approaches for control of such large vehicles:
www.Vidyarthiplus.com

• The early "V-2" missile, built by the Germans in World War II to attack England, used moveable
graphite vanes in the rocket engine exhaust. This caused some loss of thrust, and so this approach
was later abandoned. The V-2 used a gyroscopic control system to move the vanes. The control
system included three gyros, one each for the two horizontal and one vertical directions, and
responded to changes in the rocket's attitude and movement by changing the direction of the vanes.
Modern rocket vehicles have retained gyroscopic control systems, though they will most likely now
be implemented with solidstate "ring laser gyros" instead of rotating gyroscopes.
• A more modern flight control scheme is to use a "gimballed" rocket engine, in which the rocket nozzle
can be moved back and forth or side to side to change the vehicle's direction of flight.
• Another modern scheme is to use "verniers", or small auxiliary rocket engines that produce thrust
off to the sides of the vehicle to change flight direction.

Most modern large rocket vehicles are actually assemblages of separate rocket vehicles, known as "stages",
that are stacked on top of each other. A big rocket vehicle contains a large amount of fuel, and as the fuel is
burned up, the vehicle carries more and more useless dead weight. With "staging", when one stage is
exhausted, the next ignites and the first is discarded.

Staging is a tricky operation, in essence trying to launch one rocket vehicle on top of another while the whole
assembly is in flight, and some early long-range missiles used dodges to simplify the scheme. The original
American Atlas missile, for example, used "half staging", with a three-engine assembly in a skirt at the
bottom; two of the engines were discarded along with the skirt after initial boost. The Soviet R-7 missile / SL-
1 booster used "clustering", with four auxiliary boosters clustered around a similar central "core" booster, the
auxiliary boosters being discarded after initial boost. Both these schemes allowed all the engines to be ignited
at take-off, simplifying launch procedures. They were perfectly practical rocket vehicles, and in fact their
descendants are still in use.

More modern large rocket vehicles use true staging, though they often also use a set of auxiliary solid-fuel
or liquid-fuel "strap-on" boosters attached to the first stage to provide additional thrust. Each stage is
connected to the others by a "shroud", or the case of many Soviet-Russian missiles and boosters, by an
open framework. The open framework has the advantage that the upper stage can be ignited before the
lower stage is discarded, which simplifies the staging process.

The Soviets also came up with an ingenious staging scheme for their R-27 / SS-N-6 submarinelaunched
ballistic missile to produce a more compact vehicle, with the engines for the payload stage actually contained
in the fuel tank for the main stage. Pyrotechnics were used to cut open the tank so the rocket engine could
ignite.

* While the lower stages of a multistage rocket vehicle are discarded before the machine leaves the
atmosphere, the upper stages and the spacecraft they carry, if there is a distinction, need to operate in space.
Such "space vehicles" generally need "restartable" rocket engines that can be turned on or off, which is a
somewhat tricky problem because under "zero-gee" conditions the propellants do not tend to flow to the
bottom of the tanks.

www.Vidyarthiplus.com
www.Vidyarthiplus.com

www.Vidyarthiplus.com
www.Vidyarthiplus.com

fed engines fueled by LOX and propane. LOX tends to be self-pressurizing as it boils off, and propane
becomes self-pressurizing if heated slightly.

Most modern rocket engines use high-speed turbine pumps to drive the propellants into the combustion
chamber at a very fast rate. The turbopumps are turned on with a "starter", an electrically activated solid-fuel
charge that spins up the drive turbine for the turbopumps. However, even rocket vehicles that use
turbopumps still pressurize the propellant tanks, not so much to drive propellant flow rate as to prevent a
void from arising as the tank is vacated, which would otherwise stifle thrust or even cause the propellant
tanks to collapse.

The fuel line is generally wrapped around the engine nozzle, helping to cool the engine and also "preheat"
the fuel, conserving some of the combustion energy that would otherwise be wasted. This is known as
"regenerative" cooling. While regenerative cooling is common now, early rockets used water cooling, or
various types of "heat sinks" -- sets of fins or other structures that could radiate the heat away.

There is also usually a second set of valves downstream from the turbopumps. These are kept closed until
propellant pressures build up enough to ensure a regular fuel flow into the combustion chamber. If the
propellants were allowed to trickle unevenly into the combustion chamber, the burn would be irregular and
poorly controlled.

For similar reasons, the propellant feed system is usually designed to make sure that oxidizer flow reaches
its proper flow level before fuel does. However, modern liquid fuel rocket engines are often designed to burn
"fuel rich", throwing so much fuel into the combustion chamber that all of it can't be burned. This may seem
inefficient, but it allows an increase in exhaust mass flow, and so thrust, without raising exhaust temperature.
Determining the proper propellant ratio is a fine art.

The propellants are forced into the combustion chamber using an "injector" system that ensures they mix
properly, which can be visualized as something like a shower spray head. The engine burn is then initiated
with an "igniter" system. Igniter systems can be based on electrical spark plugs, pyrotechnic charges,
electroresistive heating elements, and even small igniter rockets using hypergolic storable propellants.

The exhaust then pours out the engine nozzle, producing thrust. The input to the nozzle is a narrow
constriction, where the exhaust flows at high pressure and low velocity. As the exhaust flows down the nozzle
it expands, losing pressure but gaining velocity, resulting in greater momentum transfer to the rocket vehicle.

There is a tradeoff between nozzle size and thrust. If the nozzle is too small, the exhaust will be robbed of
thrust. If the nozzle is too big, the exhaust will separate from the nozzle wall, not only making the larger size
useless but also counterproductive, since it sets up turbulence that robs the exhaust of thrust. The behavior
of the exhaust varies with atmospheric pressure, which of course falls off as the rocket climbs, and so
optimizing the exhaust design is a troublesome matter. It is also obvious that building a big engine is more
difficult and expensive than a small one. As a

www.Vidyarthiplus.com
www.Vidyarthiplus.com
www.Vidyarthiplus.com

flow to the main combustion chamber shut off propellant flow, and the engine burn halts immediately.

Of course, a controlled shutdown is absolutely essential in a restartable engine. As mentioned, ensuring


fuel flow in a restartable engine is tricky because the propellants cannot flow down to the engine, since
"down" doesn't exist under weightless conditions. There are several approaches:

• The most conceptually obvious scheme is to fit bladders in the propellant tanks that are filled by
compressed gas to force the fluids out.
• A somewhat less obvious scheme is to fit a set of screens into the tank, with the mesh becoming
finer as they near the bottom of the tank. Surface tension of the fuel on the mesh causes the
propellant to "wick" towards the bottom of the tank.
• A third, even trickier but common, approach is to use a thruster to give the rocket vehicle a small,
short acceleration to cause fuel to flow into the main engine. Ignition of the main engine sustains the
flow, and the thruster is reloaded with propellant during the main engine burn to prepare it for another
restart.

* Modern LOX-hydrocarbon rocket engines burn LOX and kerosene, or more precisely a highly refined grade
of kerosene named "RP-1" or just "RP" for short, which some sources claim stands for "rocket propellant"
and others claim stands for "refined petroleum" -- take your pick. RP looks and smells like ordinary kerosene,
but it is pure and has highly predictable burn and density characteristics. If a large rocket vehicle were fueled
with normal kerosene it might burn, but not evenly, and the fuel weight could vary by a matter of tonnes,
which would make the performance of the vehicle very unpredictable.

LOX-RP is traditionally one of the most popular liquid-fuel schemes. LOX-RP engines typically have a
specific impulse of about 260 seconds. This will be used as a "baseline" value in the rest of this document,
with specific impulse values given relative to it. This avoids the question of metric or English units and the
less-than-intuitive use of "seconds" as a unit of measure for rocket efficiency.

LOX-LH2 provides higher specific impulse, about 1.5 times that of LOX-RP, but there is price for it. LH2 is a
low-density fuel, meaning it requires a large tank, and the rocket vehicle has to be big as well. It also has to
be kept very cold, substantially colder than liquid oxygen. The LOX and LH2 tanks have to be thermally
isolated from each other, or the LOX will tend to freeze and the LH2 to boil.

LOX-kerosene and LOX-LH2 are both "cryogenic" propulsion systems, meaning they require cooled
propellants. Both use cooled LOX oxidizer, while LOX-LH2 also requires even colder LH2 fuel. Incidentally,
some sources refer only to LOX-LH2 as "cryogenic" propulsion, but this usage seems a bit misleading and
will not be used in this document. A rocket vehicle using cryogenic propellants has to be fueled a relatively
short time before launch, or the propellants will gradually vaporize away. Launch systems for such vehicles
cycle the cryogenic propellants through an external cooling system to keep losses as low as possible, and
also include a bleed valve to keep vaporized propellants from building up in the tanks and possibly rupturing
them.

www.Vidyarthiplus.com
www.Vidyarthiplus.com
Storable propellants, as the name implies, avoid this problem. They can be loaded into a rocket vehicle and
left indefinitely. One of the earliest storable propellant combinations was concentrated hydrogen peroxide
(HO), known as "high-test peroxide (HTP)", for oxidizer, and aniline (C6H7N1), a benzene derivative, for
fuel.

Since HTP tends to degrade slowly in storage over time, it was generally replaced as an oxidizer by nitric
acid (NHO3), nitrogen tetroxide (N2O4), or combinations of the two. Aniline was replaced by hydrazine
(NH2NH2); unsymmetrical dimethyl hydrazine (UDMH), with the chemical formula N(CH3)2NH2; or a mix of
the two, sometimes called "Aerozine-50". Nitrogen tetroxide and UDMH are a popular combination, and have
a specific impulse only a few percent less than that of LOX-RP.

Storables have a number of drawbacks. Not only are they usually hypergolic, they are also as a rule
extremely toxic and corrosive. Storable propellant tanks have to be lined with stainless steel, and workers
handling these propellants must wear protective clothing and respirators. There were cases in both the Soviet
and American space programs where space capsules returning to Earth fired thrusters fueled by storables
too persistently, and the fumes overcame the crews, though nobody was ever done any long-term harm. The
US space shuttle's orbital propulsion system uses storables, and when the orbiter lands, a crew in protective
clothing has to go out and "safe" it before anyone else is allowed to get near. The nasty behavior of storables
means higher cost, and they are also now generally regarded as environmentally unacceptable and avoided
when possible.

Storables were initially used in military missiles that had to be ready to fire on a moment's notice. Their tanks
were permanently fueled, capped by seals that were blown at the moment of firing. Solid propellants are now
preferred for military missiles, though the Soviets stayed with storables for much longer than the US since
they were comfortable with the technology. The Soviets even used them for submarine-launched missiles,
despite the threat posed by such toxic and violent chemicals in a closed environment. The successor Russian
government has found disposal of large quantities of toxic fuels to be a major environmental headache.

* Storables are still used on spacecraft, such as deep-space probes that will fly through space for years and
make occasional engine burns. Obviously there is no practical way to store cryogenic propellants for such a
long period of time.

Of course, the thruster systems on such spacecraft are also based on storable fuels. Thruster systems are
in general simple, very low thrust, restartable rockets. The most elaborate use storable propellants, typically
N2O4-UDMH, but "monopropellant" thrusters are also used. These feature a fuel, usually hydrazine, that
burns when passed over a catalyst. Monopropellant thrusters are much simpler and, in principle, more
reliable than a bipropellant system, but they are much less efficient, with a monopropellant hydrazine thruster
having a specific impulse less than 70% that of LOX-RP. Whether bipropellant or monopropellant, the
thrusters are fed using a pressure or electric pump system, as the high propellant flow rates of a turbopump
are not required.
The Atlas intercontinental ballistic missile (ICBM) and booster was powered by three engines based on the
Navaho engine. The half-stage featured two "LR-89" series engines -- which confusingly shared a turbopump
and so could just as well be regarded as a single engine with two thrust chambers -- and an "LR-105" series
engine. The entire engine assembly went through a series of designations, from the "MA-1" of flight test
prototypes to the "MA-5" that was used in maturity. Liftoff thrust of the full MA-5 engine assembly was about
1,920 kN (195,500 kgp / 431,000 lbf), with the thrust falling to about a third of that when the half-stage was
discarded. The Atlas would never be very useful as a weapon, but it would prove a wildly successful space
launch vehicle.

The "Thor" medium-range missile, which would evolve into the "Delta" series of launch vehicles, was
powered by a single "LR-79" or "MB-3" engine, also with 756 kN (77,000 kgp / 170,000 lbf) thrust and based
on the Navaho engine.

The initial version of the "Titan" missile was powered by a different type of engine, the Aerojet "LR-87-3",
which was a single engine with twin thrust chambers, providing a total of about 1,470 kN (150,000 kgp /
330,000 lbf) thrust, burning LOX and kerosene. The Titan upper stage was powered by an Aerojet "LR-91-
3" engine, similar to the LR-87-3 but with a single thrust chamber and providing 356 kN (36,300 kgp / 80,000
lbf) thrust. Later versions of the Titan used modified versions of the LR-87 and LR-91 that burned storable
propellants.
www.Vidyarthiplus.com
www.Vidyarthiplus.com

In the late 1950s, the US wanted to go to still more powerful LOX-RP engines. This led to the development
of the Rocketdyne "H-1" engine, with eight used on the first stage of the early "Saturn I" and "Saturn IB"
boosters, providing 837 kN (85,300 kgp / 90,000 lbf) thrust each. A improved version of the H-1 designated
the "RS-27" was used for later variants of the Delta launch vehicle. The H-1 also was the basis for the scaled-
up Rocketdyne "F-1" engine, with 6,671 kN (680,000 kgp / 1.5 million lbf) thrust. Five such engines powered
the first stage of the "Saturn V" booster that sent Americans to the Moon.

In the meantime, the US had been pioneering LOX-LH2 propulsion, first developing the Pratt & Whitney
"RL10" engines with 66.7 kN (6,800 kgp / 15,000 lbf) thrust each, with two powering the "Centaur" upper
stage of the "Atlas Centaur" booster. The Centaur upper stage still retains this engine in the 21st century, in
the form of the "RL10A-4" variant, and it is also used on the upper stages of the Boeing "Delta III" and "Delta
IV" boosters, in the form of the more advanced "RL10B-2" engine. A throttleable version, the "RL10A-5", was
used on the McDonnell Douglas DC-X/A demonstrator spacecraft in the 1990s.

www.Vidyarthiplus.com
www.Vidyarthiplus.com

Pratt & Whitney is now working on the "RL60" LOX-LH2 engine, which has a similar form factor to the RL10
series but is much more powerful, with a thrust in the 267 kN (27,200 kgp / 60,000 lbf) range.
www.Vidyarthiplus.com

The Soviets followed the SL-1 series of boosters with the much more powerful "Proton" booster, which put
the "Salyut" and "Mir" space stations into orbit. The first stage of the Proton is powered by six "RD-253"
engines using N2O4 / UDMH storable propellants and providing 3,090 kN (315,000 kgp / 695,000 lbf) thrust
each.

Later, the Soviets developed a much more powerful LOX-RP four-chamber engine, the "RD170", with 8,182
kN (834,000 kgp / 1.84 million lbf) thrust. The first stage of the "Zenit" launch vehicle is powered by a single
RD-170 engine. The Soviets also developed a variant of the RD170 designated the "RD-180" with two thrust
chambers instead of four and, unsurprisingly, half the thrust.

In an interesting irony, the RD-180 is regarded as being competitive at the very least with
American rocket engine technology, development of which stagnated for a few decades. The RD-180 has
been adopted for the latest variants of the Atlas booster, the "Atlas III" and the "Atlas V". Both these launch
vehicles have abandoned the old half-stage scheme and now use a single RD-180 engine. The RD-180
engines are provided for these boosters by a collaboration of NPO Energomash of Russia and Pratt &
Whitney in the US.

* As far as storable propellant engines used on spacecraft themselves go, one of the classic examples
was the "Service Propulsion System (SPS)" for the Apollo Command & Service Module (CSM). The SPS
generated 91.2 kN (9,300 kg / 20,500 lb) of thrust. It was a fully restartable pressure-fed engine, with no
turbopumps, featuring redundant subsystems where possible; its design philosophy was to make it as simple
and reliable as possible. Of course, the Apollo Lunar Module (LM) also used storable propellant engines,
with the descent stage engine providing 44.5 kN (4,535 kg / 10,000 lb) thrust and the ascent stage engine
providing 15.6 kN (1,590 kg / 3,500 lb) thrust.

As mentioned, the space shuttle orbiter has a secondary set of storable propellant engines, known as the
"Orbital Maneuvering System (OMS)", for maneuvers after arrival in space. Each of the two OMS engines
on the orbiter provide 27 kN (2,750 kg / 6,070 lb) of thrust.

* While conventional chemical rocket engines continue to be refined, work has also been performed
on new configurations, such as the "linear aerospike" and "rocket-based combined cycle" engines.

NASA was working with private industry on an aerospike engine for the cancelled X-33 experimental reusable
launch vehicle. An aerospike engine is very different in appearance from a conventional rocket engine.
Instead of a bell-shaped nozzle, the aerospike looks like a wedge rammed into the rear of a vehicle, with
nozzle ports, or "thrust cell chambers", along each side of the base of the wedge. Thrust cell chamber
exhaust is confined on one side by the wedge, with air pressure providing confinement on the other side.
Propellant pumps and other hardware are contained inside the wedge.

Other rocket companies have continued to experiment with the aerospike concept, launching small rockets
that feature a spike surrounded by exhaust holes. As with the cancelled X-33 engine, confinement is provided
by the spike on the inside and air pressure on the outside. The

www.Vidyarthiplus.com
time. Asphault was a poor binder, however, tending to crack at cold ambient temperatures, with the cracks
interfering with the burning process; and to flow at high ambient temperatures, requiring the rockets to be
stored nose-down. It also produced a great deal of black smoke, which caused particularly difficulties for use
as "rocket assisted take-off (RATO)" boosters for aircraft. If one aircraft took off with RATO, the next following
it would have to take off through a black haze that blocked the pilot's field of view.

After the war the Americans moved on to more sophisticated solid propellants, using synthetic polymers,
particularly synthetic rubbers such as butadiene tire rubbers, mixed with ammonium perchlorate (NH4ClO4)
oxidizer (which provides higher performance than potassium perchlorate and burns cleaner) and large
proportions of powdered aluminum. The powdered aluminum burned at a high temperature, helping improve
thrust. Use of high proportions of aluminum had been held up for some time because the conventional
wisdom said that proportions greater than 5% wouldn't burn, but this turned out to be superstition when
researchers ignorant of this "rule" tried higher proportions and found they got unprecedented levels of thrust.
Later on, small proportions of iron oxide were included to provide a high-temperature "thermite reaction" with
the aluminum powder. Modern solid fuels provide a specific impulse only a few percentage points below that
of LOX-RP.

The fact that the propellant mix was based on rubberlike polymers allowed the "grain", or solidrocket fuel
element excluding the nozzle), to be cast in large blocks that resisted shrinkage or cracking, which would
have affected the continuity of their burn at the very least and caused catastrophic failure at worst.

www.Vidyarthiplus.com

Electrical resistance heating wires could be inserted in the bottom of the grain as igniters. Since a safe solid
fuel had a high ignition temperature, the wires sometimes ignited a primer element that had a lower ignition
temperature, with this primer then igniting the grain. The primer might even have two "stages", with one
www.Vidyarthiplus.com
element with a low ignition temperature igniting one with an intermediate ignition temperature, with the
second element then igniting the grain.

The central mandrel that defined the hole configuration for the desired flight thrust profile was coated with
Teflon polymer to allow it to be removed from the cured grain. In the early days of solid fuel rockets, grease
was used instead of Teflon, but the grease contaminated the solid fuel and that approach was abandoned.

Modern solid-fuel rocket engines are ideal for military applications. They can be stored almost indefinitely
and aren't overly fussy about how they are handled, and they can be used and launched at any time with
little preparation. Solid fuels are also denser than liquid fuels, allowing missiles to be more compact, if not
that much lighter. This compactness was a particular plus for the development of solid-fuel long-range
strategic nuclear missiles, since it allowed them to be stored in a smaller and cheaper missile silo or be
carried on a submarine.

Considerable effort was invested in the late 1950s to develop processes to manufacture the large grains for
the "Minuteman" ICBM and the "Polaris" submarine-launched ballistic missile (SLBM). These processes
were not trivial: the ammonium perchlorate oxidizer had to be very finely and uniformly milled; all the
materials in the solid fuel recipe had to very uniformly mixed; and the grains had to be poured and then cured
for several days in a vacuum environment to prevent bubbles from forming.

Work was also done to synthesize high-performance solid fuels, which included proportions of nitroglycerine
and a particulate form of nitrocellulose, and in some cases a high explosive known as HMX as well. These
high-performance mixes were unsurprisingly less stable and more troublesome in every respect than
conventional mixes, and so they were only used for small final stages that were more easily handled.

The work on developing long-range solid-fuel missiles also led to technology for solid-rocket flight control,
and for "thrust termination", or shutting down the engine of the upper stage so the warhead could separate
and proceed on its proper trajectory to its target.

Small solid-fuel missiles can use fins for flight control, but that isn't practical for long-range missiles since
they boost out of the atmosphere, making fins so much dead weight. A steerable nozzle can be used as with
a liquid fuel engine, but a simpler scheme was adopted for the Minuteman and Polaris, using a pivoting ring
around the lip of a fixed nozzle to redirect the thrust. The ring was known as a "jetavator". Later, inert freon
gas was selectively injected into the throat of a fixed nozzle to deflect thrust. As far as thrust termination
went, the problem was that once a solid rocket motor is lit, it burns to termination and there's no way to shut
it off. There is a way to cheat, however, by venting the thrust from the stage to the sides or forward so the
warhead could continue on its way by itself.

www.Vidyarthiplus.com

www.Vidyarthiplus.com
www.Vidyarthiplus.com

The first large solid-fuel grain to be developed was the Thiokol "RV-A-10", with a diameter of 79 centimeters
(31 inches) and a length of 4.37 meters (14 feet 4 inches). A refined variant was used for the Sergeant
battlefield missile. There had been doubts up to that time that it was practical to built large solid-fuel rocket
grains, but the RV-A-10 proved beyond doubt that it was possible.

The RV-A-10 led the way to the much larger grains for the three-stage Minuteman ICBM and the Polaris
SLBM. The solid-rocket motor for the first stage of the Minuteman was an order of magnitude bigger than
the RV-A-10, being 1.67 meters (5 feet 5.7 inches) in diameter and 7.42 meters (24 feet 4 inches) long. Both
the Minuteman and Polaris had filament-wound final stages, though the first large-scale filament-wound
stage had been flown on a Vanguard satellite launch vehicle in 1959. High-performance solid fuels were also
introduced for the Minuteman and Polaris in later versions, and a later version of the Minuteman would also
have a second stage with a titanium casing. Ironically, the US imported most of its titanium from the Soviet
Union. Since there were very few civilian uses for titanium at the time, the Soviets had to be aware that it
was mostly being used in American weapon systems.

Work on large solid-fuel grains for the Minuteman and Polaris programs also led to the development of the
first all-solid-fuel space launch vehicle, the LTV "Scout", which would have a long career putting small
payloads into space. In addition, the Minuteman development effort had a direct connection to the
development of segmented solid rocket motors, with Aerojet testing the concept in 1961 by the simple
measures of cutting a Minuteman first stage in half and then splicing it back together with a lock ring joint.

Both Aerojet and United Research, which would later become a component of the modern United Technology
Center (UTC), performed further work and static test firings of segmented rocket boosters. In 1962, UTC
won a contract from the USAF to build a five-segment SRB for the Titan III space launch vehicle. The
resulting SRBs went into service in 1965, with two SRBs straddling the liquid-fuel Titan core. The initial Titan
III SRB motor was 3.05 meters (10 feet) in diameter and 25.8 meters (84 feet 8 inches) long. It led in the
1980s to the 5.5 segment motor for the Titan 34D and the subsequent 7 segment motor for the Titan IV,
which produces about 7,565 kN (771,000 kgp / 1.7 million lbf) thrust per SRB.

The biggest solid-rocket motor ever to be put into operation is the SRB for the US space shuttle. Each SRB
is 45.5 meters (149 feet 2 inches) long. The precise composition of the shuttle SRB grain by weight is:
69.6% ammonium perchlorate oxidizer
16.0% aluminum booster
12.4% polymer binder
2.0% epoxy curing agent
0.4% iron oxide combustion catalyst (thermite reaction with aluminum)
The shuttle SRBs are made up of four segments stacked on top of each other. The hole up the center of the
SRBs is cone-shaped at the bottom, leading to an 11-point star that runs to the top. This scheme gives
maximum thrust of 11,770 kN (1.2 million kgp / 2.65 million lbf) at liftoff, falling off to a sustained level of
thrust after that. The nozzle is steerable.
www.Vidyarthiplus.com
Unit-5 ADVANTAGES OF PROPULSION TECHNIQUES
Electric rocket propulsion – Ion propulsion techniques – Nuclear rocket – Types – Solar sail- Preliminary Concepts in nozzleless
propulsion.
www.Vidyarthiplus.com

• Other propellant materials, such as cesium and mercury, have been investigated in the past, but
xenon is generally preferable because it is not hazardous to handle and process, it does not
condense on spacecraft components that are above cryogenic temperatures, its large mass
compared to other inert gases generates higher thrust for a given input power, and it is easily stored
at high densities and low tank mass fractions. Therefore, the main focus will be on xenon as the
propellant in ion and Hall thrusters, although performance with other propellants can be examined
using the basic information provided here.

• In all electric propulsion the source of the electric power (nuclear, solar radiation receivers, or
batteries) is physically separate from the mechanism that produces thrust.

• This type of propulsion has been handicapped by heavy and inefficient power sources.

• The thrust usually is low, typically 0.005 to 1 N. In order to allow a significant increase in the vehicle
velocity, it is necessary to apply the low thrust and thus a small acceleration for a long time (week
or months).

Electric Rocket Propulsion

Simplified schematic diagram of arc-heating electric rocket propulsion system. The arc plasma temperature
is very high (~ 15,000 K) and the anode, cathode, and chamber will get hot (1000 K) due to heat transfer.
www.Vidyarthiplus.com
www.Vidyarthiplus.com
• Propellant is heated electrically (by heated resistors or electric arcs) and hot gas is then
thermodynamically expanded and accelerated to supersonic velocity through an exhaust nozzle (see
Fig.) .

• These electrothermal units typically have thrust ranges of 0.01 to 0.5 N, with exhaust velocities of
1000 to 5000 m/sec, and ammonium, hydrogen, nitrogen, or hydrazine decomposition product gases
have been used as propellants.

A Typical Ion Rocket

Simplified schematic diagram of a typical ion rocket, showing the approximate distribution of the electric
power

Electric Thruster Types

Electric thrusters are generally described in terms of the acceleration method used to produce the thrust.
These methods can be easily separated into three categories: electro-thermal, electrostatic and
electromagnetic. Common EP thruster types are described in the following.

Resistojet

Resistojets are electrothermal devices in which the propellant is heated by passing through a resistively
heated chamber or over a resistively heated element before entering a downstream

www.Vidyarthiplus.com
www.Vidyarthiplus.com

• The thruster was tested in a large space simulation chamber in the ESA Technology centre in the
Netherlands at a remarkable 30,000 V and produced an ion exhaust plume that travelled at 210 km/s
– over four times faster than state-of-the-art ion engine designs achieve.

History of ion propulsion


• A NASA engineer prepares an early ion engine for a vacuum chamber test in 1959. Lined up at right
are the major electrical parts.
• Among the most difficult challenges in the early development of ion engines was proving that injecting
electrons could neutralize an ion beam.
• Continually spewing positively charged ions will leave a spacecraft with a negative charge so great
that the ions are attracted back to the spacecraft.
• The solution is an electron gun that dumps the electrons into the ion stream, thus neutralizing both
spacecraft and exhaust. But the beam's interaction with the walls of even a large vacuum chamber
makes it very difficult to conduct meaningful beam neutralization experiments on Earth.
• These uncertainties led to considerations for flight testing electric engines.
• Another challenge of electronic propulsion involved developing an efficient technique to produce
ions.
• Working at NASA's Lewis, Harold Kaufman invented an electron-bombardment technique to ionize
mercury atoms.
• At NASA/Marshall, a process was under development whereby cesium atoms would become ionized
upon contact with a hot tungsten or rhenium surface.
• Marshall's major development in electrical propulsion centered, however, on a 30kilowatt ion engine
development contract, initiated in September 1960 with Hughes Research Laboratory in Malibu,
California.
At first, Marshall directed Hughes to design a laboratory model of an ion engine. The 0.01 lb.thrust model
would be followed by the development of a 0.1 lb.-thrust engine. Marshall later modified the Hughes contract
to include a flight test model ion engine, primarily to determine whether a beam neutralization problem existed
in space.
• On Aug. 1, 1961, NASA awarded a contract to the Astro-Electronics Division of RCA to design and
build a payload capsule for flight-testing electric propulsion engines.
• The program called for seven capsules, three for ground tests and four for actual flight tests. Each
capsule was expected to carry two electric engines.
• The first was expected to carry one cesium-fueled ion-engine representing Stuhlinger's design with
the Hughes engine.
• The second was expected to carry one mercury-fueled ion engine representing Kaufman's design
with the Lewis engine. Plans called for the engines to operate from 1 to 2 kW of power.
• Hughes demonstrated an ion engine on Sep. 27, 1961, at its research laboratories in Malibu.
Stuhlinger was among those on hand to greet the scientific and technical writers who attended the
event.

www.Vidyarthiplus.com
www.Vidyarthiplus.com

What happens to the ions once they leave the spacecraft?


• Positive ions shoot out of the back of DS1, making it move forward. At the same time, a beam of
electrons with negative charges is shot out of a cathode neutralizer. Things with positive and negative
charges are attracted to each other so the negative electrons fly towards the positive ions.
• Since a positive ion is an atom that is missing one or more electrons, when the electrons get to the
positive ions, they fill in the missing electron's position and make the ion back into a neutral atom.
• Once the ions have been neutralized, the atoms thus formed float off into the vast emptiness of
space.

www.Vidyarthiplus.com
www.Vidyarthiplus.com

Nuclear Propulsion

Introduction
For those who are interested in the exploration and development of space by humans, nuclear
propulsion technology is a very attractive option.
Why?
Compared with the best chemical rockets, nuclear propulsion systems (NPS's) are more reliable and
flexible for long-distance missions, and can achieve a desired space mission at a lower cost.
• The reason for these advantages in a nutshell is that NPS's can get "more miles per gallon" than a
chemical rockets.

• Nuclear propulsion includes a wide variety of propulsion methods that use some form of nuclear
reaction as their primary power source. Many military submarines, and, owing to crude prices and
emissions, a growing number of large civilian surface ships, especially icebreakers, use nuclear
reactors as their power plants (nuclear marine propulsion for civil use and nuclear navy for military
use). In addition, various types of nuclear propulsion have been proposed, and some of them tested,
for spacecraft applications:

• Antimatter catalyzed nuclear pulse propulsion


• Bussard ramjet

www.Vidyarthiplus.com
www.Vidyarthiplus.com

• Fission-fragment rocket
• Fission sail
• Fusion rocket
• Gas core reactor rocket
• Nuclear electric rocket
• Nuclear photonic rocket
• Nuclear pulse propulsion
• Nuclear salt-water rocket
• Nuclear thermal rocket
• Radioisotope rocket
• In a nuclear thermal rocket a working fluid, usually hydrogen, is heated to a high temperature in a
nuclear reactor, and then expands through a rocket nozzle to create thrust.
• The nuclear reactor's energy replaces the chemical energy of the reactive chemicals in a traditional
rocket engine.
• Due to the higher energy density of the nuclear fuel compared to chemical ones, about 10 7 times,
the resulting efficiency of the engine is at least twice as good as chemical engines even considering
the weight of the reactor, and even higher for advanced designs. The most traditional type uses a
conventional (albeit light-weight) nuclear reactor running at high temperatures to heat the working
fluid that is moving through the reactor core. This is known as the solid-core design, and is the
simplest design to construct.

A solid-core design
• The solid-core has the downside that it can only be run at temperatures below the melting point of the
materials used in the reactor core.

www.Vidyarthiplus.com
• Since the efficiency of a rocket engine is strongly related to the temperature of the working fluid, the
solid-core design needs to be constructed of materials that remain strong at as high a temperature as
possible.
• Even the most advanced materials melt at temperatures below that which the fuel can actually create,
meaning that much of the potential energy of the reactions is lost.
• Usually, with hydrogen propellant the solid-core design is expected to deliver specific impulses (Isp) on
the order of 800 to 900 seconds, about twice that of liquid hydrogenoxygen designs such as the Space
Shuttle main engine.
• Other propellants are sometimes proposed such as water or LOX; although they would provide reduced
performance, their greater availability can reduce payload costs where the mission delta-v is not too
high, for example within its lunar space or between Earth orbit and Martian orbit.
• The weight of a complete nuclear reactor is so great that solid-core engines would be hard-pressed to
achieve a thrust-to-weight ratio of 1:1, which would be needed to overcome the gravity of the Earth on
launch.
• Nevertheless the overall weight of the engine and fuel for a given amount of total impulse is lower.
• This means that solid-core engines are only really useful for upper-stage uses where the vehicle is
already in orbit, or close to it, or launching from a lower gravity planet, moon or minor planet where the
required thrust is lower.
• To be a useful Earth launch engine, the system would have to be either much lighter, or provide even
higher specific impulse. Both would, of course, be even better.

Sketch of nuclear thermal rocket

The Advantage of Nuclear Propulsion Systems


• Nuclear propulsion systems have the ability to overcome the Isp limitations of chemical rockets because
the source of energy and the propellant are independent of each other.
• The energy comes from a critical nuclear reactor in which neutrons split fissile isotopes, such as 92-U-
235 (Uranium) or 94-Pu-239 (Plutonium), and release energetic fission products, gamma rays, and
enough extra neutrons to keep the reactor operating.
• The energy density of nuclear fuel is enormous.
• For example, 1 gram of fissile uranium has enough energy to provide approximately one
megawatt (MW) of thermal power for a day.
• The heat energy released from the reactor can then be used to heat up a low-molecular weight propellant
(such as hydrogen) and then accelerate it through a thermodynamic nozzle in same way that chemical
rockets do. This is how nuclear thermal rockets (NTR's) work.
• There are two main types of NTR's: solid core and gas core.
www.Vidyarthiplus.com

• Solid-core NTR's (See Figure) have a solid reactor core with cooling channels through which the
propellant is heated up to high temperatures (2500-3000 K).
• Although solid NTR's don't operate at temperatures as high as some chemical engines
(due to material limitations), they can use pure hydrogen propellant which allows higher
Isp's to be achieved (up to 1000 s), since Isp is approximately 1/M propellant0.5, where Mpropellant is the
molecular weight of the propellant.

Schematic Diagram of a Solid Nuclear Thermal Rocket (NTR) Engine

• In gas-core NTR's, the nuclear fuel is in gaseous form and is inter-mixed


with the hydrogen propellant.
• Gas core nuclear rockets (GCNR) can operate at much higher temperatures
(5000 - 20000
K), and thus achieve much higher Isp's (up to 6000 s).
• Of course, there is a problem in that some radioactive fission products will
end up in the exhaust, but other concepts such as the nuclear light bulb
(NLB) can contain the uranium plasma within a fused silica vessel that easily
transfers heat to a surrounding blanket of propellant.
• At such high temperatures, whether an open-cycle GCNR, or a closed-cycle
NLB, the propellants will dissociate and become partially ionized.
• In this situation, a standard thermodynamic nozzle must be replaced by a
magnetic nozzle which uses magnetic fields to insulate the solid wall from
the partially-ionized gaseous exhaust.
• NTR's give a significant performance improvement over chemical engines,
and are desirable for interplanetary missions.
• It may also be possible that solid core NTR's could be used in a future launch
vehicle to supplement or replace chemical engines altogether.
• Advances in metallurgy and material science would be required to improve
the durability and T/W ratio of NTR's for launch vehicle applications.
• An alternative approach to NTR's is to use the heat from nuclear reactor to
generate electrical power through a converter, and then use the electrical
power to operate various types of electrical thrusters (ion, hall-type, or
magneto-plasma-dynamic (MPD)) that operate on a wide variety of
propellants (hydrogen, hydrazine, ammonia, argon, xenon, fullerenes).
• This is how nuclear-electric propulsion (NEP) systems work.
To convert the reactor heat into electricity, thermoelectric or thermionic devices could be used, but these
have low efficiencies and low power to weight ratios.
• The alternative is to use a thermodynamic cycle with either a liquid metal
(sodium, potassium), or a gaseous (helium) working fluid.

www.Vidyarthiplus.com
www.Vidyarthiplus.com

• These thermodynamic cycles can achieve higher efficiencies and power to


weight ratios.
• No matter what type of power converter is used, a heat rejection system is
needed, meaning that simple radiators, heat pipes, or liquid-droplet radiators
would be required to get rid of the waste heat.
• Unlike ground-based reactors, space reactors cannot dump the waste heat
into a lake or into the air with cooling towers.
• The electricity from the space nuclear reactor can be used to operate a
variety of thrusters.
• Ion thrusters use electric fields to accelerate ions to high velocities.
• In principle, the only limit on the Isp that can be achieved with ion thrusters
is the operating voltage and the power supply.
• Hall thrusters use a combination of magnetic fields to ionize the propellant
gas and create a net axial electric field which accelerates ions in the thrust
direction.
• MPD thrusters use either steady-state or pulsed electromagnetic fields to
accelerate plasma (a mixture of ions and electrons) in the thrust direction.
• To get a high thrust density, ion thrusters typically use xenon, while Hall
thrusters and MPD thrusters can operate quite well with argon or hydrogen.
• Compared with NTR's, NEP systems can achieve much higher Isp's. Their
main problem is that they have a low power to weight ratio, a low thrust
density, and hence a very low T/W ratio.
• This is due to the mass of the reactor, the heat rejection system, and the
low-pressure operating regime of electrical thrusters.
• This makes NEP systems unfeasible for launch vehicle applications and
mission scenarios where high accelerations are required; however, they can
operate successfully in low-gravity environments such as LEO and
interplanetary space.

www.Vidyarthiplus.com
www.Vidyarthiplus.com

• A higher exhaust velocity is desired, because less propellant will be required for a
given space mission.
• A performance parameter that rocket engineers like to use is the specific impulse
(Isp) which is simply the ratio of the thrust to the weight consumption rate of propellant
• (Isp = F / ( dm/dt * g) = vexhaust / g , g=9.81 m/s2)
• for example, if you have an engine with a specific impulse of 500 seconds, that means
you have an exhaust velocity of 4905 m/s and your engine will produce 500 pounds
of force for every pound of propellant it consumes per second.

Figure 1 shows a plot of the mass ratio (Minitial / Mfinal) for various mission ∆V's and specific impulses.
It is pretty clear from this diagram that a high specific impulse is desired to minimize the propellant
requirements. The specific impulse, Isp, is a rocket engine's equivalent of an automobile's "miles per
gallon" rating.

Limitations of Chemical Rocket Engines


In chemical rocket engines, such as the Space Shuttle Main Engine (SSME), the chemical reaction between
the hydrogen and oxygen releases heat which raises the combustion gases (steam and excess hydrogen
gas) up to high temperatures (3000-4000 K).
• These hot gases are then accelerated through a thermodynamic nozzle, which
converts thermal energy into kinetic energy, and hence provides thrust.
• The propellant and the heat source are one in the same. Because there is a limited
energy release in chemical reactions and because a thermodynamic nozzle is being
used to accelerate the combustion gases that do not have the minimum possible
molecular weight, there is a limit on the exhaust velocity that can be achieved.

www.Vidyarthiplus.com
www.Vidyarthiplus.com

• The maximum Isp that can be achieved with chemical engines is in the range of 400
to 500 s. So, for example, if we have an Isp of 450 s, and a mission delta-V of 10 km/s
(typical for launching into low earth orbit (LEO)), then the mass ratio will be 9.63.
• The problem here is that most of the vehicle mass is propellant, and due to limitations
of the strength of materials, it may be impossible to build such a vehicle to just to
ascend into orbit.
• Early rocket scientists got around this problem by building a rocket in stages, throwing
away the structural mass of the lower stages once the propellant was consumed.
• This effectively allowed higher mass ratios to be achieved, and hence a space
mission could be achieved with low-Isp engines. This is what all rockets do today,
even the Space Shuttle.
• In spite of the relatively low Isp, chemical engines do have a relatively high thrust-
toweight ratio (T/W)2.
• A high T/W (50-75) is necessary for a rocket vehicle to overcome the force of gravity
on Earth and accelerate into space.
• The thrust of the rocket engines must compensate for the weight of the rocket
engines, the propellant, the structural mass, and the payload.
• Although it is not always necessary, a high T/W engine will allow orbital and
interplanetary space vehicles to accelerate quickly and reach there destinations in
shorter time periods.
Fission and fusion
• Fission and fusion are different types of nuclear reactions in which energy is released from the high-powered
bonds between particles in the atomic nucleus.
• The atomic nucleus is most stable when binding energies between particles are strongest.
• This occurs with iron and nickel. For lighter atomic nuclei, energy can be extracted by combining these nuclei
together, a process known as nuclear fusion.
• For nuclei heavier than those of iron or nickel, energy can be extracted by splitting them apart in a process called
nuclear fission.
• Because the binding force in the atomic nucleus contains enormous energy, fission and fusion can both provide
tons of power, in principle. However, practical considerations make the exploitation of nuclear power more difficult
than something as simple as starting a fire.
• For fission, highly purified feedstock, usually uranium or plutonium isotopes, must be used. Isotopes are favored
because their instability makes them easier to break apart. The purification of these isotopes is extremely
expensive and requires multimillion-dollar centrifuges.
• In fusion, an extremely high threshold energy must be reached to combine atomic nuclei. In nature, the
only place where this occurs is in the core of a star. The temperature required is in the millions of degrees.
Superheated plasma and the focusing of laser power are two methods to achieve this threshold energy.

www.Vidyarthiplus.com
www.Vidyarthiplus.com

• The fission reactions which we use to generate energy in nuclear power plants usually require an input
of energy to start. The most common reaction is the splitting of the nucleus of specific types uranium
atoms.
• To start nuclear fusion large amounts of energy are needed to force two atomic nuclei together. When
the nuclei come together energy is released. Depending on the reaction that energy might be more or
less than the energy required to initiate the reaction.
• Nuclear fusion is the reaction that ultimately drives the universe. Stars combine atomic nuclei, which
releases the energy that we experience as heat and light. The most common reaction in stars is the
combination of hydrogen nuclei to form helium. The energy input to cause this reaction to occur comes
from the massive gravity in the star that overcomes all other forces to push atomic nuclei together.
• The attempts to generate energy on Earth from nuclear fusion are attempts to hold hydrogen in such
high-energy environments to force this reaction to take place in a controlled manner. Thus far scientists
have been unable to create such a controlled environment that is sustainable.
• There are many more fusion reactions in stars beyond the hydrogen to helium reaction, however. Deep
within stars the pressures due to the gravity of the star push all kinds of nuclei together to form more and
more complex atoms. It is believed that the source of all of the elements in the universe beyond hydrogen
is due to such reactions deep within stars.
• Sometimes the elements that are created in the centers of the stars are unstable outside of that high-
energy environment. When released from the center of stars through numerous types of cosmological
incidents, these unstable atoms will start to break down or undergo fission. Some of these elements are
so unstable that they will break down so fast we will never observe them in nature. Others are only slightly
unstable and will last for millions or billions of years, on average. These are the radioactive elements that
we are familiar with for fission reactions.
• One example of such an unstable atom is uranium-235. So named because of the number of neutrons
and protons in its nucleus, U-235 survives, on average, about a billion years. This is known as its half-
life. If you have one pound of U-235 it will naturally decay over about a billion years so that there is only
a half of a pound of U-235 after that time and the rest is changed to something else.
• If we add energy to U-235, however, in the form of forcing another neutron into the nucleus of the atom,
we create uranium 236 which is highly unstable and will break down very quickly, releasing energy in the
process. This is the basis of the reaction in nuclear power plants. When a neutron of a specific energy is
forced into the nucleus the atom will become unstable and it will split into two atoms with smaller nuclei.
When that occurs, neutrons are released that are just the right energy to split additional atoms, causing
what is known as a "chain reaction". This chain reaction will continue until many, many more atoms are
split, releasing large amounts of energy.
• Controlled nuclear fusion is the "Holy Grail" of energy production. Fusion reactions combine relatively
common isotopes of hydrogen into helium. The results of these reactions are stable atoms that don't
undergo further reactions. Fission power plants, the ones that we are using now, however, use rare
isotopes of unstable elements. The resulting atoms that are left over are also unstable. Those unstable
atoms release radiation and must be carefully handled.

www.Vidyarthiplus.com
www.Vidyarthiplus.com

• As you can see, while both are nuclear reactions, there are enormous differences between fusion and
fission. The study of both reactions is in the realm of nuclear physics and both can tell us a lot about the
nature of the universe.

Solar Sail
Introduction

Solar Sails are Spacecraft which utilize the momentum transfer of solar photons onto large, highly
reflecting sails for passive propulsion. The idea of sailing is not new: Ziolkovsky (1921) and Zander (1924)
introduced this 'exotic' idea for propulsion in space. The advantage is obvious: Solar Sails do not need
to carry an active main propulsion system nor any propellant for it. Therefore, extended missions
in our solar system and beyond seem possible. Through the continuous low thrust, trajectories could be
realized which would allow planetary missions with high science priority to be performed. High energetic
missions in particular, such as a Mercury orbiter mission, main belt asteroid and comet rendezvous as
well as sample return missions could be realized within reasonable flight times.
• Some of these missions can not be reasonably performed with conventional propulsion. Against common
impression, the low thrust level, moreover, does not necessarily increase flight time when compared to a
conventional, chemically propelled spacecraft. For missions with high energy requirement very often
multiple planetary flybys have to be introduced for chemical propulsion in order to allow the mission to
be feasible, which in most cases increases flight time. Solar sail spiral trajectories do not necessarily
require these gravity assists and can cut down trip times.

www.Vidyarthiplus.com
www.Vidyarthiplus.com

Solar sails

www.Vidyarthiplus.com
www.Vidyarthiplus.com

www.Vidyarthiplus.com
www.Vidyarthiplus.com

The goals of the study were: to evaluate the benefits of solar sails for planetary missions; to develop a solar
sail technology roadmap; and, to study the feasibility of a joint DLR/NASA solar sail demonstration mission
that could be performed at low-cost.

www.Vidyarthiplus.com
www.Vidyarthiplus.com

Artist's Impression; Solar Sail scenario in Mars Orbit

Feasibility Study Results


• The joint study concluded that a number of mid and far-term missions could be enhanced or enabled
using solar sails. The study also concluded that a low-cost technology demonstration mission in Earth
orbit is feasible. A demonstration mission is the recommended approach for the development of this
advanced propulsion concept. This would demonstrate and validate the basic principles of sail
fabrication, packaging, storage, deployment, and control.
• The mission scenario comprises a “piggyback” launch of a small spacecraft plus solar sail with a total
mass of less than 100kg on an ARIANE 5 into a geosynchronous transfer orbit, where a 40m x 40m sail
would be deployed. The aluminized sail film is to be supported by deployable light-weight composite
structure booms which are currently being developed at the Institute of Structural Mechanics in
Braunschweig. By proper orientation of the sail towards the Sun during each orbit, the orbital energy
would increase, such that after roughly 500 days the orbital radius of the Moon would be achieved. On-
board cameras are foreseen to observe the sail deployment. An additional science payload could provide
remote sensing data of the Earth and also of previously not very well explored lunar areas.

• Solar sails (also called light sails or photon sails, especially when they use light sources other than
the Sun) are a proposed form of spacecraft propulsion using large membrane mirrors.

www.Vidyarthiplus.com
www.Vidyarthiplus.com

• Radiation pressure is about 10-5 Pa at Earth's distance from the Sun and decreases by the square of the
distance from the light source (e.g. sun), but unlike rockets, solar sails require no reaction mass.
• Although the thrust is small, it continues as long as the light source shines and the sail is deployed.
• Radiation pressure is the pressure exerted upon any surface exposed to electromagnetic radiation. If
absorbed, the pressure is the power flux density divided by the speed of light. If the radiation is totally
reflected, the radiation pressure is doubled. For example, the radiation of the Sun at the Earth has a
power flux density of 1,370 W/m2, so the radiation pressure is 4.6 µPa (absorbed)
• In theory a lightsail (actually a system of lightsails) powered by an Earth-based laser could even be used
to decelerate the spacecraft as it approaches its destination
• Solar collectors, temperature-control panels and sun shades are occasionally used as expedient solar
sails, to help ordinary spacecraft and satellites make minor attitude control corrections and orbit
modifications without using fuel.
• This conserves fuel that would otherwise be used for maneuvering and altitude control. A few have even
had small purpose-built solar sails for this use.
• For example, EADS Astrium's Eurostar E3000 geostationary communications satellites use solar sail
panels attached to their solar cell arrays to off-load transverse angular momentum, thereby saving fuel
(angular momentum is accumulated over time as the gyroscopic momentum wheels control the
spacecraft's attitude - this excess momentum must be offloaded to protect the wheels from over spin).
• The science of solar sails is well-proven, but the technology to manage large solar sails is still
undeveloped.
• Mission planners are not yet willing to risk multimillion dollar missions on unproven solar sail unfolding
and steering mechanisms.
• This neglect has inspired some enthusiasts to attempt private development of the technology, such as
the Cosmos 1.
• The concept was first proposed by German astronomer Johannes Kepler in the seventeenth century.
• It was again proposed by Friedrich Zander in the late 1920s and gradually refined over the decades.
• Serious interest in lightsails began with an article by engineer and science fiction author Robert L.
Forward in 1984.
How Solar Sail works
• The spacecraft arranges a large membrane mirror which reflects light from the Sun or some other source.
• The radiation pressure on the mirror provides a small amount of thrust by reflecting photons.
• Tilting the reflective sail at an angle from the Sun produces thrust at an angle normal to the sail.
• In most designs, steering would be done with auxiliary vanes, acting as small solar sails to change the
attitude of the large solar sail (see the vanes on the illustration labeled Cosmos 1, below).
• The vanes would be adjusted by electric motors.
• In theory a lightsail driven by a laser or other beam from Earth can be used to decelerate a spacecraft
approaching a distant star or planet, by detaching part of the sail and using it to focus the beam on the
forward-facing surface of the rest of the sail.
• In practice, however, most of the deceleration would happen while the two parts are at a great distance
from each other, and that means that, to do that focusing, it would be necessary to give the detached
part an accurate optical shape and orientation.

www.Vidyarthiplus.com
www.Vidyarthiplus.com

• Sails orbit, and therefore do not need to hover or move directly toward or away from the sun. Almost all
missions would use the sail to change orbit, rather than thrusting directly away from a planet or the sun.
The sail is rotated slowly as the sail orbits around a planet so the thrust is in the direction of the orbital
movement to move to a higher orbit or against it to move to a lower orbit. When an orbit is far enough
away from a planet, the sail then begins similar maneuvers in orbit around the sun.
• The best sort of missions for a solar sail involves a dive near the sun, where the light is intense, and sail
efficiencies are high. Going close to the Sun may be done for different mission aims: for exploring the
solar poles from a short distance, for observing the Sun and its near environment from a non-Keplerian
circular orbit the plane of which may be shifted some solar radii, for flying by the Sun such that the sail
gets a very high speed.
• An unsuspected feature, until the first half of the 1990s, of the solar sail propulsion is to allow a sailcraft
to escape the solar system with a cruise speed higher or even much higher than a spacecraft powered
by a nuclear electric rocket system. The spacecraft mass-to-sail area ratio does not need to achieve
ultra-low values, even though the sail should be an advanced all-metal sail. This flight mode is also known
as fast solar sailing. Proven mathematically (like many other astronautical items well in advance of their
actual launches), such sailing mode has been considered by NASA/Marshall as one of the options for a
future precursor interstellar probe exploring the near interstellar space beyond the heliosphere. [5]
• Most theoretical studies of interstellar missions with a solar sail plan to push the sail with a very large
laser Beam-powered propulsion Direct Impulse beam. The thrust vector (spatial vector) would therefore
be away from the Sun and toward the target.

www.Vidyarthiplus.com
www.Vidyarthiplus.com

Nozzleless Propulsion
Introduction
The concept of nozzleless rocket motors stems from the possibility of obtaining the required booster
configuration in an integrated rocket ramjet type of vehicle. Despite the fact that expansion of the chamber
gases cannot be effected in nozzleless system as efficiently as in nozzled rockets, an overall gain is still possible
because of higher propellant loading in the given volume. Typically a performance gain of upto 15% has been
projected by ISRO. In addition the system becomes much simpler leading to higher reliability.
The most critical aspects of a nozzleless system are the choice of propellant and geometric configuration
supplemented with a detailed understanding of erosive burning at lateral velocities extending into supersonic
flow regime. The theoretical modeling of such systems has not yet come to a proven state.
Earlier studies about the properties of a propellant suitable for nozzleless configuration have shown stringent
demands on the burning rate characteristics, mechanical strength and strain capacity.
Nozzleless propulsion system, the most advanced concept was successfully demonstrated by AFRPL, USA
in 1979. It was expected that such a system will reduce the cost of production in view of the elimination of the
nozzle assembly, the reduction of the case insulation requirements etc., and also will improve the reliability on
account of the system simplicity.

The properties required for an ideal propellant for a nozzleless configuration shows that:-

1. The burning rate of the propellant must be higher in order to prevent a long and inefficient tail off.
2. A superior strain capacity at low temperature in order to allow optimum web fraction, propellant loading
and total performance is required and,
3. A higher stress capacity at high temperature so that the grain structure can survive the shear loads
produced by the large axial pressure differential between the head end and aft-end of a burning
nozzleless grain is also required.

The above requirements reported by the ISRO shows that the cost of development and propellant ingredient
are typically higher for nozzleless motor. However, it is reported that inspite of these factors, a straight forward
nozzleless booster can be designed to yield comparable performance at a price reduction of about 10% and
a performance gain of upto 15% if the propellant strength and burning-rate/pressure exponent can be
optimized.

Nozzleless Propulsion System


In the conventional solid rocket motor, the propellant is burnt inside a rocket chamber and the hot
gases thus generated are accelerated to supersonic condition through a convergent-divergent type
nozzle. The heat energy of the gases is converted into kinetic energy inside the nozzle.

www.Vidyarthiplus.com
www.Vidyarthiplus.com

Nozzleless propulsion system attempts to perform the main task of the nozzle inside the grain port
itself, thereby saving the weight of the nozzle which can be replaced by additional propellant. The basic
configuration of such system is shown below.

On ignition of the grain, high volume of gas will be generated in the star portion of the grain (section AA)
because of large surface area available. This gas will converge at the section CC and will flow through
the tubular portion of the grain at section BB.

The gas is expected to reach sonic condition at the interface of the conical and cylindrical section and accelerate
to supersonic condition inside the tubular portion.

Simultaneously, small amount of mass addition will take place inside the tubular portion due to the burning,
which will progressively increase as a function of time.

Due to the existence of long tubular portion, erosive burning of the grain in the tubular portion is expected.
This will be further accelerated due to attainment of supersonic condition of the gas.

Figures 2 shows the different phases of grain burning and the formation of nozzle shape due to the erosive
burning inside the grain as described above.

After the completion of the burning of the star portion, the gas will continue to be generated (much lesser
quantity) by the conical portion of the grain, more or less as an end burning system. However, mass
addition at the tubular portion will continue.

This is expected to function till the propellant burns completely at the aft end section due to erosive burning.

www.Vidyarthiplus.com
www.Vidyarthiplus.com

Beyond this time, a nozzle hardware system will be necessary. This will be provided with a silica phenolic throat
insert embedded inside the grain as shown in the figure.

Additional Reading:
• Sutton, G.P., “Rocket Propulsion Elements”, John Wiley & Sons Inc., New York, 7 th Edn., 2001.

www.Vidyarthiplus.com

Das könnte Ihnen auch gefallen